You are on page 1of 139

PROFESSIONAL EDUCATION

(Secondary)
MULTIPLE CHOICE
1.
Which does Naom Chomsky
assert about language learning for
children?
I.
Young children learn and
apply grammatical rules and
vocabulary as they are exposed to
them.
II.
Begin formal teaching of
grammatical rules to children as
early as possible.
III.
Do not require initial formal
language teaching for children.
A. I and III
C. I only
B. II only
D. I and II
2.
Which is/are the sources of
mans intellectual drives, according
to Freud
A.
Id
C. Id -ego
B.
Super ego D. Ego
3.
Theft of school equipment
like TV, computer, etc. by
teenagers in the community itself
is becoming a common
phenomenon. What does this
incident signify?
A.
Prevalence of poverty in the
community.
B.
Inability of school to hire
security guards.
C.
Deprivation of Filipino
schools.

D.
Communitys lack of sense of
co-ownership.
4.
As a teacher, you are
reconstructionist, Which among
these will be your guiding
principle?
A.
I must teach the child every
knowledge, skills and value that he
needs for a better future.
B.
I must teach the child to
develop his mental powers to the
full.
C.
I must teach the child so he
is assured of heaven.
D.
I must teach the child that
we can never have real knowledge
of anything.
5.
The concepts of Trust vs.
mistrust, autonomy vs. shame &
self-doubt, and initiative vs. guilt
are most closely related with works
of _____.
A. Erickson C. Freud
B. Piaget D. Jung
6.
Student Z does not study at
all but when the Licensure
Examination for Teachers comes,
before he takes the LET, he spends
one hour or more praying for a
miracle, i.e., to pass the
examination. Which attitude
towards religion or God is
displayed?

A.
Religion as fake C. Religion as
authentic
B.
Religion as magic D. Religion
as real
7.
Teacher B engages her
students with information for
thorough understanding, for
meaning and for competent
application. Which principle
governs Teacher Bs practice?
A Constructivist C. Behaviorist
B. Gestalt
D. Cognitivist
8.
Student B claims: I cannot
see perfection but I long for it. So
it must be real. under which group
can he be classified?
A.
Idealist
C.
Realist
B. Empiricist D. Pragmatist
9.
What does extreme
authoritarianism in the home
reinforce in learners
A. Doing things on their own
initiative
B. Ability to direct themselves
C. Dependence on others for
directions.
D. Creativity in work.
10. You arrive at knowledge by
re-thinking of latent ideas. From
whom does this thought come?
A. Experimentalist C. Idealist
B. Realist
D.
Existentialist

11. Behavior followed by


pleasant consequences will be
strengthened and will be more
likely to occur in the future.
Behavior followed by unpleasant
consequences will be weakened
and will be more likely to be
repeated in the future. Which one
is explained
A.
Freuds psychoanalytic theory
B.
Thorndikes law effect
C.
B.F. Skinners Operant
conditioning theory
D.
D. Banduras social learning
theory
12. Principal B tells her teachers
that training in the humanities is
most important. To which
educational philosophy does he
adhere?
A.
Existentialism
C.
Progressivis
B.
Perennialism
D.
Essentialism
13. All subject in Philippine
elementary and secondary schools
are expected to be taught using
the integrated approach. This
came about as a result of
implementation of ___________.
A.
Program for Decentralized
Education
B.
School-Based Management
C.
Basic Education Curriculum

D.
D. Schools First Initiative
14. Principal C shares this
thought with the teachers: Subject
matter should help students
understand and appreciate
themselves as unique individual
who accept complete responsibility
for their thoughts, feelings and
actions. From which philosophy is
this thought based?
A. Perennialism C. Existentialism
B. Essentialism
D.
Progressivism
15. Based on Piagets theory,
what should a teacher provide for
children in the concrete operational
stage?
A.
Activities for hypothesis
formulation.
B.
Learning activities that
involve problems of classification
and ordering.
C.
Games and other physical
activities to develop motor skills.
D.
Stimulating environment with
ample objectives to play with.
16. To come closer to the truth we
need to go back to the things
themselves. This is the advice of
the ______.
A. Behavioris
C.
idealist
B. phenomenologists D.
pragmatists

17. Researches conducted show


that teachers expectations of
students become self-fulfilling
prophecies. What is this
phenomenon called?
A. Halo effect
C. Ripple
effect
B. Pygmalion effect
D.
Hawthorne effect
18. Teacher F is convinced that
whatever a student performs a
desired behavior, provided
reinforcement and soon the
student will learn to perform the
behavior on his own. On which
principle is Teacher Fs conviction
based?
A. Cognitivism
C.
Behaviorism
B. Environmentalism
D.
Constructivism
19. Teacher U teaches to his
pupils that pleasure is not the
highest good. Teachers teaching is
against what philosophy?
A. Realism
C. Epicureanism
B. Hedonism
D.
Empiricism
20. Studies in the areas of
neurosciences discloses that the
human brain has limitless capacity.
What does this imply
A.
Some pupils are admittedly
not capable of learning.

B.
Every pupil has his own
native ability and his learning is
limited to this native ability.
C.
Every child is a potential
genius.
D.
Pupil can possibly reach a
point where they have learned
everything.
21. Availment of the Philippine
Education Placement Test for adults
and out-of-school youths is in
support of the governments
educational program towards _____.
A. equitable access C. quality and
relevance
B.
quality
D.
relevance
22. A mother gives his boy his
favorite snack every time the boy
cleans up his room. Afterwards,
the boy cleaned his room everyday
in anticipation of the snack. Which
theory is illustrated?
A. Associative learning
B. Classical conditioning
C. Operant conditioning
D. Pavlonian conditioning
23. Which group of philosophers
maintain the truth exists in an
objective order that is independent
of the knower?
A. Idealists C. Existentialists
B. Pragmatists D. Realists
24. Under which program were
students who were not

accommodated in public
elementary and secondary schools
because of lack of classroom,
teachers, and instructional
materials, were enrolled in private
schools in their respective
communities at the governments
expense?
A.
Government Assistance
Program
B.
Study Now-Pay later
C.
Educational Service Contract
System
D. National Scholarship Program
25. Which of the following
prepositions is attributed to Plato?
A.
Truth is relative to a
particular time and place.
B.
Human beings create their
own truths.
C.
Learning is the discovery of
truth as latent ideas are brought to
consciousness
D.
Sense perception is the most
accurate guide to knowledge.
26. In a treatment for alcoholism,
Ramil was made to drink an
alcoholic beverage and then made
to ingest a drug that produces
nausea. Eventually, h nauseated at
he sight and smell of alcohol and
stopped drinking alcohol. Which
theory explains this?
A.Operant conditioning
C. Associative learning

B. Social learning theory


D. Attribution theory
27. In a social studies class,
Teacher I present a morally
ambiguous situation and asks his
students what they would do. On
whose theory is Teacher Is
technique based
A.
Kohlberg
C. Piaget
B.
Bandura
D. Bruner
28. Quiz to formative test while
periodic is to _________.
A. criterion-reference test
C. norm-reference test
B. summative test
D. diagnostic test
29. Your teacher is of the opinion
that the world and everything in it
are ever changing and so you the
skill to cope with change. What is
his governing philosophy?
A. Idealism
C. Experimentalism
B. Existentialism
D. Realism
30. The search for related
literature by accessing several data
bases by the use of a telephone
line to connect a computer that
have database is termed ______.
A.
compact disc search
C. online search
B. manual search
D.
computer search
31. In which competency do my
students find the greater difficulty?
In item with a difficulty index of

A. 0. 1
C. 0.5
B. 0. 9
D. 1.0
32. We encounter people whose
prayer goes like this: O God, if
there is a God; save my soul if I
have a soul, from whom is this
prayer?
A.
Stoic
C. Agnostic
B.
Empiricist D. Skeptic
33. If teacher wants to test
students ability to organize ideas,
which type of test should she
formulate?
A. Short answer
C. Essay
B. Technical problem type
D. Multiple-choice type
34. Who among the following
needs less verbal counseling but
needs more concrete operational
forms of assistance? The child who
________.
A.
has mental retardation
B.
has attention-deficit disorder
C.
has learning disability
D.
has conduct disorder
35. Which applies when
skewness is zero?
A.
Mean is greater than median.
B.
Median is greater than the
mean.
C.
Scores have three modes.
D.
Scores are normally
distributed 36. Which does NOT
belong to the group of alternative
learning systems

A. Multi-grade grouping
C. Graded education
B. Multi-age grouping
D. Non-graded grouping
37. The first thing to do in
constructing a periodic test is for a
teacher to ______.
A.
decide on the number of
items for the test
B.
go back to her instructional
objectives
C.
study the content
D.
decide on the type of test to
construct
38. Which one can enhance the
comparability of grades?
A.
Using common conversation
table for translating test scores in
to ratings.
B.
Formulating tests that vary
from one teacher to another.
C.
Allowing individual teachers
to determine factors for rating
D.
Individual teachers giving
weights to factors considered for
rating.
39. The cultivation of reflective and
meditative skills in teaching is an
influence of _____.
A. Shintoism
C.
Confucianism
B. Zen Buddhism D. Taoism
40. Teacher Y does normreferenced interpretation of scores.

Which of the following does she


do?
A.
She describes group
performance in relation to a level of
mastery set.
B.
She uses a specified content
as its frame of reference.
C.
She compares every
individual students scores with
others scores.
D.
She describes what should be
their performance.
41. The best way for a guidance
counselor to begin to develop
study skills and habits in
underachieving student would be
to ______.
A.
have these underachieving
students observe the study habits
of excelling students.
B.
encourage students to talk
study habits from their own
experiences
C.
have them view filmstrips
about various study approaches
D.
give out list of effective study
approaches
42. Teacher F wanted to teach the
pupils the skills to do cross
stitching. He check up quiz was a
written test on the steps of cross
stitching. What characteristic of a
good test does it lack?
A. Scorability
C.
Objectivity

B. Reliability
D. Validity
43. For which may you use the
direct instruction method?
A.
Become aware of the
pollutants around us.
B.
Appreciate Miltons Paradise
Lost.
C. Use a microscope properly.
D. Distinguish war from
aggression.
44. In the context on the theory on
multiple intelligences, what is one
weakness of the paper-pencil test?
A.
It is not easy to administer.
B.
It puts the non-linguistically
intelligent at a disadvantage
C.
It utilizes so much time.
D.
It lacks reliability.
45. NSAT and NEAT results are
interpreted against set mastery
level. This means that NSAT and
NEAT fall under _________.
A.
intelligence test C. criterionreferenced test
B. aptitude test D. normreferenced test
46. Teacher B uses the direct
instruction strategy. Which
sequence of steps will she follow?
I.
Independent practice
II.
Feedback and correctiveness
III.
Guided student practice
IV.
Presenting and structuring

V.
Reviewing the previous days
work
A. V-IV-III-II-I
C. V-II-IV-III-I
B. III-II-IV-I-V
D. I-V-II-III-IV
47. Which guideline must be
observed in the use of prompting
to shape the correct performance
of your students?
A.
Use the list intrusive prompt
first.
B.
Use all prompts available.
C.
Use the most intrusive
prompt first.
D.
Refrain form using prompts.
48. What measure/s of central
tendency do/es the number 16
represent in the following data:
14,15,17,16,19,20,16,14,16?
A.
Mode
C. Median
B.
Mode and medianD. Mean
49. Which holds true to
standardized tests?
A.
They are used for
comparative purposes.
B.
They are administered
differently.
C.
They are scored according to
different standards.
D.
They are used for assigned
grades.
50. Study this group of tests which
was administered with the
following results, then answer the
question.

Subject
Mean
SD
Ronnels scrore
Math 56
10
43
Physics
41
9
31
English
80
16
109
In which subject(s) were the scores
most homogenous?
A. Math
C. Physics
B. English D. Physics and Math
51. In the parlance of test
construction what does TOS mean?
A. Table of Specifics
C. Table
of Specific Test Items
B. Term of Specifications
D.
Table of Specifications
52. Which is a major advantage of
a curriculum-based assessment?
A.
It is informal nature.
B.
It connects testing with
teaching.
C.
It tends to focus on anecdotal
information on student progress.
D.
It is based on a normreferenced measurement model.
53. Teacher H gave first-grade class
a page with a story in which picture
take the place of some words.
Which method did she use?
A.
The whole language
approach
B.
The Spaulding method
C.
The rebus method
D.
The language experience
approach

54. Out of 3 distracters in a


multiple choice test items, namely
B, C, and D, no pupil choice D as
answer. This implies that D is
________.
A.
an ineffective distracter
B.
a vague distracter
C.
an effective distracter
D.
a plausible distracter
55. The burnout malady gets worse
if a teacher doesnt intervene to
change whatever areas he or she in
control. Which one can renew a
teachers enthusiasm?
A.
Stick to job
C. Judge
someone else as wrong
B.
Initiate changes in jobs
D. Engage in self-pity
56. If teacher has to ask more
higher-order questions, he has to
ask more ____ questions.
A.
closed
C. concept
B. Fact
D. convergent
57. What can be said of Peter who
obtained a score of P75 in a
Grammar objective test?
A.
His rating is 75 items in the
test correctly.
B.
He answered 75 % of the test
items correctly.
C.
He answered 75.
D.
He performed better that 5%
of his classmate.

58. I drew learners into several


content areas and encouraged
them to solve a complex question
for interdisciplinary teaching.
Which strategy did I use?
A.
Problem-centered learning
B.
Unit method
C.
Reading-writing activity
D.
D. Thematic instruction
59. Which guidance in test
construction is NOT observed in
this test item Jose Rizal wrote
____.
A.
The central problem should
be packed in the stem.
B.
There must be only one
correct answer.
C.
Alternatives must
grammatical parallelism.
D.
The alternative must be
plausible.
60. To elicit more students
response, Teacher G. made use of
covert responses. Which one did
she NOT do?
A.
She had the students write
their response privately.
B.
She showed the correct
answers on the overhead after the
students have written their
responses.
C.
She had the students write
their responses privately then
called each of them.

D.
She refrained from judging on
the students responses.
61. Direct instruction id a facts,
rules, and actions as indirect
instruction is for _____ ,____, _____.
A.
hypotheses, verified data and
conclusions
B.
concepts, patterns and
abstractions
C.
concepts, processes and
generalizations
D. guesses, data and conclusions
62. Which test has broad
sampling of topics as strength?
A. Objective test C. Essay test
B. Short answer test
D. Problem
test .
63. Teacher T taught a lesson
denoting ownership by means of
possessives. He first introduced
the rule, then gave examples,
followed by class exercises, then
back to the rule before he moved
into second rule. Which presenting
technique did he use
A.
Combinatorial
C. Partwhole
B.
Comparative
D.
Sequential
64. In his second item analysis,
Teacher H found out that more from
the lower group got the test item #
6 correctly. This means that the
test item _____.

A.
has a negative discriminating
power.
B.
has a lower validity.
C.
has a positive discriminating
power.
D.
has a high reability.
65. Teacher E discussed how
electricity flows through wires and
what generates the electric charge.
Then she gave the students wires,
bulbs, switches, and dry cells and
told the class to create a circuit
that will increase the brightness of
each bulb. Which one best
describes the approach used?
A.
It used a taxonomy of basic
thinking skills.
B.
It was constructivist.
C.
It helped students
understand scientific methodology.
D.
None of the above
66. Teacher W wants to review and
check on the lesson of the previous
day. Which one will be most
reliable?
A.
Having students identify
difficult homework problems.
B.
Having students correct each
others work.
C.
Sampling the understanding
of a few students.
D.
Explicitly reviewing the taskrelevant information necessary for
the days lesson.

67. Shown a picture of children in


sweaters inside the classrooms, the
students were asked this question:
In what kind of climate do these
children live? This is a thought
questions on ______.
A. inferring
C. applying
B. creating
D. predicting
68. Study this group of tests
which was administered with the
following results, then answer the
question.
Subject
Mean SD
Ronnels
scrore
Math 56
10
43
Physics
41
9
31
English
80
16
109
In which subject(s) did Ronnel
perform best in relation to the
groups performance?
A.Physics and Math
C. Physics
B.English
D. Math
69. Which criterion should guide a
teacher in the choice of
instructional devices?
A.
Attractiveness
C.
Novelty
B.
Cost
D.
Appropriateness
70. Study this group of tests which
was administered with the
following results, then answer the
question.
Subject
Mean
SD
Ronnels scrore

Math
56
10
43
Physics
41
9
31
English
80
16
109
In which subject(s) did Ronnel
perform poorly in relation to the
groups performance?
A.
English
C. Math
B.
English and Math D. Physics
71. What is most likely to
happen to our economy when
export continuously surpasses
import is a thought question on
______.
A. creating
C. relating causeand-effect
B. synthesizing D. predicting
72. Which method has been
proven to be effective in courses
that stress acquisition of
knowledge?
A. Socratic method
C. Indirect instruction
B. Cooperative learning
D. Mastery learning
73. Which is the first step in
planning an achievement test?
A.
Define the instructional
objective.
B.
Decide on the length of the
test.
C.
Select the type of test items
to use.

D.
Build a table of specification.
74. Which activity should a teacher
have more for his students if he
wants to develop logicalmathematical thinking?
A.
Drama
C. Problem
solving
B.
Choral reading
D.
Storytelling
75. Why should a teacher NOT use
direct instruction all the time
A.
It requires much time.
B.
It requires use of many
supplemental materials.
C.
It is generally effective only
in the teaching of concepts and
abstractions.
D.
It reduces students
engagement in learning.
76. Which are direct measures of
competence?
A.
Personality tests C. Paperand-pencil test
B.
B. Performance tests D.
Standardized tests
77. Which is one role play in the
pre-school and early childhood
years?
A.
Develops competitive spirit.
B.
Separates reality from
fantasy.
C.
Increase imagination due to
expanding knowledge and
emotional range.

D.
Develops the upper and
lower limbs.
78. Teacher A discovered that his
pupils are very good in
dramatizing. Which tool must have
helped him discover his pupils
strength?
A.
Portfolio assessment
C.
Journal entry
B.
Performance test
D.
Paper-and-pencil test
79. Teacher Ms pupils are quite
weak academically and his lesson
is already far behind time table.
How should Teacher M proceed with
his lesson?
A. Experientially C. Logically
B. Inductively
D.
Deductively
81. I want to teach concepts,
patterns and abstractions. Which
method is most appropriate?
A.
Indirect instruction
C.
Direct instruction
B.
Discovery
D. Problem
solving
82. If your Licensure Examination
for Teacher (LET) items sample
adequately the competencies listed
in the syllabi, it can be said that
LET possesses ______ validity.
A.
concurrent
C. content
B.
construct
D.
predictive

83. Read the following then


answer the questions:
TEACHER: IN WHAT WAYS OTHER
THAN THE PERIODIC TABLE MIGHT
WE PREDICT THE UNDISCOVERED
ELEMENTS?
BOBBY:
WE COULD GO TO THE
MOON AND SEE IF THERE ARE
SOME ELEMENTS THERE WE DONT
HAVE.
BETTY:
WE COULD DIG DOWN
TO THE CENTER OF THE EARTH
AND SEE IF WE FIND ANY OF THE
MISSING ELEMENTS.
RICKY:
WE COULD STUDY
DEBRIS FROM THE METEORITES-IF
WE CAN FIND ANY.
TEACHER: THOSE ARE ALL GOOD
ANSWERS. BUT WHAT IF THOSE
EXCURSIONS TO THE MOON, TO
THE CENTER OF THE EARTH OR TO
FIND METEORITES WERE TOO
COSTLY AND TIME CONSUMING?
HOW MIGHT WE USE THE
ELEMENTS WE ALREADY HAVE
HERE ON EARTH TO FIND SOME
NEW ONES?
Question: The Teacher questions in
the above exchange are examples
of ____ questions.
A.
Fact
C. Direct
B.
Concept
D. Closed
84. In Krathwohs taxonomy of
objectives in the affective, which is
most authentic?

A.Characterization
C.
Responding
B. Valuing
D. Organization
85. In the light of the facts
presented, what is most likely to
happen when? is a simple
thought question on _____.
A. inferring
C.
synthesizing
B. generalizing
D.
justifying
86. The teachers first task in the
selection of media in teaching is to
determine the
A.
choice of the students
B.
availability of the media
C.
objectives of the lesson
D.
technique to be used
87. In self-directed learning, to
what extent should a teachers
scaffolding be?
A.
To a degree the student
needs it.
B.
None, to force the student to
learn by himself
C.
To the maximum, in order to
extend to the student all the help
he needs.
D.
To the minimum, to spend up
development of students sense of
independence.
88. Which is a form of direct
instruction?
A.
Discovery process
C. Programmed instruction

B. Problem solving
D. Inductive reasoning
89. With synthesizing skills in mind,
which has the highest diagnostic
value?
A.
Essay test
C.
Completion tes
B.
Performance test
D.
Multiple choice test
90. How can you exhibit legitimate
power on the first day of school?
A.
By making your students feel
they are accepted for who they are.
B.
By informing them you are
allowed to act in loco parents.
C.
By making them realize the
importance of good grades.
D.
By making them feel you
have mastery of subject matter.
91. Based on Edgar Dales Cone of
Experience, which activity is
farthest from the real thing?
A. View images
C. Watch a
demo
B. Attend exhibit D. Hear
92. Which can effectively measure
students awareness of values?
A.
Projective techniques C.
Likert scales
B.
Moral dilemma
D.
Anecdotal record
93. I combined several subject
areas in order to focus on a single
concept for interdisciplinary

teaching. Which strategy/method


did I use?
A.
Problem-entered learning
B.
Thematic instruction
C.
Readingwriting activity
D.
Unit method
94. The test item Group the
following items according to shape
is a thought test item on _______.
A.
creating
C. classifying
B. generating
D.
comparing
95. For maximum interaction, a
teacher ought to avoid _____
questions.
A.
informational
C.
leading
B.
rhetorical D. divergent
96. By what name is Socratic
method also known ?
A.
Mastery learning
C.
Morrison method
B.
Indirect instruction
D.
Questioning Method
97. Which is an appropriate way
to make manage off-task behavior?
A.
Make eye contact.
B.
Stop your class activity to
correct a child who is no longer on
task.
C.
Move closer to the child.
D.
Redirect a childs attention to
task and check his progress to
make sure he is continuing to work

98. Which one can best evaluate


students attitudinal development?
A.
Essay test C. Observation
B. Portfolio
D. Short answer
test
99. What should a teacher do for
students in his class who are on
grade level
A.
Give them materials on their
level and let them work at a pace
that is reasonable for them, trying
to bring them up to a grade level.
B.
Give them the same work as
the other students, because they
will absorb as much as they are
capable of.
C.
Give them the same work as
the other students, not much, so
that they wont feel embarrassed.
D.
Give them work on the level
of the other students and work a
little above the classmates level to
challenge them.
100. With-it-ness, according to
Kourin, is one of the characteristics
of an effective classroom manager.
Which phrase goes with it?
A.
Have hands that write fast.
B.
Have eyes on the back of
your hands.
C.
Have a mouth ready to
speak.
D.
Have minds packed with
knowledge.

101. On whose philosophy was


A.S.Neil Summerhill, one of the
most experimental schools based?
A.
A. Rousseau
C.
Montessori
B.
B. Pestalozzi
D. John
Locke
102. Which Filipino trait works
against the shift in teachers role
from teacher as a fountain of
information to teacher as
facilitator?
A. Authoritativeness
C. Hiya
B.Authoritarianism
D.
Pakikisama
103. Teacher A is a teacher of
English as Second Language: she
uses vocabulary cards, fill-in-the
blank sentences, dictation and
writing exercises in teaching a
lesson about grocery shopping.
Based on this information, which of
the following is a valid conclusion?
A.
The teacher is applying
Blooms hierarchy of cognitive
learning.
B.
The teacher is teaching
variety of ways because not all
students learn in the same manner.
C.
The teacher wants to make
her teaching easier by having less
talk.
D. The teacher is emphasizing
reading and writing skills.

104. With specific details in mind,


which one has (have) a stronger
diagnostic value?
A.
Multiple choice test
B.
Non-restricted essay test
C.
Restricted essay test
D. Restricted and non-restricted
essay tests
105. Teacher B is a teacher of
English as Second Language: she
uses vocabulary cards, fill-in-the
blank sentences, dictation and
writing exercises in teaching a
lesson about grocery shopping.
Based on this information, which of
the following is a valid conclusion?
A.
The teacher is reinforcing
learning by giving the same
information in a variety of
methods.
B.
The teacher applying Blooms
hierarchy of cognitive learning.
C.
The teacher is wants to do
less talk.
D.
The teacher is emphasizing
listening and speaking skills.
106. In a criterion-referenced
testing, what must you do to
ensure that your test is fair
A.
Make all of the questions true
or false.
B.
Ask each student to
contribute one question.

C.
Make twenty questions but
ask the students to answer only ten
of their choice.
D.
Use objectives for the unit as
guide in your test construction.
107. Read this question: How will
you present the layers of the earth
to your class? This is a question
that _________.
A.
directs
B. leads the students to evaluate
C. assesses cognition
D. probes creative thinking
108. In Krathwohs affective
domain of objectives, which of the
following is the lowest level of
effective behavior?
A.
Valuing
C.
Responding
B. Characterization
D.
Organization
109. Which is NOT a sound purpose
for asking questions?
A.
To probe deeper after an
answer is given
B.
To discipline a bully in class
C.
To remind students of a
procedure
D.
To encourage self-reflection
110. It is not wise to laugh at a
two-year old child when he utters
bad word because in his stage he is
learning to __________.
A.
considered others views
B.
distinguish right from wrong

C.
socialize
D.
distinguish sex differences
111. Research tells that teachers
ask mostly content questions.
Which of the following terms does
NOT refer to content questions?
A.
Closed
C. Concept
B.
Direct
D. Convergent
112. In mastery learning, the
definition of an acceptable
standard of performance is called a
A.
behavior
C. SMART
B.
condition D. criterion
measure
113. Based on Edgar Dales Cone
of Experience, which activity is
farthest from the real thing?
A.
Read
C. View images
B.
Hear
D. Attend exhibit
114. After giving an input on a
good paragraph, Teacher W asks
her students to rate a given
paragraph along the elements of a
good paragraph. The students
task is in level of _________.
A.
application C. evaluation
B.
analysis
D.
synthesis
115. Which is one characteristic of
an effective classroom
management?
A.
It quickly and unobtrusively
redirects misbehavior once it
occurs.

B.
It teaches dependence on
other for self-control.
C.
It respects cultural norms of
a limited group students.
D.
Strategies are simple enough
to be used consistently
116. Read the following then
answer the questions:
TEACHER: IN WHAT WAYS OTHER
THAN THE PERIODIC TABLE MIGHT
WE PREDICT THE UNDISCOVERED
ELEMENTS?
BOBBY:
WE COULD GO TO THE
MOON AND SEE IF THERE ARE
SOME ELEMENTS THERE WE DONT
HAVE.
BETTY:
WE COULD DIG DOWN
TO THE CENTER OF THE EARTH
AND SEE IF WE FIND ANY OF THE
MISSING ELEMENTS.
RICKY:
WE COULD STUDY
DEBRIS FROM THE METEORITES- IF
WE CAN FIND ANY.
TEACHER: THOSE ARE ALL GOOD
ANSWERS BUT WHAT IF THOSE
EXCURSIONS TO THE MOON, TO
THE CENTER OF THE EARTH OR TO
FIND METEORITES WERE TOO
COSTLY AND TIME CONSUMING?
HOW MIGHT WE USE THE
ELEMENTS WE ALREADY HAVE
HERE ON EARTH TO FIND SOME
NEW ONES?

Question: which questioning


strategy/ies does/do the exchange
of thoughts above illustrate?
A.
Funneling
B.
Sowing and reaping
C.
Nose-dive
D.
Extending and lifting
117. How can you exhibit referent
power on the first day of school?
A.
By making them feel you
know what you are talking about.
B.
By telling them the
importance of good grades.
C.
By reminding your students
your authority over them again and
again.
D.
By giving your students a
sense of belonging and
acceptance.
118. A sixth grade twelve-year old
boy comes from a dysfunctional
family and has been abused and
neglected. He has been to two
orphanages and three different
elementary schools. The student
can decode on the second grade
level, but he can comprehend
orally material at the fourth or fifth
grade level. The most probable
cause/s of this students reading
problem is/are ______.
A.
emotional factors
B.
poor teaching
C.
neurological factors

D.
immaturity
119. Which questioning practice
promotes more class interaction?
A.
Asking the question before
calling a student
B.
Focusing on divergent
questions.
C.
Focusing on convergent
questions.
D.
Asking rhetorical questions
120. The following are sound
specific purpose of questions
EXCEPTS
A.
To call the attention of an
inattentive student
B.
To teach via student answers
C.
To stimulate learners to ask
questions
D.
To arouse interest and
curiosity
121. Which technique should a
teacher use to encourage response
if his students do not respond to his
question?
A.
Ask a specific student to
respond, state the question, and
wait a response.
B.
Tell the class that it will have
detention unless answers are
forthcoming.
C.
Ask another question, an
easier one.
D.
Wait for a response.

122. Which types of play is most


characteristic of a four to six-year
old child?
A.
Solidarity plays and
onlookers plays
B.
Associative and cooperative
plays
C.
Associative and onlookers
plays
D.
Cooperative and solidarity
plays
123. The principle of the individual
difference requires teachers to
___________.
A.
give greater attention to
gifted learners
B.
provide for a variety of
learning activities
C.
treat all learners alike while
in the classroom
D.
prepare modules for slow
learners in class
124. Referring to Teacher S, Nicolle
describes her teacher as fair,
caring and someone you can talk
to. Which power or leadership
does Teacher S have?
A. Referent power
C. Reward
power
B. Legitimate power D. Expert
power
125. During the Spanish period,
what was/were the medium/media
of instruction in schools?
A.The Vernacular
C. Spanish

B.English
D. Spanish and
the Vernacular
126. Rodel is very aloof and cold in
his relationship with classmates.
Which basic goals must have not
been attained by Rodel during his
developmental years, according to
Ericksons theory of psychological
development?
A.
Autonomy C. Initiative
B.
Trust
D. Generativity
127. How can you exhibit expert
power on the first day of school?
A.
By making them feel you
know what you are talking about.
B.
By telling them the
importance of good grades.
C.
By reminding then your
students your authority over them
again and again.
D.
By giving your students a
sense of belonging and
acceptance.
128. Teacher P wants to develop
the skill of synthesizing in her
pupil. Which one will she do?
A.
Ask her students to formulate
a generalization from the data
shown in graphs.
B.
Ask her students to answer
questions beginning with What if

C.
Tell her pupils to state data
presented in graphs.

D.
Directs her students to ask
questions on the parts of the lesson
not understood.
129. John Watson said Men are
built not born. What does this
statement point to?
A.
The ineffectiveness of
training on a persons
development.
B.
The effect of environmental
stimulation on a persons
development.
C.
The absence of genetic
influence on a persons
development.
D.
The effect of heredity.
130. Teacher H strives to draw
participation of every student into
classroom discussion. Which
students need is she trying to
address? The need
A.
to show their oral abilities to
the rest of the class
B.
to be creative
C.
to feel significant and be part
of a group
D.
to get everything out in the
open
131. Teacher Gs lessons objective
has something to do with the skill
of synthesizing? Which behavioral
term is most appropriate?
A.
Test
C. Appraisal

B.
Assess
D.
Theorize
132. Which is a sound classroom
management practice?
A.
Avoid establishing routines;
routines make your student robots.
B.
Establish routines for all daily
needs and tasks.
C.
Apply rules and policies on a
case to case basis.
D.
Apply reactive approach to
discipline.
133. A child who gets punished for
stealing candy may not steal again
immediately. But this does not
mean that the child may not steal
again. Based on Thorndikes
theory on punishment and
learning, this shows that
______________.
A.
punishment strengthen e a
response
B.
punishment remove a
response
C.
punishment does not remove
a response
D.
punishment weakens a
response
134. Which assumption underlines
the teachers use of performance
objectives?
A.
Not every form of learning id
observable.

B.
Performance objectives
assure the learner of learning.
C.
Learning is defined as a
change in the learners observable
performance.
D.
The success of the learner is
based on teachers performance.
135. As a teacher, what do you do
when you engage yourself in major
task analysis?
A.
Test if learning reached
higher level thinking skills
B.
Breakdown a complex task
into sub-skills
C.
Determine the level of
thinking involved
D.
Revise lesson objectives
136. The following are used in
writing performance objectives
EXCEPT
A. delineate
C. integrate
B.
diagram
D. comprehend
137. Teacher B clears his throat to
communicate disapproval of a
students behavior. Which specific
influence technique is this?
A.
Signal interference
C.
Interest boosting
B.
Direct appeal
D.
Proximity control
138. An effective classroom
manager uses low-profile
classroom control. What is a lowprofile classroom technique?

A.
B.
C.

Note to parents
After-school detention
Withdrawal of privileges

D.
Raising the pitch of the voice
139. The primary objective of my
lesson is: To add similar fractions
correctly. Before I can do this I
must first aim at this specific
objective: to distinguish a
numerator from a nominator.
What kind of objective is the
latter?
A.
Major
C. Enabling
B.
Terminal
D. Primary
140. With which goals of
educational institutions as provided
for by the Constitution is the
development of work skills aligned?
A.
To develop moral character
B.
To teach the duties of
citizenship
C.
To inculcate love of country
D.
To develop vocational
efficiency
141. In instructional planning it is
necessary that parts of the plan
from the first to the last have
___________.
A.
clarity
C.
coherence
B.
symmetry D. conciseness
142. All of the following describe
the development of children aged
eleven to thirteen EXCEPT

A.
they shift from impulsivity to
adaptive ability
B.
sex differences in IQ
becomes more evident
C.
they exhibit increased
objectivity in thinking
D.
they show abstract thinking
and judgement
143. If a teacher plans a
constructivist lesson, what will he
most likely do? Plan how he can
_______.
A.
do evaluate his students
work
B.
do reciprocal teaching
C.
do lecture to his students
D.
engage his students in
convergent thinking
144. In mastery learning, the
definition of an acceptable
standard of performance is called a
A. SMART
C. behavior
B. criterion measure
D.
condition
145. A stitch on time saves nine,
so goes the adage. Applied to
classroom management, this
means that we _______.
A.
may not occupy ourselves
with disruptions which are worth
ignoring because they are minor
B.
must be reactive in our
approach to discipline

C.
have to resolve minor
disruptions before they are out of
control
D.
may apply 9 rules out of 10
consistently
146. Ruben is very attached to his
mother and Ruth to her father. In
what developmental stage are they
according to Freudian psychological
theory?
A.
Oedipal stage
C. Anal
Stage
B.
Latent stage
D.
Pre-genital stage
147. What was the prominent
educational issues of the mid
1980s?
A. Bilingual Education C.
Accountability
B. Value Education
D.
Mainstreaming
148. Which behavior is exhibited
by a student who is strong in
interpersonal intelligence?
A.
Works on his/her own.
B.
Keeps interest to
himself/herself
C.
Seeks out a classmate for
help when problem occurs.
D.
Spends time meditating.
149. Which is behavioral term
describes a lesson outcome in the
highest level of Blooms cognitive
domain?

A. Create
C.
Analyze
B. Evaluate
D. Design
150. The main purpose of
compulsory study of the
Constitution is to ___.
A.
develop students into
responsible, thinking citizens
B.
acquaint student with the
historical development of the Phil
Constitution
C.
make constitutional experts
of the students
D.
prepare students for lawmaking
151. A goal-oriented instruction
culminates in _______.
A.
planning activities
B.
evaluation
C.
identification of topics
D.
formulation of objectives
152. Based on Freuds
psychoanalytic theory which
component (s) of personality is
(are) concerned with a sense of
right and wrong?
A.
Super ego
C. Id
B.
Super-ego and Ego
D.
Ego
153. Who among the following puts
more emphasis on core
requirements, longer school day,
longer academic year and more
challenging textbooks?
A.
Perennialist C. Progressivist

B.
Essentialist D. Existentialist
154. A student passes a research
report poorly written but ornately
presented in a folder to make up
for the poor quality of the book
report content. Which Filipino trait
does this practice prove?
A.
art of academics
B.
substance over porma
C.
art over science
D.
porma over substance
155. Which one may support
equitable access but may sacrifice
quality?
A.Open admission
C. Deregulated tuition fee hike
B. School accreditation
D. Selective retention
156. Based on Piagets theory,
what should a teacher provide for
children in the sensimotor stage?
A.
Games and other physical
activities to develop motor skill.
B.
Learning activities that
involve problems of classification
and ordering.
C.
Activities for hypothesis
formulation.
D.
Stimulating environment with
ample objects to play with.
157. A teachers summary of a
lesson serves the following
functions, EXCEPT
A.
it links the parts of the
lesson.

B.
it brings together the
information that has been
discussed.
C.
it makes provisions for full
participation of students.
D.
it clinches the basic ideas or
concepts of the lesson.
158. As a teacher, you are
rationalist, Which among these will
be your guiding principle?
A.
I must teach the child that
we can never have real knowledge
of anything.
B.
I must teach the child to
develop his mental powers to the
full.
C.
I must teach the child so he
is assured of heaven
D.
I must teach the child every
knowledge, skills and value that he
needs for a better future.
159. Bruners theory on intellectual
development moves from enactive
to iconic and symbolic stages. In
which stage(s) are diagrams helpful
to accompany verbal information?
A.
Enactive and iconic
C.
Symbolic and enactive
B.
Symbolic
D. Iconic
160. Students scores on a test
were: 72,72,73,74,76,78,81,83,85.
The score 76 is the ___.
A.
mode
C. average
B.
mean
D. median

161. Standard deviation is to


variability as mode to _____.
A.
level of difficulty
C.
correction
B.
discrimination
D.
central tendency
162. A teacher who equates
authority with power does NOT
usually __________.
A.Shame
C. develop selfrespect in every pupil
B.retaliate
D. intimidate
163. Which is a true foundation of
the social order?
A. Obedient citizenry
B. The reciprocation of rights
and duties
C. Strong political leadership
D. Equitable distribution of
wealth
164. Standard deviation is to
variability as mean is to _______.
A.
coefficient of correlation C.
discrimination index
B.
central tendency
D.
level of difficulty
165. Teacher Q does not want
Teacher B to be promoted and so
writes an anonymous letter against
Teacher B accusing her of
fabricated lies. Teacher Q mails this
anonymous letter to School
Division Superintendent. What

should Teacher Q do if she has to


act professionally?
A.
Submit a signed justifiable
criticism against Teacher B, if there
is any.
B.
Go straight to the School
Division Superintendent and gives
criticism verbally.
C.
Hire a group to distribute
poison letters against Teacher B for
the information dissemination.
D.
Instigate student activists to
read poison letter over the
microphone.
166. Each teacher is said to be a
trustee of the cultural and
educational heritage of the nation
and is under obligation to transmit
to learners such heritage. Which
practice makes him fulfill such
obligation?
A.
Use the latest instruction
technology
B.
Observing continuing
professional education.
C.
Use interactive teaching
strategies
D.
Study the life of Filipino
heroes
167. Which type of reports to onthe spot description of some
incident, episode or occurrence
that is being observed and
recorded as being of possible
significance?

A.
B.

Autobiographical report
Value and interest report

C.
Biographical report
D.
Anecdotal report
168. Teacher A is directed to pass
an undeserving student with a
death threat. Which advice will a
hedonist give?
A.
Pass the student. Why suffer
the threat?
B.
Dont pass him. You surely
will not like someone to give you a
death threat in order to pass.
C.
Pass the student. That will be
use to the student, his parents and
you.
D.
D. Dont pass him. Live by
principle of justice. You will get
reward, if not in this life,
in the
next.
169. If you agree with Rizal on how
you can contribute to our nations
redemption, which should you work
for?
A.
Opening our doors to foreign
influence
B.
Upgrading the quality of the
Filipino through education
C.
Stabilizing the political
situation
D.
Gaining economic recovery
170. All men are pretty much
alike. It is only by custom that they
are set apart, said one Oriental

philosopher. Where can this


thought be most inspiring?
A.
In a multi-cultural group of
learners.
B.
In multi-cultural and
heterogeneous groups of learners
and indigenous peoples group.
C.
In a class composed of
indigenous people.
D.
In heterogeneous class of
learners.
171. In what way can teachers
uphold the highest possible
standard of quality education?
A.
By continually improving
themselves personally and
professionally
B.
By wearing expensive clothes
to change peoples poor perception
of teachers
C.
By working out undeserved
promotions
D.
By putting down other
professions to lift the status of
teaching
172. How would you select the
most fit in government position?
Applying Confucius teaching, which
would be the answer?
A.
By course accreditation of an
accrediting body
B.
By merit system and course
accreditation
C.
By merit system
D.
By government examinations

173. The attention to the


development of a deep respect and
affection for our rich cultural past is
an influence of ______
A. Confucius
C. Teilhard de
Chardin
B. Heqel
D. Dewey
174. A teacher / student is held
responsible for his actions because
s/he _______.
A.
has instincts
C. has
mature
B.
has a choice
D. is reason
175. The typical autocratic teacher
consistently does the following
EXCEPT
A.
intimidating students
C. shaming students
B.
ridiculing students
D. encouraging students
176. Teacher H and teacher I are
rivals for promotion. To again the
favor of the promotion staff,
teacher I offers her beach resort for
free for members of he promotional
staff before ranking. As one of the
contenders for promotions, is this
becoming of her to do
A.
Yes. This will be professional
growth for the promotional staff.
B.
No. This may exert undue
influence on the members of the
promotional staff and so may fail to
promote on the basis of merit.

C.
Yes. The rare invitation will
certainly be welcomed by an
overworked promotional staff.
D.
Yes. Theres nothing wrong
with sharing ones blessings.
177. Rights and duties are
correlative. This means that.
A.
rights and duties regulate the
relationship of men in society.
B.
rights and duties arise from
natural law.
C.
each right carries with it one
or several corresponding duties.
D.
rights and duties ultimately
come from God.
178. In the Preamble of Code of
Ethics of Professional Teachers,
which is NOT said of teachers?
A.
LET passer
B.
Duly licensed professionals
C.
Possess dignity and
reputation
D.
With high moral values as
well as technical and professional
competence
179. What should you do if a parent
who is concerned about a grade his
child received compared to another
students grade, demands to see
both students grades?
A.
Refuse to show either record.
B.
Show only his childs record.
C.
Refuse to show any record
without expressing permission from
principal.

D.
Show both records to him.
180. Teacher often complain of
numerous non-teaching
assignments that adversely affect
their teaching. Does this mean
that teachers must be preoccupied
only with teaching?
A.
Yes, if they are given other
assignments, justice demands that
they be properly compensated.
B.
Yes, because other
community leaders, not leaders,
not teachers, are asked to lead in
community activities.
C.
Yes, because every teacher is
expected to provide leadership and
initiative in activities for
betterment of communities.
D.
No, because teaching is
enough full time job.
181. Which illustrates a
development approach in guidance
and counseling?
A.
Spotting on students in need
of guidance
B.
Teaching students how to
interact in a positive manner
C.
Acting as a mentor
D.
Making the decision for the
confused student
182. Whose influence is the
education program that puts
emphasis on self-development

through the classics, music, and


ritual?
A.
Buddha
C.
Confucius
B.
MohammedD. Lao tsu
183. Helping in the development of
graduates who are maka-Diyos is
an influence of ___.
A.
naturalistic morality
B.
classical Christian morality
C.
situational morality
D.
dialectical morality
184. What is the mean of this score
distribution 4,5,6,7,8,9,10?
A.
7
C. 8.5
B.
B. 6
D. 7.5
185. Are percentile ranks the same
as percentage correct?
A.
It cannot be determined
unless scores are given.
B.
It cannot be determined
unless the number of examinees is
given
C.
No
D.
Yes
186. Teacher F is a newly converted
to a religion. Deeply convinced of
his new found religion, he starts
Monday classes by attacking one
religion and convinces his pupil to
attend their religion services on
Sundays. Is this in accordance with
the Code of Ethics of Professional
Teachers?

A.
Yes. What he does
strengthens value education.
B.
No. A teacher should not use
his position to proselyte others.
C.
Yes. In the name of academic
freedom, a teacher can decide
what to teach.
D.
Yes. What he does is a value
education.
187. In a study conducted, the
pupils were asked which nationality
they preferred. If given a choice.
Majority of the pupils wanted to
Americans. In this case, in which
obligation, relative to the state, do
schools seem to be failing? In their
obligation to
A.
respect for all duly
constituted authorities
B.
promote national pride
C.
promote obedience to the
laws of the state
D.
install allegiance to the
Constitution
188. A guest in one graduation
rites told his audience: Reminder,
you are what you choose to be.
The guest speaker is more of a/an
_____.
A.
realistic
C. pragmatist
B.
Idealistic
D. existentialist
189. From whom do we owe the
theory of deductive interference as
illustrated in syllogism?
A.
Plato
C. Socrates

B.
Aristotle
D.
Pythagoras
190. In what way can teachers
uphold the highest possible
standard of quality education?
A.
By continually improving
themselves personally and
professionally
B.
By putting down other
professions to lift the status of
teaching
C.
By wearing expensive clothes
to change peoples poor perception
of teachers
D.
By working out undeserved
promotions
191. Two students are given the
WISC III. One has a full scale IQ of
91, while the other has an IQ of
109. Which conclusion can be
drawn?
A.
Both students are functioning
in the average range of intellectual
ability
B.
Another IQ test should be
given to truly assess their
intellectual potential.
C.
The first student is probably
below average, while the second
has above average potential.
D.
The second student has
significantly higher intellectual
ability.
192. Which describes normreferenced grading?

A.
What constitutes a perfect
score
B.
The students past
performance
C.
An absolute standard
D.
The performance of the
group
193. Teacher A knows of the illegal
activities of a neighbor but keeps
quiet in order not to be involved in
any investigation. Which
foundational principle of morality
does Teacher A fail to apply?
A.
Always do what is right
B.
The principle of double effect
C.
The end does not justify the
means
D.
Between two evils, do the
lesser evil
194. Teacher Q does not want
Teacher B to be promoted and so
writes an anonymous letter against
Teacher B accusing her of
fabricated lies. Teacher Q mails this
anonymous letter to School
Division Superintendent. What
should Teacher Q do if she has to
act professionally?
A.
Hire a group to distribute
poison letters against Teacher B for
the information dissemination.
B.
Submit a signed justifiable
criticism against Teacher B, if there
is any.

C.
Go straight to the School
Division Superintendent and gives
criticism verbally.
D.
Instigate student activists to
read poison letter over the
microphone
195. A teachers summary of a
lesson serves the following
functions, EXCEPT
A.
it makes provisions for full
participation of students.
B.
it brings together the
information that has been
discussed.
C.
it links the parts of the
lesson.
D.
it. clinches the basic ideas or
concepts of the lesson
196. Soc exhibits fear response to
freely roaming dogs but does not
show fear when a dog is on a leash
or confined to a pen. Which
conditioning process is illustrated?
A. Extinction
C.
Generation
B. Discrimination D. Acquisitio
197. Ruben is very attached to his
mother and Ruth to her father. In
what developmental stage are they
according to Freudian psychological
theory?
A. Latent stage
C. Anal
Stage

B. Pre-genital stage
D. Oedipal stage
198. The concepts of Trust vs.
mistrust, autonomy vs. shame &
self-doubt, and initiative vs. guilt
are most closely related with works
of _____.
A.
Jung
C. Erickson
B.
Freud
D. Piaget
199. A goal-oriented instruction
culminates in _______.
A.
evaluation
B.
formulation of objectives
C.
identification of topics
D.
planning activities
200. Which of the following is
considered a peripheral device?
A.
Keyboard C. Monitor
B.
CPU
D. Printer

16.

65.

115.

165.

17.

66.

116.

166.

18.

67.

117.

167.

19.

68.

118.

168.

20.

69.

119.

169.

21.

70.

120.

170

ANSWER KEY

22.

71.

121.

171.

PROFESSIONAL EDUCATION

23.

72.

122.

172.

(Secondary)

24.

73.

123.

173.

25. no answer

74.

124.

174.

26.

75.

125.

175.

27.

76.

126.

176.

28.

77.

127.

177.

29.

78.

128.

178.

1.

50.

100.

2.

B
198.
102.

51.
C
B

101. no answer 151.


3.
D
52.
A
199. A

A
200.

53.
D

5.

54.

104.

154.

30.

79.

129.

179.

6.

55.

105.

155.

31.

80.

130.

180.

7.

56.

106.

156.

32.

81.

131.

181.

8.

57.

107.

157

33.

82.

132.

182.

9.

58.

108.

158.

34.

83.

133.

183.

10.

59.

109.

159. no answer

35.

84.

134.

184.

11.

60.

110.

160.

36.

85.

135.

185.

12.

61.

111.

161.

37.

86.

136.

186.

13.

62.

112.

162.

38.

87.

137.

187.

14.

63.

113.

163.

39.

88.

138.

188.

15.

64.

114.

164.

40

89.

139.

189.

4.
answer

152.

no answer

150.

103.

197.

D
D
153. no

41.

90.

140.

190.

42.

91.

141.

191.

43.

92.

142.

192.

44.

93.

143.

193.

45

94.

144.

194.

46.

95.

145.

195.

47.

96.

146.

196.

48.

97.

147.

49.

98.

148.

99.

149.

General Education (secondary)


MULTIPLE CHOICE
1.
Which refers to FUNNEL
EFFECT?
A. The belief that every
criminal gets caught and is
punished.
B. The belief that crime is
under control in the United States.

C. The idea that only a very few


suspects arrested for committing a
crime are actually punished.
D. The idea that all crimes put into
the same criminal justice system.
2.
One of the most outstanding
accomplishments of the
cooperative movement is the
encouragement of thrift. Which
maximum of God puts this into
practice?

A. God Helps those who help


themselves.
B. Look at the birds: they do
not plant seeds, gather a harvest
and put it in barns; yet your Father
in heaven takes care of them!
C. He is near to those who call
to Him, who call to Him with
sincerity.

D. Happy are those who are


merciful to others; God will be
merciful to them!
3.
Which is NOT personal
integrity?
A. Time
C. Order
B. Place
D. Harmony
4.
Carters part in relinquishing
U.S. control of the Central Zone to
Panama is described as a victory
for ________.
A. conservatism
C. isolationism
B. anti-imperialism
D. imperialism
5.
Of the following changes in
the socio-economic, political
cultural and physical that have
occurred in the Filipino family,
which one remains to be TRUE?
A.
The loss of the traditional
evening prayer and ritual of
blessing (mano)
B.
The unity of the family
despite competing demands
C.
The continued parental
influence over childrens language
dress and other behavior
D.
The continued support for
parents and siblings
6.
Mang Tacio has been
unemployed for quite sometime
due to his negative attitude toward
work. Which program of the

Department of Social Welfare and


Services will help him?
A. Income in Kind Program
C. HRD Program
B. Anti-Medicancy Program
D. Social Insurance Program
7.
The main message of the
Moral Recovery Program launched
by Leticia Ramos Shahani starts
with the ________.
A. world
C. self
B. family
D. nation
8.
Which are limited only to the
sale of real property and stock
transaction?
A. Business incomes
C. Employment incomes
B. Capital gains
D. Passive incomes
9.
If a farmer would want
assistance like pricing, guarantee
for all agricultural produce or
cooperative management training,
where would he go?
A.
Support services of the
Department of Agrarian Reform
B.
Special Agrarian Court under
the Regional Trial Court
C.
DAR adjudication board
D.
Land Bank
10. The Soviet Unions attempt
to establish a missile base in Cuba

is interpreted as a direct violation


of
A. The Truman Doctrine
B. The Monroe Doctrine
C. The Strategic Arms
Limitation Talks (SALT)
D. The Declaration of
Independence
11. As a representative of the
Urban Poor Commission of the
Association of Religious Superiors
(ARS), which action will you most
likely take to resolve the long-term
roots of structural inequalitiesproliferation of child labor and child
prostitution?
A.
Raise views of human rights
abuse.
B.
Organize regular programs
for information and discussion
of human rights
C.
Conduct skills training
D.
Raise questions over the
governments commitment to
rebuild human rights
12. What values are being given
priority by juries in criminal cases?
A. The rights of the criminal
over the strict interpretation of the
law
B. The safety of the community
over the sympathy for the criminal
C. The needs of the criminals
over the advice of the judge

D. The punishment of the


criminal over the safety of the
community
13. Lucys husband has been a
drug dependent. She wanted him
rehabilitated to be economically
productive. Where will she commit
her husband?
A. DARN
C. DARE
B. Bukang Liwayway Ceter
D. NFPI
14. Which will solve poverty
caused by capitalism?
A. Fascism
C. Empirism
B. Communism
D. Socialism
15.
Which family obligation is
especially valued?
A. Supplying groceries to
relatives in remote barrios
B. Providing health assistance
to relatives living in the same
locale
C. Sending to college
relatives in remote barrios
D. Keeping immediate family
members out of trouble

16. Which theory was asserted


by the Pan-German belief in the
superiority of the Aryan race and
that the strength of the German
culture came from a strong,
healthy and rustic lineage?
A. Dependency Theory
B. Culture of Poverty Theory
C. Social Darwinist Theory
D. Theory of Capitalism
17. Why was San Andres
Cooperative Association of Paco,
not exempted
from taxation?
A. I accumulates reserves and
undivided net savings of
P8,000,000.00
B. I accumulates reserves and
undivided net savings of
P10,000.00
C. I accumulates reserves and
undivided net savings of
P9,000,000.00
D. I accumulates reserves and
undivided net savings of
P11,000.00
18. Juliet Villaruel was a
landowner from Cabio, Nueva Ecija.
Under the CARL, she was claiming
8 hectares, 5 hectares of which
represented the retention limit and
the 3 hectares for her only child.
Why was her child denied 3
hectares?

A. Her son was 15 years old


who was actually tilting the farm
B. Her son was 17 years old who
was managing the farm
C. Her son was 13 years old
who has been helping till the farm
D. Her son was 19 years old
who was actually tilting or
managing the farm
19. Which part of the Allied
action has been detailed in this
passage?
After Saddam Hussein violated
international arguments by sending
Iraq troops to Kuwait and missiles
into their neighboring countries,
the Allies responded with military
action.
A. The Allied bases in Saudi Arabia
B. The ground was in the desert
C. The movement of Allied troops in
Iraq
D. The campaign in the Baghdad
area
20. Which one is the human right
to life?
A. Peace
C. Own Property
B. Live in national and international
order
D. Fair trial
21. Two days after Japan
attacked Pearl Harbor, Roosevelt
made the following statements:

In the past few years and most


violently in the past few days, we
learned a terrible lesson. We must
begin the great task that is before
us by abandoning once and for all
the illusion that we can ever again
isolate ourselves from the rest of
the humanity.
In the statement, Roosevelt is
expressing the ideas of ________.
A. an internationalist
C. an imperialist
B. an anti-imperialist
D. an isolationist
22. The following are defects
present at the time of marriage
which is voidable and annullable
EXCEPT
A. impotence
C. fraud
B. deceit
D. threat
23. Why does a pendulum in a
grandfather clock once set in
motion continue to swing, thereby
regulating the clocks movement?
This is due to the Law of
A. Universal Gravitation
C. Applied Force
B. Action and Reaction
D. Inertia
24. New ponies, perennial plants
that produce shows flowers can be
propagated from the parent plant

by dividing corns that grow


underground. The reproductive
form resembles a _____________.
A. bulb
C. seed
B. runner
D. bud
25. The San Pascual Credit
Cooperative of Quezon City wishes
to apply for a loan of five pesos
from one of the financial institution,
EXCEPT?
A. Development Bank of the
Philippines
B. Central Bank of the Philippines
C. Philippine National Bank
D. Land Bank of the Philippines
26. The following are legitimate
children EXCEPT?
A.
those born by artificial
insemination.
B.
those legitimate.
C.
those born during a valid
marriage of parents.
D.
those born out a valid
marriage of parents.
27. Of the following, which is
imposed a final tax of ten percent
(10%)
A. PSCO and lotto winnings.
B. Books literary works and
musical compositions
C. Currency banks deposit
D. Royalties
28. Which of the following foreign
policy actions today is a direct

result of early American imperialist


policy?
A.
Station of American troops in
West Germany
B.
Americans military support
of Israel in the Middle East
C.
Americans patrolling of
waters of the Libyan coast
D.
Americans establishment of
military bases I nth Philippines
29. Which area of the brain
controls feelings on the side of a
persons face?
A. The left occipital lobe
C. The right parietal lobe
B. The left parietal lobe
D. The right occipital lobe
30. Why does a bullet when
discharged into the air eventually
fall to the ground? This is due to
the Law of ___________.
A. Universal Gravitation
C. Inertia
B. Applied Force
D. Action and Reaction
31. The principle under which
thermostat operates is the same
when?
A.
a gas expands to fill the
container in which it is held.
B.
a pendulum swings when it is
set into motion.
C.
a chemical reaction occurs
when two substances combine.

D.
the level of mercury rises or
falls in a glass tube.
32. Which explains the reason
why there are continuous and
increasing human rights violations?
A.
The United Nations General
Assembly approved only
resolutions on human rights and
the basic freedoms which are not
binding
B.
The solutions used are
ineffective.
C.
The United Nations as an
international body is rather slow in
the exercise of its powers
D.
The United Nations uses as a
single solution on all forms of
human rights violations.
33. In an experiment, a vacuum
is created when air is removed
from a tube. A coin and bits of
confetti are released in the vacuum
at the same time. They fall at the
same rate and reach the bottom at
the same time. The experiment
proves that
I.
In a vacuum, the rate of
accelerator is the same for all
objects regardless of weight.
II.
Outside a vacuum air
resistance is what makes different
objects fall at different rates.
III.
Gravity has no effect at all on
objects that fall in a vacuum.

A. I and II
C. I, II and III
B. I and III
D. II and III
34. Thousands of street children
in large Brazilian cities were
murdered by parliamentary death
squads which includes police
officers. What could be the reason
why these operations were not
suppressed by the government?
A.
Totalitarian governments do
not give protection
B.
The business people even
funded these operations to clean
up their streets and neighborhoods.
C.
The Universal Declaration of
Human Rights was only lip service
D.
To how they treat their
people was nobody elses business.
35. Which of the following
procedures used by a farmer is
NOT related directly to preventing
erosion?
A. Contour plowing around a
hill
B. Planting more seeds than
are necessary to yield a bountiful
crop.
C. Planting grass in gullies to
act as a filter
D.
Planting crops in
alternate rows (Strip farming)
36. You buy a new refrigerator for
P12,800.00 and make a down

payment of P2,500.00. If you


finance the remainder at 8%
annually for three years, how much
will you actually pay for the
refrigerator?
A. P12,190.00
D. P12,772.00
B. P10,309.00
E. P15,272.00
C. none of these
37.
4 1/5
or
4 1/5 + 3
2/7 = __________.
+ 3 2/7
________
A. 7 3/12
C. 7 17/35
B. 7 3/35
D. 7 1/35
E. none of these
38. In which kingdom should
MOLD be classified?
A. Protista
C. Fungi
B. Plantae
D. Animalia
39. Which of the following BEST
demonstrates the greenhouse
principle?
A. A heated aquarium
B. A car with rolled-up windows
C. A microwave oven
D. A solar battery-powered
calculator
40. Which of the following should
you expect to be true about the
rate of cellular respiration for a

group of students who are the


same age, height, and weight?
A.
Athletes would tend to have
higher rates of cellular respiration
than nonathletes.
B.
Africans would have higher
rates of cellular respiration than
Asians.
C.
Boys would have a higher
rates of cellular respiration than
girls.
D.
Nonathletes would have
higher rates of cellular respiration
than Athletes.
41. Which kingdom should
STREPTOCOCCUS be classified?
A. Protista
C. Fungi
B. Plantae
D. Monera
42. What is the function of
DIFFUSION in the human body?
A.
Regulates blood flow
B.
Plays an insignificant role in
the bodys functioning
C.
Allows an even distribution of
substances throughout all cells of
the body
D.
Comes into play in times of
extreme illness
43. In an experiment, a drop of
blue ink is placed on the surface of
a glass of water. In a few minutes,
the drop of ink is dispersed
throughout the water, turning it
light blue. The result of the
experiment proves that

A.
molecules of ink and
molecules of water are in constant
motion
B.
heat causes the ink to
disperse
C.
a new compound is formed
by the combination of ink and
water
D.
ink molecules have less
density than water molecules
44. Which Law of force and
motion explains this occurrence,
when a rocket is propelled upward
by the powerful downward
discharged of exhaust gases?
A. Universal Gravitation
C. Applied Force
B. Action and Reaction
D. Inertia
45. An elderly woman suffered a
stroke-a restriction of blood flow to
the brain. if the stroke caused to
the right side of her body to
become temporarily paralyzed, she
most likely experienced a
decreased blood flow to
A. the left side of her body
C. the left side of her brain
B. the front of her brain
D. the right side of her brain
46.Which of the following methods
can all diabetics control their
condition and avoid heart disease
and blindness?

I.
Regulates their intake of
glucose
II.
Increase the levels of insulin
in the body by taking insulin
injections
III.
Maintaining a reasonable
exercise regimen to keep weight
down
A. I
C. I and II
B. II
D. I and III
47. Scientist also find that other
stalky vegetables such as carrots
also help lower pressure. This
statement is BEST classified as
A. experiment
C. finding
B. nonessential fact
D.
prediction
48. Each of the following objects
is designed to employ the
buoyancy principle EXCEPT a
A. life preserver
C.
submarine
B. kite
D. canoe
49. What is the difference
between the largest 4-digit number
and the smallest 4-digit numbers?
A. 8999
D. 8888
B. 8000
E. none of these
C. 9998
50. Four mangoes cost P29.00 at
that price what will 2 dozen
mangoes cost?
A. P217.50
C. P348.50
B. P188.50
D. P870.00
E. none of these

51. Which of the following


internal forces interrupt the
external forces erosion?
I.
Forces that cause volcanoes
II.
Forces that cause ocean
trenchers
III.
Forces that cause create
mountains
A. I, II and III
C. II and III
B. I and III
D. I and II
52. The How many gallons of
water will fill a fish tank that is 18
inches by 12 inches by 48 inches
(There are 231 cubic inches per
gallon) Round your answer to the
nearest gallon.
A. 45 gallons
D. 47
gallons
B. 40 gallons
E. 37
gallons
C. none of these
53. What is the sum of all the
two digit numbers which are
divisible by 5?
A. 945
D. 1050
B. 950
E. none of these
C. 960
54. How many whole numbers
can divide 30 exactly?
A. Eight
C. Five
B. Six
D. Four

E. none of these
55. Which one explains why
oxygen, a gas is the largest
component of the Earths crust?
A.
Oxygen gives Earths crust its
lightness
B.
Oxygen is the most abundant
element in the world
C.
Oxygen is capable of
combining with most of the
elements in the Earths crust
D.
Oxygen is needed to sustain
all life on Earth
56. To pass the English Test,
Lucille must get 75% of the items
correct. Out of 80 questions, how
many must she correctly answer?
A. 55
D. 65
B. 60
E. 70
C. none of these
57. Employees at Shairaa
Musicmart get a 20% discount on
all purchases. If Teresa buys three
tapes at P47.49 each. How much
will she have to pay after her
employee discount?
A.
P16.98
D.
P17.98
B.
P19.98
E.
none of these
C.
P18.98
58.
One package is 100 pounds,
and the other is 150 pounds. The

weight of the second package is


how many times of the first?
A. 1 times heavier
D. 10 pounds heavier
B. as heavy
E. 20 pounds heavier
C. none of these
59. A carpenter wanted three
piece of wood each 1 5/8 feet long.
If he planned to cut them from a 6foot piece of wood, how much of
the piece would be left?
A. 4 3/8 ft
C. 4 7/8 ft
B. 1 1/8 ft
D. 3 ft
E. none of these
60. How much larger is the
supplement of a 57 degree angle
than the complement of a 75
degree angle?
A. 108 degrees
C. none of
these
B. 18 degrees
D. 123 degrees
c. 105 degrees
61. If a baseball player hits 10
home runs in the first 45 games, at
the same rate how many home
runs can he expect to hit during
162-games season?
A. 38
C. 36
B. 42
D. 40
E. none of these

62. Which are the next three


terms in the progression 1/125,
-1/25, 1/57 terms?
A. 2,6,-26
C. 4,8,-28
B. 3,7,-27
D. 1,5,-25
E. none of these
63. How many ways can a
committee of 4 people be selected
from a group of 7 people?
A. 35
D. 210
B. 70
E. none of these
C. 140
64. Which is the length of the
hypotenuse of a right triangle with
legs 5 inches and 12 inches?
A. 17 in.
C. 11 in.
B. 13 in.
D. 20 in.
E. none of these
65. Which of the following is the
BEST example of self-preservation?
A.
A mouse runs when it sees a
cat.
B.
A dog barks when it sees its
owner
C.
A Man decide to quit smoking
D.
A salmon swims back to the
place of its birth to lay eggs.
66. A meter was cut at the 35-cm
mark. What is the ratio of the
smaller piece to the larger piece?
A. 7:13
C. 35:100

B. 65:35 D. 65:100
E. none of these
67. The hypotenuse of a triangle is
25 feet. If one leg is 24 feet, what
is the length of the other leg?
A. 6 ft
D. none of these
B. 5 ft
E. 7ft
C. 20 ft
68. Which is the equivalent
common fraction of the repeating
decimal 3.242424?
A. 107/33
C. 109/33
B. 110/33
D. 108/33
E. none of these
69. Tides, caused by the moons
gravity, create a fractional force
that is gradually slowing down
Earths rotational speed. One
million years from now, scientist
may discover that compared to
today, Earths
A. day is longer
C. day is
shorter
B. year is shorter D. year is longer
70. How much topsoil is needed
to cover a garden 25 feet by 40
feet to a depth of 6 inches?
A. 480 cuft
D. none of these
B. 440 cuft
E. 460 cuft
C. 500 cuft
71. A car dealer is offering a
rebate of P7,500.00 on any new-car
purchase. If the purchase price of
a car is P200,000.00 more than it

was last year. What is the rate of


the discount offered by the rebate?
A. 10%
C. 7.5%
B. not enough information is given
D. 13.3%
E. 14.2%
72. In the progression 18,
-12,8which term is 512/729?
A. the 8th
C. the 9th
B. the 6th
D. the 7th
E. none of these
73. Which of the following facts
support the big bang theorys
explanation of the creation of the
universe?
A.
The universe does not
expand nor contract.
B.
The universe seldom expand.
C.
The universe will have
background radiation.
D.
The universe has no
beginning nor end.
74. Mr. Garcia owns a 10
hectares tract of land. He plans to
subdivide this tract into hectare
lots. He must first set aside 1/6 of
the total land roads. How many
lots will this tract yield?
A. 30
C. 42
B. 35 D. 45
E. none of these
75. Find m in the proportion
m/12=30/24.
A. 30
C. 20
B. 15
D. 25
E. none of these

76. If P75,000 is shared among


three children in the ratio of 3:7;15,
the size of the smaller share is?
A. P9
C. P25
B. P15
D. P35
E. none of these
77. In how many ways can you
arrange three mathematics books
(Algebra, Geometry, Trigonometry)
in order on shell?
A.6
C. 12
B.8
D. 24
E. none of these
78. Which are the next three
terms in the progression 1,4,168
terms?
A.64,256,1024
C.
66,258,1026
B.67,259,1027
D.
65,257,1025
E. none of these
79.
Which one should be TRUE is
Earths rotational axis not tilt?
I.
Days and nights would be the
same length everywhere on Earth.
II.
There would be no hours of
darkness on points along the
equator
III.
Earth would have no seasons.
IV.
Each part of the Earth would
have the same daily temperature
pattern
A.
II and IV
C.
IV and I
B.
III and IV
D.
I and III

80. The carat is a unit of


measure used to weight precious
stones. It equals 3.086 grains.
How many grains does 2.8 carat
diamond weigh?
A. 864.08
C. 8.6408
B. 86.408
D 5640.8
E. none of these
81. Robert Frost wrote the poem
Acquainted with the Night from
which the stanza is taken:
I have been one acquainted with
the night.
I have walked out in rain-back in
rain.
I have outwalked the farthest city
light.
The poet in the stanza talks of
A. isolation and loneliness.
B. happiness in having been
acquainted with the night.
C. joy getting out of the house.
D. youthful delight playing in the
rain.
82. What is meant by AT SIX AND
SEVENS in this sentence?
We moved into the house
last week, but Im afraid everything
is still at six and sevens.
A. The things have not been
shipped
C. In an orderly manner
B. In a state of confusion
D. The boxes are still intact

83. Sa Espiritu ni Bathala ang


nangangalaga ng kanilang
kalusugan ang ipinahihiwatig na
katangian ay _______.
A. malinis
C. maliksi
B. mabisa
D.
makapangyarihan
84. The stanza below is taken from
Barter by Sara Teasdale
Life has loneliness to sell,
Music like a curve of gold,
Scent of pine trees in the rain,
Eyes that love you, arms that hold,
And for your spirits still delight,
Holy thoughts that star the night.
To what does Teasdale
compare music?
A. The scent of pine trees
C. Eyes that love
B. A curve of gold
D. The rain
85. Which word ends with [S]
pronounced [Z]?
A. Maps
C. Laughs
B. Jokes
D. Buys
86. Which of the following lines is a
simile?
A. Holding wonder like a cup
C. Eyes that love you, arms
that hold
B. Life has loneliness to sell
D. Buy it ang never count the
cost

87. Which is the BEST WAY to write


the underlined portion of this
sentence? A person should keep in
mind some basic safety rules when
you are deciding whether or not to
use a fire extinguisher.
A. Rules you decide
C. Rules you are deciding
B. Rules when you decided
D. Rules when deciding
88. What is meant by the
expression TO GET BLOOD OUT OF
A STONE in this sentence?
Geraldo has owned me fifty
thousand pesos for over a year
now. Ive asked him for it on
several occasions, but its like
trying to get blood out of a stone.
A. Something is impossible.
B. Someone refuses to
cooperative.
C. Someone is willing to give
what is asked.
D. Someone wants revenge.
89. What correction should be
made to this sentence? First born
often pattern their behavior after
theyre parents and other adults.
A. Replace their to theyre
C. Replace theyre to their
B. Change pattern to patterned
D. No correction is necessary

90. What is meant by SOFT


OPINION in this sentence?
Rebecca realized that if she stayed
in her present job it would mean
competing with an envious rival.
Leaving the company would
probably be a soft option
A. An action that is difficult to take
C. An action that is easier
B. An action that is not agreeable
D. An action that is weakly funded
91. What correction should be
made to this sentence? Most State
tourism departments and some
travel agencies have bed and
breakfast listings.
A. Insert a comma after agencies
C. Change tourism to Tourism
B. Change have to has
D. Change State to state
92. What is suggested in the
opening line?
June 13, 1986-they came from all
over America- 200,000 heroes
strong, with their families.
A.
The writer holds great
admiration for the veterans
B.
The writer was a veteran of
the war
C.
The writer is opposed to the
Vietnam War
D.
The writer is a flag-waving
patriot
93. To gain the attention of the
audience, the trick is __________.

A. start low, speak hurriedly


C. start high, speak rapidly
B. start high, speak loudly
D. start low, speak slowly
94. What correction should be
made to this sentence? Recently,
educators exammined the
effectiveness of computer
instruction in schools.
A. Replace educators with
educators
B. Change the spelling of
exammined to examined
C. Change schools to Schools
D. Replace computer with
computers
95. Which is BEST WAY to write the
underlined portion of this
sentence?
There is smoke detectors in many
homes to warm residents of a fire,
but fire extinguishers can actually
help people fight fires.
A. Theyre is
C. Their are
B. Their is
D. There are
96. What do the following lines
CONVEY?
Midnight, not a sound from the
pavement.
Has the moon lost her memory?
She is smiling alone.
In the lamp light the withered
leaves
Collect at my feet
And the wind begins to moan.

A. Confusion
C. Loneliness
B. Optimism
D. Eagerness
97. What correction should be
made to this sentence?
Most fire-related deaths result
from households fries, yet many
people do not have fire
extinguishers in their homes.
A. Remove the comma after fires
C. Change have to has
B. Change result to results
D. Replace deaths with deaths
98. Which verb in the sentence is
pronounced with the ending as [d]?
They laughed and joked as they
walked and played.
A. Joked
C. Walked
B. Laughed
D. Played
99. Which of the following words
DOES NOT contains the [voiceless
th]?
A. Mouth C. Teet B.Breath D.
Health
100. Which is the BEST way to
write the underlined portion of this
sentence?
However, their VCR kept them
from missing their favorite prime
time shows.
A. Keepes
C. Had kept
B. Keeps
D. Keeped
101. Which is the BEST way to
write the underlined portion of this
sentence?

Researchers also speculate that


some teachers might have given
boys more computer time because
parents and teachers expected
boys to need computers for future
careers.
A. Will expect
C. Will have
expected
B. Expected
D. Expecting
102. Which word contain the [ae]
sound?
A. Carriage
C. Castle
B. Cabin
D. Can
103. What correction should be
made to this sentence?
One of their theories is that the
first child receives more of the
parents attention than other
children so first-borns tend to be
more intellectual.
A. Change is to are
C. Change parents to parents
B. Insert a comma after children
D. Change theories to theorys
104. What is meant by LAST
DITCH in this sentence?
The aged bishop prepared to
fight to the last ditch to defend his
good name.
A. Ones last courage C. Ones last
hope
B. Ones last strategy D. Ones last
defense

105. Which is the BEST way to


write the underlined portion of this
sentence?
Recently psychologists have been
researching birth order, their
research suggests that personality
and intelligence are based partly
on where a child ranks in the
family.
A. Order, Their
C. Order, or
their
B. Order and their D. Order, their
106. What is meant by TWO PINS
in this sentence?
For two pins I could have hit him on
the nose.
A. A second course of action
C. The second chance
B. Without much persuading
D. Have a second alternative
choice
107. Which is the BEST way to
write the underline portion of this
sentence?
The hosts also benefit from running
such a business because they can
stay at home make money, and
meeting a variety of people.
A. Get to meet
C. And meet
B. To meet
D. And be meeting
108. Which is the BEST way to
write the underlined portion of the
sentence

The studies revealing that, for


various reasons, girls spent less
time working with computers than
boys.
A. Revealing studies
C. Studies revelations
B. Studies revealed
D. Studies will reveal
109. Which word contains the
voiced Th?
A. Thank
C. Think
B. These
D. Thing
110. Which word is read on a high
note to describe the kind of day?
This is a cold day.
A. Is
C. Day
B. Cold
D. This
111. What is the mood of these
lines?
Daylight, I must wait for the sunrise
I must think of a new life
And I mustnt give in.
When the dawn comes
Tonight will be a memory, too
And a new day will begin.
A. Afraid
C. Depressed
B. Sarcastic D. Hopeful
112. Which is the BEST way to
write the underlined portion of this
sentence?
However, if a fire extinguisher is
handy, a quick-thinking person
often can use them to put out small
fire.
A. They
C. Them

B. Him
D. It
113. What correction should made
to the sentence?
Buying fire extinguisher knowing
how to use it, and placing it in a
location familiar to all family
members can help protect families
against fire.
A. Insert a comma after
extinguisher
B. Change the spelling of families
to familys
C. Change placing to place
D. Insert a comma after help
114. Which one is the right to
human dignity?
A.
Choose the goals and means
of development
B.
Share in scientific and
technological advances of the
world
C.
Right to information
D.
Sovereignty over our natural
resources
115. Which antidote would have a
similar effect if vinegar or citrus
juice were not available?
A. Milk
C. Vegetable
oil
B. Raw egg white D. Water
116. Which method of reproduction
provides for the most variety of
offspring?

A. Cloning
C. Asexual
reproduction
B. Sexual reproduction
D.
Cellular reproduction
117. Spouses Jose S. Luz and
Celerina Luz filed a petition to
adopt Gregorio Luz Ona, their
nephew. The spouses are childless
and they reared from his birth 1971
until 1975 and they continue to
support him. Gregorio had to be
left in the Philippines when the
spouse went to the United States
where Jose is employed. The MSSD
recommended the adoption to the
court on the premise that
petitioners are in a better position
to provide for the minor child than
the natural parents who are
impoverished.
What could be the possible
decision of the court on he
petition?
A.
The court denied the petition
because the spouses are already
aliens.
B.
The petition for adoption was
granted because the court finds
that it is to the best interest of the
child.
C.
The petition for adoption was
denied because the spouses are
non-residents of the Philippines.
D.
The court denied the petition
on the premise that the trial

custody required by PD 603 cannot


be effected for spouses are nonresidents.
118. The undeclared war in Korea
most closely resembled the
situation of
A. The Spanish-American War
C. World War I
B. Vietnam War
D. World War II
119. Which is the MOST important
perceived need and problem of the
Filipino family?
A. Unemployment or financial
problem
B. Proveness to vices
C. Protectiveness of children
D. Double standard on the roles of
male and female
120. Which has become a
prototype of other schemes that
defeated the real and true purpose
of the CARL?
A. The conversion of farmlands to
industrial complexes
B. The stocks option scheme of
Hacienda Luisita
C. The conversion from agriculture
to subdivision
D. The voluntary offer to sell
121. Carolina Diaz filed a petition
for habeas corpus against Mr. and
Mrs. Ramon Alde to recover
custody of Lina Diaz Tan alias

"Gracia Alde, the natural daughter


of Carolina Diaz, who was a
hostess.
What could be the possible action
of the court on the petition filed by
Carolina Diaz?
A.
Her petition would be
granted because she now works as
a clerk in a prestigious office.
B.
Her petition would be denied
because she was a former hostess.
C.
Her petition would be denied
because when Gracia was given to
the Aldes it was tantamount to
abandonment of the child, resulting
to termination of parental authority.
D.
Her petition would be
granted because she is the natural
mother.
122. Why did the register of deeds
charge Lucio Cruz registration fee
the instrument relative to his loan?
A. His loan was 30,000.00
C. His loan was P50,000.00
B. His loan was P60,000.00
D. His loan was P40,000.00
123. Which one BEST defines
personal integrity?
A. The unity between
ignorance and reality
B. The unity of mans deeds,
words, thoughts and realities
C. The unity of mans social,
political and physical aspects

D. The relationship between


virtue and conduct
124. Which of the following
ethnocentric behavior?
A. A tourist who lectures his foreign
hosts on the uncivilized nature of
their marriage customs
B. A student who tutors an
immigrant in English
C. A Hispanic community group
demands that public aid forms be
published in English and Spanish
D. A peace Corps volunteer who
helps dig wells in Central Africa
125. Which one is the right to
human dignity?
A. Political independence
B. Honor and reputation
C. Form association
D. Social and economic
reforms
126. Where would you commit a
drug dependent for him to achieve
a natural, tensionless, and anxietyfree state?
A. NFPI
C. DARE
B. DARN
D. Bukang Liwayway
Center
127. Mary Rose, an 18 year old was
sexually abuse by 3 teenagers from
well-to-do families from Makati.
Despite pressures, she came out
into the open to get justice. Which
need did Mary Rose satisfy?
A. Need for family unity

C. Need for universal solidarity


B. Need for civic responsibility
D. Need for personal integrity
128. 10
- 3 2/7 or
10-3 2/17=
__________
A. 7 2/17
C. 6 1/17
B. 6 2/17
D. 6 15/17
E. none of these
129. At which time during the year
does the ozone level present a
particular health threat in urban
areas for people with respiratory
problem?
A. Spring C. Summer
B. Fall
D. Winter
130. What day follows the day
before yesterday if 2 days from
now will be Sunday?
A. Tuesday
D. Wednesday
B. none of these E. Thursday
C. Saturday
131. Which is the BEST evidence
that helium gas is lighter than air?
A. Helium has the lowest boiling
point of all elements.
B. Helium atoms do not combine
with other air atoms.
C. Helium-filled balloons rise in air.
D. By volume, helium makes up
only 0.0005% of air.
132. During a recent shopping
spree, Tomas and Nena bought
some new accessories for their
apartment. Nena choose a

crocheted throw pillow at P24.95,


and Tomas purchased a rural
landscape painting for P135.00.
How much did they actually spend
if they paid 7% sales tax on their
purchases?
A. P 171.15 D. P 159.95
B. P 139.25 E. P 148.75
C. none of these
133. Which location will have most
nearly twelve hours of daylight and
twelve hours of darkness during
December? A town that is located
A. halfway between the equator
and South Pole
B. close to equator
C. close to the North Pole
D. close to the South Pole
134. A nation in which loess would
likely to be found today is
no answer
A. Iceland
C. Japan
B. United States
D. Ecuador
135. It was also discovered that the
chemical 3m butylphthalide can
lower the blood pressure of rats.
This statement is classified as
________.
A. prediction
C. finding
B. experiment
D.
nonessential fact
136. Which is NOT among the
hazardous effects of water pollution
to health?

A.
The epidemic threat of
hepatitis and dysentery
B.
The increase incidence of
liver cancer
C.
The dumping of mercury in
the sea causing blindness, brain
damage or death
D.
The presence of certain
bacteria in the digestive tract
causing methemoglobinemia
137.Without the process of meiosis,
we can infer that offspring from
sexual reproduction would
A.
have a high degree of
genetic variety
B.
have twice assigned number
of chromosomes
C.
be identical
D.
have a number of mutations.
138. The Jones family has four
children, all girls. The fifth child
born is a boy. This change is the
result of
A. conception classes takes by
the parents
B. the timing of fertility cycles.
C. the fathers contribution of a
Y chromosomes.
D. the law of averages finally
catching up.
139. Which of the following will
occur if a cold bottle of soda is left
open on a kitchen counter?
A.
The pressure that the soda
exerts on the bottle will increase.

B.
The temperature of the soda
will decrease.
C.
The amount of dissolved
carbon dioxide gas will decrease
D.
The amount of dissolved
carbon dioxide gas will remain the
same.
140. What do you predict will
happen when you bring two bar of
magnets closer together?
A. They will repel each other.
B. They will create an alternating
current.
C. Noing will happen.
D .They will attract each other.
141. A tightly coiled spring
demonstrate?
A. Steam energy C. Potential
energy
B. Kinetic energy D. Chemical
energy
142. What does a stick of unlit
dynamite demonstrate?
no answer
A. Chemical energy C. Kinetic
Energy
B. Nuclear Energy
D. Potential
Energy
143. Which is the main goal of drug
abuse education?
A. Arrest
C. Control
B. Prevention
D.
Rehabilitation

Situation 1 Below is the poem


written by Edgar Lee Masters in
1915:
At first I suspected something
__________
She acted so calm and absentminded.
And one day I heard the back door
shut,
As I entered the front, and I saw
him slink
Back of the smokehouse into the
lot,And across the filed.
And I meant to kill him on sight.
But that day, walking near Fourth
Bridge,
All of a sudden I saw him standing,
Scared to death, holding his
rabbits,
And all I could say was, Dont,
Dont Dont,
As he aimed and fired at my heart/
144. Who is the speaker of this
poem?
A. Tom Merritt
C. God
B. Merritts wife
D. The sheriff
145. The way in which the poet
present these words in line 12
implies that Tom
A.
tried to annoy the other man.
B.
was shot before he finished
the statement
C.
did not want to hurt the other
man.

D.
begged the man to stop
seeing his wife.
146. The poet introduces the poem
with lines 1-3 to show us that Tom
Merritt
A. had suspected that his wife
was seeing another man
B. was sure that his wife was ill
C. was a very suspicious
person
D. was sure that his wife still
loved him
147. Which of the following
techniques is used in the poem?
A. Verse C. Free verse
B. Rhyme D. Personification
Situation 2 below is an excerpt
from John F. Kennedys Inaugural
Address. Read the excerpt and
answer the questions that follows
In your hands, my fellow
citizens, more than mine, will rest
the final success or failure of our
course. Since this country as
founded, each generation of
Americas has been summoned to
give testimony to its national
loyalty. The graves of young
Americans into answered the call to
service surround the globe.
Now the trumpet summons
us again-not as a call to bear arms,
though arms we need, not as a call
to battle, though embattled we are;
but a call to bear the burden of a

long twilight struggle, year in and


year out, rejoicing in hope, patient
in tribulation, a struggle against
the common enemies of many
tyranny, poverty, disease, and war
itself.
Can we forge against these
enemies a grand and global
alliance, North and South, East and
West, that can assure a more
fruitful life for all mankind? Will
you join me in this historic effort?
In the long history of the
world, only a few generations have
been granted the role of defending
freedoms in its hour of maximum
danger. I do not shrink
responsibility; I welcome it. I do
not believe that any of us would
exchange places with any other
people with any other people or
any other generation. The energy,
the faith, the devotions which we
bring to this endeavor will light our
country and all who serve it, and
the glow from that fire can truly
light the world.
And so, my fellow Americans,
ask not what your country can do
for you; ask what you can do for
your country.
My Fellow citizens of the
world, ask not what America will do
for you, but what together we can
do for the freedom of man.

148. One of the purposes of the


speech is to motivate listeners to
A. serve their country
C. prepare themselves for battle
B. enlist the armed forces
D. preserve the right to bear arms
149. The speech is characterized
by all of the following stylistic
devices
EXCEPT
A. the use of the personal
pronouns we and us to build
rapport with listeners
B. catchy turns of phase in
which subjects and objects are
inverted
C. a standard, predictable
rhythm and the use of rhyme
D. the repetition of key words
150 The tone of the speech can
BEST be characterized as
A. sad
C. light-hearted
B. uplifting
D. sarcastic
151. In the speech, Kennedy paints
a picture of the United States as a
nation that is
A. longing to return to the past
C. struggling to survive
B. on the brink of world war
D. the leading defender of freedom
Situation 3 Below is a
Boigraphical sketch of an American
movie writer:
How did Elvis Presley Achieve
Recognition

Success often comes to those


with humble beginnings. Elvis
Aaron Presley was born on January
8, 1935 in Tupelo, Mississippi. He
first sang in a church anf taught
himself to play the guitar, but he
never learned to read music. By
1953, he had moved to Memphis,
Tennessee, graduated from high
school. And enrolled in night school
to become an electrician. That
year, at Sun Records, Presley
recorded a personal record for his
mother, a song that was heard by
the companys president. As a
result of the presidents recognition
Presleys first record Thats All
Right Mama, was out in 1954.
He toured the South, and in
1955 five of his record were
released simultaneously. His first
national television appearance was
that year of Jackie Gleasons The
Stage Show. But Presley became
known for his appearance on The
Ed Sullivan Show, where young
singer gyrated as he sang rock n
roll music. During the live
television performance, Presley
wad photographed only from the
waist up because his motions were
considered obscene.
Elvis the Pelvis began his
film career in 1956 with LOVE ME
TENDER and signed a long-term

film contract. The movie critics


were not always kind, but
teenagers flocked to Presleys
films. Within a few short years,
Presley had established a career
that would span twenty-five years
of ups and down and make him one
of the most popular entertainers in
history. Long after his untimely
death at age 42, Presley would be
remembered as The King of Rock
n Roll.
152. The author uses the phrase
ups and downs to refer to Presleys
A. gyrations as performed
B. increasing and decreasing
finances
C. successes and disappointments
in his career
D. use of drugs, uppers and
downers
153. The main idea of the sketch is
that
A. singers are more
successful if they appear in films
B. there has always been
obscenity on television
C. opportunity and luck are
often as important as hard work
D. celebrities are usually
more famous after their death
154. The last sentence reveals that
the authors attitude toward
Presley
is one of

A. indifference
C. disbelief
B. admiration
D. disgust
155. The statement that success
often comes to those with humble
beginnings would apply best to
which of the following figures?
A. Ramon Magsaysay
C. Corazon C. Aquino
B. Gloria M.
D. Joseph Estrada
Situation 4- The poem below is
entitled Suburban Prophecy
which is
written by Howard
Nemerov
On Saturday, the powermowers whine
Begins the morning. Over
this neighborhood
Rises the keening, petulant
voice, begin
Green oily teeth to chatter
and munch the cud.
Monster, crawling the carpets
of the world,
Still send from underground against
your blades
The roots of things battalions
greens and curled
And tender, that will match
your blades with blades
Till the revolted throats shall
strangle on
The tickle of their dead, till
straws shall break

Crankshafts like camels, and the


sun go down
On dinosaurs in swamps.
And night attack
Follows and by the time the
Sabbath dawns
All armored beasts are eaten by
their lawns.
156. To what does the phrase your
blades in line 8 refer?
A. Lawmowers
C. Carpets
B. Roots
D. Monster
157. The poets use of words such
as whine, voice, teeth, chatter and
munch is to suggest that the
power-mowers are
A. very powerful
C. like cows
B. alive
D. green
158. The imagery in the first stanza
appeals to the readers sense of
A. sight C. smell
B. touch D. hearing
159. How long does the action of
poet take place?
A. A week
C. An
Afternoon
B. Twenty-four hours D. A
morning
Situation 5 Ang sumusunod ay
isang talumpati na may pamagat
na SA KABATAAN
na isinulat ni
Onofre Pagsanghan
Isa sa mga salitang napagaralan natin sa wikang Pilipino ay

salitang NABANSOT. Kapag ang


isang bagay daw ay dapat pang
lumaki ngunit itoy tumigil na sa
paglaki, ang bagay na ito raw ay
NABANSOT. Marami raw uri
ngpagkabansot ngunit ang
pinakamalungkot na uri raw ay ang
ng isipan, ng puso at ng diwa.
Ang panahon ng kabataan ay
panahon ng pagklaki, ngunit ang
ating paglaki ay kailangang paglaki
at pag-unlad ng ating buong
katauhan. Hindi lamang ng atinmg
sukat at timbang. Kung ga-poste
man ang ating at ga-pison man
angating bigat ngunit kung ang
pag-iisip namat natiy ga-kulisap
lamang kay pangit na kabansutan.
Kung tumangkad man tayong
tangkad-kawayan at bumilog man
tayong bilog-tapayan, ngunit kung
tayo namay tulad ni bondying ay
di pagkatiwalaan anong laking
kakulangan. Kung magkakatawan
tayong katawang Tarza at
mapatalas ang ating isipang
sintalas ng kay Rizal, ngunit kung
ang ating kalooban namay itim na
duwende ng kasamaan anong
kapinsalaan para sa kinabukasan.
Kinabukasan, kabataan, tayo
raw ang pag-asa ng inang Bayan.
Tayo raw ang maghahatid sa kanya
sa lagnit ng kaganaan at
karangalan o hihila sa kanya sa

putik ng kahirapan at kahihiyan.


Ang panahon ng pagkilos ay
ngayon, hindi bukas, hindi sa isang
taon. Araw-araw ay tumatawid
tayong palangit o bumabaluktod
tayong paputik. Tamang-tama ang
sabi ng ating mga ninunong kung
ano raw ang kinamihasnan ay
siyang pagkakatandaan. Huwag
nating akalaing makapagpapabaya
tayo ng ating pag-aaral ngayon at
sa araw ng bukas ay bigla tayong
maging mga dalubhasang
magpapaunlad sa bayan. Huwag
nating akalaing makapaglulublob
tayo ngayon sa kalaswaan at
kahalayan at sa mahiwagang araw
ng bukas bigla tayong magiging
ulirang mga magulang.
Kabataan, tunay na pag-ibig
sa bayan, ang tunay na
nasyonalismo, ay wala sa tamis ng
pagnarap wala rin sa pagpag ng
dila. Ang tunay na pag-ibig ay
nasa pawis ng gawa.
160. Alin salita ang paulit-ulit na
binabangit ni Onofre Pagsanghan?
A. Nabansot
C. Kabataan
B. Bayan
D. Kung
161. Sa alin makikita ang tunay na
NASYONALISMO?
A. Diwa
C. Sulat
B. Gawa
D. Salita
162. Bakit di dapat tumangkad
tulad ni bondying?

A.Di ito mapagkakatiwalaan


C. May kakulangan ito
B.Di totoo ito
D. Magulo
kasi ito
163. Alin sa mga sumususnod ang
mensahe ng taluimpati?
A.
Ang mataas na paniniwala at
taimtim na pananalig ay
kailangang taglayin upang ang
hangarin sa buhay ay ating kamtin.
B.
Ang panahon ng kabataan ay
panahon ng paglaki at
pagbabagong makabuluhan.
C. Ang gawa ang siyang sukat
ng kadakilaan.
D. Ang kabataan ay siyang
pag-asa ng bayan.
164. Anong tayutay ang tinutukoy
nito?
Durog ang katawang bumagsak sa
semento si Miguel
A. Pagtutulad
C.
Pagmamalabis
B. Pagbibigay katauhan D.
Pagwawangis
165. Anong aral ang ibinibigay ng
sumususnod na salawikain?
Ang taong napapailalim ay
naipapaibabaw rin.
A. Maaring ngayon ay hirap
pagdating ng bukas ay may
ginhawa rin
B. Tiyak ang pag-unlad kapag
nauna ang hirap

C. Kung ano ang ibig natin ay


mangyayari
D. Magtiis kung dumarating
ang hirap.
166. Sa aling salita magkakaroon
ng saglit na paghinto kung
pinapilitang si Rose ang nakabasag
ng pinggan?
Hindi si Rose ang nakabasag ng
pinggan.
A. Rose
C. Hindi
B. Pinggan
D. Nakabasag
167. Alin ang naayong pamagat sa
tanagang sinulat ni Jose Villa
Panganiban?
Ano man sa daigdig,
Maaring magamit,
Ano mang masaisip;
Di sukat maiipit.
A. Pagkainip
C. Pag-asa
B. Paraya
D. Pagbibigay
168. Ano ang ipinahihiwatig ng
salitang may salungguhit?
Matayog ang lipad ni Pepe kayat
bata pa siyay nagsisikap na siya.
A. May kayabangan si Pepe.
C. Marunong si Pepe.
B. Mataas ang pangarap ni Pepe.
D. Ibig ni Pepeng maabot ang
langit.

169. Alin antas ng tono ng


lumilitaw sa bahaging may
salungguhit ng pangungusap na
nagdududa?
Nagpuputol ng puno ang lalaki.
A. 1
C. 4
B. 2
D. 3
170. Alin ang kahulungan ng
AGAW-BUHAY?
A. Masiglang-masigla
C. Pagpapatuloy ng buhay
B. Malapit ng mamatay
D. Mahirap na buhay
171. Kaninong tula hango ang
sumusunod?
Ang hindi magmahal sa sariling
wika mahigit sa hayop at
malansang isda
A. Jose Rizal
C. Apolinario
Mabini
B. Emilio Jacinto D. Graciano Lopez
Jaena
172.Ano ang pinakaangkop na
kahulugan nito?
Nagsasaya tayo ngayon sapagkat
ang inyong namatay na kapatid ay
muling nabuhay; ang nawawala ay
muling nakita.
A. Ang pagbabalik ay dapat
ipaghanda nang malaki.
B. Ang pagbabago ng kapatid
ay dapat pahalagahan.
C. Dapat silang magsaya sa
muli nilang pagsasama-sama

D. Ang pagsasama nila ay


dahil sa muling pagbabalik ng
kapatid.
173. Alin ang kahulugan ng
KAHIRAMANG SUKLAY?
A. Kakilala C. Karibal
B. Kaibigan
D. Kalahi
174. Alin sa mga sumusunod ang
aral na ibinibigay ng epikong
Muslim na INDARAPATRA AT
SULAYMAN?
A. Pagmamahal
C. Katapatan
B. Katapangan
D. pagtanaw ng utang-na-loob
175. Alin sa mga salita ang
kasingkahulugan ng salitang may
salungguhit?
Ang ama ni Anita ay kilalang
bulanggugo sa kanilang lalawigan
A. Laging ibinubulong
C. Laging handang makipag-away
B. Laging handang gumasta
D. Laging handang makipagtalo
176. Si Mariano Ponce ay
propagandistang may sagisag sa
panulat na ________.
A. Tamaraw
C. Kapre
B. Tikbalang
D. Kalapate
177. Alin ang di karaniwang anyo
ng pandiwang HINTAY KA?
A. Tay
C.Intay
B. Tayka
D.Teka
178. Ano ang kahulugan ng
taludtod na ito

Ang anak mo ay alagaan sa


marubdob na pagsuyo sikapin mo
sa sariliy huwag siyang maging
luko talipandas sa paglaki na sa
sama marahuyo sa lahi mot sa
Bayan moy isang tinik sa balaho.
A. Mahalin ang anak ng walang
hangganan.
B. Tamang pagpapalaki sa
anak ang dapat.
C. Suyuin ang anak at ibigay
lahat ng hilig.
D. Paligayahin ang tahanan.
179. Which is the BEST way to
write the underlined portion of this
sentence?
Many viewers taped shoes to watch
later.
A. Tapped C. Tape
B. had taped
D. Had tapped
180. Which word in the passage
does NOT require a change in
pitch to show confidence
I am the master of my fate, I am
the captain of my soul.
A. Am
C. Master
B. Captain
D. Fate
181. Which pitch is used for the
word STRANGE in this sentence?
What a strange story!
A. 3
C. 4
B. 2
D. 1
182. Carl Sundburg wrote Jazz
Fantasia which has for its first
stanza:

Drum on your dreams, better on


your bajos, sob On the long cool
winding saxophones.Go to it, O
jazzmen.
Which words illustrate alliteration
A. Batter and banjos C. Sob and
winding
B. Long and cool
D. to and
it
183. Anong uri ng panghalip ang
salitang may salungguhit sa
pangungusap?
May padalang tulong ang
pamahalaan para sa kanila.
A. Pambalana
C. Paari
B. Palagyo
D. Palayon
184. Alin uri ng parirala ang may
salungguhit sa pangungusap?
Utang sa kanyang sipag at sikap sa
paggawa ang kanilang maalwang
pamumuhay
A. Pangngalan
C. Pawatas
B. Pangngalanng-diwa D. Pangukol
185. Sabihin ang aspekto ng
pandiwa sa pangungusap na ito.
Mag-aral sa bahay ng mga araling
ukol sa halaman.
A. Pawatas
C.
Imperpektibo
B. Kontemplatibo D.
Perpektibo
186. Kilalanin ang uri ng
pariralang may salungguhit.

Ang pangangalaga sa mga


likas na yaman ay tungkulin nating
lahat.
A. Pangngalan
C. Pang-ukol
B. Pangngalang-diwa D. Pawatas
187. Si Dr. Jose Rizal ay sumulat
ng aklat ng itinampok sa ibat ibang
bansa.
Ang pangungusap ay nagagamit
bilang _______________.
A. panuring
C. tuwirang layon
B. pamuno D. paksa
188. Lines 11 and 12 are taken
from the poem
maggie and milly and molly
and may.
For whatever lose (like a you or a
me)
Its always ourselves we find in the
sea
Which of the following ideas is the
author expressing?
A.
The sea is a source of life and
death.
B.
The sea represents all of our
moods.
C.
The sea is the best place for
a person to reflect about life.
D. The sea and its
surroundings can give people a
fresh view on life.
189. Nasa anong kaganapan ng
pandiwa ang pangungusap?

Naglaro ng basketball sa Rizal


Stadium ang koponan ng aming
pamantasan.
A. Sanhi
C. Kagamitan
B. Tagaganap
D. Ganapan
190. Alin antas ng tono ang
lumitaw sa bahaging may
salungguhit ng pangungusap na
nagsasalaysay? Magbabasa ng
mga gawain ang guro
A. 1
C. 2
B. 3
D. 4
191. Dadalaw sa mga paaralan si
Dr. Filemon S. Salas, ang
tagapamanihala ng mga paaralang
lungsod, sa lungsod ng Pasay.
Ang pangungusap ay nagagamit
bilang _____________.
A. panuring C. tuwirang layon
B. paksa
D. pamuno
192. Alin sa mga sumusunod ang
aral na ibinigay ng ANG ALAMAT NI
MARIANG MAKILING na ikinuwento
ni Jose Rizal?
A. Pagyamanin at pangalagaan ang
ating bayan at lahi pagkat hiyas at
yaman natin ito.
B. Pag-ibig ang
makapagbabago sa mundong ito.
C. Kabanalan ang magpatawad at
tulungan ang isang nagkasala.
D. Dahil sa pagmamalabis at
pagsasamantala, maraming biyaya
ang sa kanyay nawawala.

193. A Politician wants to get his


message to 2/3 of the population of
48,000
in Bulacan. However advertising
campaign reaches only 3/ 4 of the
number
he intended. How many people
does he actually reach
A. 16,000
C. 24,000
B. 10,000
D. 36,000
E. none of these
194. Alin sa mga sumusunod ang
mensahe ng epiko ng Ilokano na
BIAG NI LAM-ANG?
A. Pinatutunayan ng epiko ang
yaman ng Ilokano sa lahat ng
bagay.
B. Kailangan paniniwalaan ang ukol
sa bisa ng mga anting-anting dahil
sa mga pangyayaring
nagpapatunay dito.
C. Dito nagpapatunay na walang
kamatayan.
D. Masasalat ang mga katutubong
ugali at mga tradisyong dapat
pagyamanin at panatilihin upang
pakinabangan ng kabataan.
195. Alin ang di karaniwang anyo
ng pandiwang WINIKAKO?
A. Ikako
C. Kako
B. Wikako
D. Wika ko

196. The Miranda Family


purchased a 250-pound side of
beef and had it packaged. They
paid P365.00 for the side beef.
During the packaging, 75 lb of beef
were discarded as waste. What was
the cost per pound for packaged
beef
A. P 2.08 per lb
D. P 2.06 per lb
B. none of these
E. P 2.30 per lb
C. P 2.50 per lb
197. Which is the sum of the
infinite progression 3/2. 1, 2/3,
4/9?
A. 6 1/2
C. 4
B. 5
D. 7
E. none of these
198. What indoor relative humidity
range would probably be
comfortable to preventing
temperature and humidity levels
are extremely low?
A. 90% to 100 %
C. 20% to 30 %
B. 60% to 70 %
D. 30% to 40 %
199. As a representative of the
Urban Poor Commission of the
Association of Religious Superiors
(ARS), which action will you most
likely take to resolve the long-term

roots of structural inequalitiesproliferation of child labor and child


prostitution?
A. Raise views of human rights
abuse.
B. Organize regular programs for
information and discussion of
human rights.
C. Conduct skills training.
D.
Raise questions over
governments commitment to
rebuild human rights.

200. What are the next four


numbers in this sequence
8,5,4,9,17_____,_____,____
A. 4,3,2,1
D. none of these
B. 5,4,3,2
E. 3,2,1,0
C. 6,3,2,0

10.
ANSWER KEY
GENERAL EDUCATION

11.

Secondary
1.
2.
3.
4.
5.
6.
7.
8.
9.

C
151.
A
152.
B
153.
D
154.
D
155.
B
156.
C
157.
B
158.
A
159.

12.
51.

101.

52.

102.

53.

103.

54.

104.

55.

105.

56.

106.

57.

107.

58.

108.

59.

109.

13.

14.

15.

16.

17.

NO ANSWER

18.

NO ANSWER

19.

NO ANSWER
NO ANSWER

20.

C
160.

D
161.

A
162.

C
163.

D
164.

C
165.

C
166.

B
167.

C
168.

D
169.

B
170.

60.

110.

61.

111.

62.

112.

63.

113.

64.

114.

65.

115.

66.

116.

67.

117.

68

118.

69.

119.

70.

120.

21.
22.
23.
24.
25.
26.
27.
28.
29.
30.
31.
32.
33.
34.
35.

C
171.

71.

A
172.

A
173.

C
174.

B
175.

D
176.

D
177.

C
178.

B
179.

A
180.

D
181.

B
182.

A
183.

B
184.

A
185.

72.
73.
74.
75.
76.
77.
78.

E
C
A
B
B
A
A
A

121.
122.
123.
124.
125.
126.
127.
128.

D
NO ANSWER
D
C
B
C
D
D

36.
37.
38.
39.
40.
41.
42.
43.

E
186.

C
187.

C
188.

A
189.

A
190.

D
191.

C
192.

A
193.

79.

129.

44.

C
194. D

80.

130.

45.

C
195.

131.

46.

A
196. A

82.

132.

47.

C
197.

B
198.

A
199.

D
200.

84.
85.

D
B
D

133.
134.
135.

B
?
C

48.
49.
50.

136.

87.

137.

88.

138.

89.

139.

90.

140.

91.

141.

92.

142.

93.

143.

94.

144.

95.

145.

96.

146.

97.

147.

98.

148.

99.

149.

100.

150.

81.

83.

86.

PART II: ANALYZING TEST ITEMS


Direction: Encircle the letter of the
best answer.
1.
The following statements
refer to the characteristics of the
Philippines as one of the important
territories of Asia, except for one
A.
It is a tropical country and
geographically located in the
northeastern part of the world.
B.
Philippines has one of the
highest literacy rates in Asia with
existing policy of free and
compulsory six years in
elementary.
C.
The establishment of the first
Philippine Republic gave birth to
the first constitutional democracy
in Asia
D.
The free port of Manila is one
of the worlds busiest ports,
compared with other Southeast
Asian nations
2.
Globes are essentially
important tool in the study of
Geography. A map is more
convenient to use but all map
projections have some errors in
presentation of distance and

shapes. Given these conditions,


which among the known mapping
projections will we use to give the
true size and shape of the earths
land masses?
A. Azimuthal Mapping
C. Equal Area Mapping
B. Conformal Mapping
D. Equidistant Mapping
3.
Most scientists believe that El
Nio phenomenon which was
greatly experiences in 1982, 1983
occurred when trade winds that
normally blow from east to west
slow down and actually change
direction. The winds blow warm
water toward South America where
the normal weather patterns are
changed for a time. Given this
global change, which among the
following conditions is not
considered as direct effect of the El
Nio phenomenon?
A.
Occurrences of hurricanes
and heavy rainfall in the mid-Pacific
region in a normal condition
B.
Some species of fish like
cold-water loving salmon
disappeared along the west coast
of the United States but tuna and
shrimp were attracted to the warm
waters
C.
The gradual warming of the
earth is known as the Greenhouse

effect could dramatically change


climates worldwide
D.
Water temperature in some
parts of the Pacific Ocean rose to
as much as 14 degrees higher than
to normal condition
4.
China has over one fifth of
the worlds population and its
leaders believe that its large
population hinders the economic
progress of the country and to
address this problem they want to
achieve equal death rate and birth
rate in population. This
government action to limit the
population is known as the
A. Baby Boom
C. Population
pyramid
B. Population distribution
D. Zero population growth
5.
Given the current condition
of Africa, it is still considered as the
most rural and least urbanized
continent in the world. With these
characteristics, which among the
following sentences is the least
contributory factor to the current
socio-political and economic
problems of Africa?
A.
Few states are ethnically
homogenous and infancy of a
strong sense of national unity
among tribal leaders
B.
Insufficient capital
technology, political instability and

poorly trained workforce to push


development among the African
nations
C.
The increasing percentage of
school-age children who did not
attend school low literacy rate and
a grate number of worlds AIDS
deaths
D.
The replacement of
Christianity among the Islamized
society in Africa along with the
continuous practice of animism
among few villages
6.
The biggest and most
populated island in the Philippine
Archipelago
A.
Cebu
D. Mindanao
B.
Leyte
E. Negros
C.
Luzon
7.
As of 1996, what sector of
Philippine economy shows an
increase near to 50%?
A. Agricultural Sector
D. Service
Sector
B. Energy Sector E. Tourism Sector
C.
Mining Sector
8.
Which of the following is not
included in the main geographical
and political-cultural subdivisions of
Asia?
A. North Asia
D.
Southeast Asia
B. Northwest Asia
E.
Southwest Asia
C. South Asia

9.
This continent is the largest
and it covers almost 33% of the
earths surface. It is consider as
the most diverse continent and
home for earliest civilization and
major religions of the world
A.
Africa
D. Europe
B.
Asia
E. North America
C.
Austria and Oceania
10. Which area of the world has
been described as the symbol of
worldwide demographic,
environmental and societal
stress/problems?
A.
Africa
D. North
America
B.
Asia
E. South America
C.
Austria/Oceania
11. Which of the following
continent is considered as the
worlds most productive in
agriculture?
A. Africa
D. North
America
B. Australia and Oceania E. South
America
C.
Europe
12. If the Caribbean Islands may
be combined with Mexico and
Central America, it could be
collectively and properly described
as:
A.
Latin America
D. South
America

B.
Middle America
E. The
Americas
C.
North America
13. The name Oceania is
widely used to refer to the
scattered group of islands
A.
Central and Southern Pacific
Ocean
D. Micronesia (Little islands)
B.
Central and Western Pacific
Ocean
E. Polynesia (Many islands)
C.
Melanesia (Black Islands)
14. What country is considered
as largest producer of Diamond,
although few of its diamonds are of
gem quality?
A.
Australia
D. New Zealand
B.
Canada
E. South
Africa
C.
Mexico
15. The worlds largest island is:
A.
Australia
D. Greenland
B.
China
E.
Madagascar
C.
Iceland
16. What sea is often included in
the physical definition of North
America?
A. Baltic Sea
D. Read Sea
B. Caribbean Sea E. South China
Sea
C.
Mediterranean Sea

17. It is the longest mountain


chain in the world
A.
Andes
D. Kunlun
B.
Himalayas E. Pamir
C.
Karakoram
18. What religious group suffered
heavily in Europe during the Nazi
Holocaust?
A.
Atheist
D. Jewish
B.
Buddhist
E. Muslim
C.
Christian
19. Most of the people in Latin
America during the colonial period
came from what known place in the
world?
A.
Amazon Rainforest
D. Oceania
B.
Andes Mountain
E. Scandinavian
C.
Iberian Peninsula
20. It is considered as one of the
worst eruption of the century that
almost affects the earths
atmosphere due to its ash falls
A. Mt. Hibuk-Hibok D. Mt.
Pinatubo
B. Mt. Kanlaon
E. Taal
Volcano
C.
Mt. Mayon
21. Which part of the earth
where we can experience six (6)
months of continuous daylight and
six (6) months continuous darkness
each year?

A.
Greenland D. North Pole
B. Greenwich, England E. Pacific
Ocean
C.
Iceland
22. In what part of the world can
you approximately find the
International Date Line (IDL)?
A.
Atlantic Ocean
D.
Pacific Ocean
B.
Greenwich, England E. Pole
C.
North Pole
23. In what part of the world can
you find the longest and largest
coral reef which is also known s the
Great Barrier Reef?
A.
Africa
D. North
America
B.
Australia
E. South America
C.
Europe
24. It is a common knowledge
that all bodies of water are
interconnected with one another
but this ocean covers 1/3 of the
world from Arctic to Antartica
A.
Arctic Ocean
D. Pacific
Ocean
B.
Indian Ocean
E. South
Atlantic Ocean
C.
North Atlantic Ocean

B.
C.

Philippines
Singapore

E. Thailand

25. Which of the following


countries is not part of the
Association of the South East Asian
Nations (ASEAN)?
A.
Malaysia
D. Taiwan

29. The opening of the Suez


Canal connects the trading route
between these two bodies of water
A. Atlantic Ocean and Pacific Ocean
D. Mediterranean Sea and Red Sea

26. Among the South American


nations, this country is the only
Portuguese speaking country
A.
Argentina
D. Mexico
B.
Brazil
E. Peru
C.
Chile
27. The first African nation that
tried to establish industrialization
to lessen the worsening
unemployment rate of the country
A.
Bostwana D. South Africa
B.
Egypt
E. Zaire
C.
Kenya
28. It is known to the ancient and
modern world that this is the home
of rich and key cities of the world.
It is also of the major tourist
destination and it provides a well
designed airports and railways
across the continent
A.
Africa
D. Europe
B.
Asia
E. South America
C.
Australia

B. Caribbean Sea and Gulf of


Mexico
E. South China Sea and Philippine
Sea
C.
Indian Ocean and Bay of
Bengal
30. This is the known Peninsula
in Europe that occupied by Norway,
Sweden, Denmark also included is
Iceland and Finland
A.
Apennines D. Jutland
B.
Balkan
E.
Scandinavian
C.
Iberian
PART III: ENHANCING TEST TAKING
SKILLS
Direction: Encircle the letter of the
best answer.
1.
Map projections have
particular purposes; specifically
what do we call projections that
show all areas on Earths surface in
proper proportion to visualize
patterns of distribution with spatial
area?
A. Albers conic projection
D. Lamber azimuthal projection
B. Equal area projection
E. Mercator projection
C. Gnomonic projection
2. When these imaginary lines are
used in combination, it gives you a
unique destination to every point
on Earth?
A.
International Date Line (IDL)

D. North Pole and South Pole


B.
Lines of Longitude and
Latitude
E. Prime Meridian &
Equator
C.
North, East, West and South
Hemisphere
3.
Earthquakes and volcanic
activities were frequently
experienced by the Philippines due
to:
A.
Climatic change in Asia for a
long period of time
B.
Erosion caused by heavy
winds and typhoons all year round
C.
High and low humidity
brought by Northeasterly and
Southeasterly wind
D.
Short, sharp and shift flowing
rivers from Luzon to Mindanao
E.
Unstable Pacific floor under
the Philippine territory
4.
Manila has a good medical
facilities but there is a great need
for medical people specially in the
rural areas, the main factor for this
situation is:
A.
Great exodus of medial
practitioners like Nurses, Medical
Technician and Medical Doctors
going abroad
B.
Low quality of graduates that
cannot passed both local and
international standards for medical
practices

C.
Political unrest and security
of rural hospital across the nation
due to terrorism and local conflicts
D.
Small number of medical
graduates like medical doctors and
nurses to support the needs of the
country
E.
Traditional medicine is a
strong competitor for the scientific
advancement of our Medicinal
Science in rural areas
5.
The Rood of the World may
be associated to:
A.
Diverse ethnic group of Asia
B.
Frequent occurrence of
earthquakes and volcanic eruptions
C.
Numerous islands, arranged
in a series of arcs
D.
Series of high mountains and
plateaus
E.
Super continent Pangaea and
its cultural heritage
6.
In the 19th century, most
Europeans considered Africa as
The Dark Continent because:
A.
Most African nations had
been subjected to European
domination
B.
Most inhabitants are literally
black or dark people of this
continent
C.
Most of its economic and
political activities were dependent
to industrialized nations

D.
Most of the African land area
is covered by vast desert and
rugged mountains
E.
Most of the African interior
was unexplored and not colonized
by European and other neighboring
nations
7.
The following statement
refers to the general characteristics
of African people in terms of their
cultural traits except for:
A.
Few nations have developed
a strong sense of national unity
among its people
B.
Imposition of colonial
boundaries among African people
further divide the continent
C.
Indigenous churches brought
common traditions and practices
among African people
D.
Traditional values prevailed
over the African Tribes
E.
Various languages further
perpetual tribal identities over and
above national identity
8.
The population statistics as
of 2004 shows evidences that
population explosion will bring:
A.
A 100% increase by 2050 in
terms of population
B.
Africa to its greatest annual
increase among the other
continents

C.
Declined life expectancy in
developing countries due to famine
and diseases
D.
One child policy to Asian
nations
E.
Stability in terms of
population growth rate
9.
One of the major issues
between US and Mexico which led
US government to take action
against the growing population of
Mexico.
A.
Assist rapid economic growth
of Mexico to eliminate poverty
B.
Control the great demand of
farmers for the south flowing
Colorado River for dry region
C.
Control the increase of illegal
crossing of Mexicans to the US
border in search of employment
D.
Manifest a virtual veto power
of US towards Mexicos economic
policy
E.
Support peasant rebels stage
war against the state
10. What do you call the new
racial type created due to
intermarriages of most Caucasian
and African slaves brought to Brazil
and Colombia?
A.
Aborigine D. Mulatto
B.
Indian
E. Native
American
C.
Mestizo

11. Which of the following is not


a human activity that threats the
incredible biological diversity of the
Amazon Wildlife?
A.
El Nio Phenomenon
D. Massive
deforestation
B.
Development of rural
settlement E. Road construction
C.
Increased in demand for
lumber
12. The word Narcotraficantes
is associated with:
A.
Annual occurrences of El Nio
Phenomenon
B.
Colombias illegal drug trade
C.
Destruction of the Amazon
Forest
D.
Trading of Black slaves from
Africa
E.
Volcanic and earthquake
activities in the Andes mountains
13. Which of the following
reasons why people cannot
establish a permanent settlement
in Antartica?
A.
Danger of nuclear weapons
testing over the territory
B.
Environmental problems like
oil spills
C.
It is the coldest place on
earth and experiencing the
strongest wind
D.
Ozone concentration in the
area above the continent

E.
The growing controversy over
the claims of different nations over
the territory
14. The three leading financial
centers of the world included the
following key cities of:
A.
Amsterdam, Frankfurt, Seoul
D. Hong Kong, Paris, Rome
B.
Beijing, Sydney, Washington
DC
E. London, New York, Tokyo
C.
Berlin, Mexico City, Singapore
15. The leading industry both for
value and employment of many
people in Australia.
A.
Electronics & information
technology
B.
Food processing
C.
Forestry and fishing
D.
Production of machinery for
transportation
E.
Tourism
16. Chernobyl will be
remembered as
A.
Commonwealth of
Independent States
B.
Economic bloc of former
USSR to European Community
C.
Place of the nuclear reactor
explosion
D.
Site of the 1992 Olympics
E.
Space launcher of Russian
Cosmonauts that blasted

17. The following are general


characteristics of an industrialized
country except for one.
A.
Economic and political
stability
B.
High literacy rate
C.
Longer life expectancy
D.
Low infant mortality
E.
One child policy per family
18. Below are listed 5 continents
1
Africa
4 North America
2
Antartica 5 South America
3
Europe
Arrange the continents from
biggest to smallest in terms of land
area.
A.
31254
D. 4 3 2 5
1
B.
23415
E. 5 2 4 1
3
C.
14523
19. Based on the early historical
accounts of Europe these two
nations had strong ties with the
European culture and the Asian
nations.
A. Australia and New Zealand
D. North Korea and South Korea
B. India and Sri Lanka
E. Turkey and Russia
C. Iran and Iraq
20. Among the 5 continents
below
1
Africa
4
Australia/Oceania

2
Antartica 5 Europe
3
Asia
Arrange the continents from
smallest to biggest in terms of its
population based on the estimated
2004 data
A.
53124
D. 1 3 4 2 5
B.
31452
E. 4 5 1 3 2
C.
24513
21. The following statements
refer to the African condition that
hinders its potential towards
economic growth except for one:
A.
Diverse natural resources
B.
Insufficient capital
technology
C.
Political instability
D.
Poorly trained workforce
E.
Small purchasing power
22. Which of the following Asian
countries has historical ties with
European city and it was
considered as the heart of the
Byzantine and Ottoman Empire?
A. Iraq (Mesopotamia)
D. Saudi Arabia
B.
Israel
E.
Turkey
C.
Russia
23. The following occurrences are
all natural hazards that the world is
experiencing across the time
except for one:

A.

Deforestation
D. Tsunami
B.
Diastrophism
E. Volcanic Eruption
C.
El Nio and La Nia
24. This is known super continent
that broke up million and million
years ago.
A.
Atlantis
D.
Pangaea

B.
Gondwanaland
E.
Tethys
C.
Laurasia
25. Among the developing Asian
nations, it is widely promoted to be
one of the better solution to
poverty and unemployment
problems
A.
Cooperation
D.
Privatization

KEY TO CORRECTION

B.
Importation
Urbanization
C.
Industrialization

E.

PART II: Analyzing Test Items

PART III: Enhancing Test Taking Skills

1.

D 2.

3.

C4.

1.

B2.

5.

D6.

3.

E4.

7.

D8.

5.

D6.

9.

B10.

7.

C8.

11.

D12. B

9.

C10. D

13.

A14.

11.

A12.

15.

D16. B

13.

C14. E

17.

A18.

15.

B16.

19.

C20. D

17.

E18.

21.

D22. D

19.

E20.

23.

B24.

21.

A22.

25.

D26. B

23.

27.

D28. D

24.

29.

D30. E

25.

1.
The adoption of scientific
techniques to control and
manipulate environment such as
modern methods for farming and
fishing is called
PRACTICE TEST I

A. social change C. cultural


change

B. technological change

A.

symbolic

D. discovery change

B.

continuous D. changeless

2.
Heavily sanctioned folkway
which no one dares to question
because they embody moral laws
are called

6.
Aling Maria has taught her
daughter how to prepare good and
delicious desserts from local fruits
and vegetables. What cultural
transmission process is this?

A.
beliefs
mores
B.

C.

customs

D. tradition

3.
What is that inevitable
folkway which no one dares to
question because they embody
moral laws are called
A.

change

C. progress

B.
development
evolution

D.

4.
Nora Aunor became a legend
in the showbusiness because of her
talent, determination and hard
work. What role does this
illustrate?
A.
achieve role
hereditary role

C.

B.
ascribed role
temporary role

D.

5.
Pinning veil around the bride
and the groom/exchange of rings,
releasing of doves, etc. during
wedding ceremony show that
culture is

C. historical

B.
socialization
acculturation

D.

9.
Mildred, an accountant, who
now lives with a married man was
disowned by her family and friends.
This punishment is a form of
A. isolation

C. ostracism

A.
observation
enculturation

C.

B. deportation
assassination

B.
acculturation
indoctrination

D.

10. When a certain culture is


acceptable to one group and
questionable to others, it shows
that:

7.
Mr. Francisco was teaching
something on mores, folkways,
customs and beliefs in his Social
Studies class. What was his lesson
about?

D. character

A. culture is borrowed C. culture


is varied
B. culture is a shared product

A.
material culture C. tangible
culture

D. culture is relative

B. non-material culture D.
hereditary culture

11. The teaching and learning of


religion and values mostly make
use of

8.
It was the first day of school.
Miss Dela Rosa prepared activities
which will make her Grade III
children sing, play, talk, learn and
introduce themselves to the class.
What process did the teacher
emphasize?
A.
enculturation
indoctrination

C.

A.
enculturation
ulturation

C.

B.
indoctrination
acculturation

D.

12. In order to achieve the goals


for change, all include the following
strategies EXCEPT
A.
Change must involve all
sectors of the society

B.
Change must be centralized
where only a few initiate the
change

15. Which of the following factors


is the reason for communication
gap in many homes?

C.
Change must be holistic that
focuses on the individual and the
system

A.

Lack of parental guidance

B.

Modern technology

C.

Inadequate education

D.

Highly urbanized lifestyle

D.
Change should come from
the leaders and the people
13. Which change is bought
about by discovery or
modernization to increase
production?
A. cultural change C.
technological change
B. societal change D. social
change
14. Central to cultural renewal
and social change is a revolution of
expectations from ourselves as well
as from our institutions. This
concept includes the following
resources from Filipinos, EXCEPT:
A.
Assertiveness of Filipino
families
B.

A colonized education

C.

A cultural-fair media

D.
A religious that upholds
justice and righteousness

16. Which of the following


statements regarding the Values
Conceptual Framework is TRUE?
A.
It is imposing on every
student
B.
It is prescriptive of many
desirable traits
C.
It is specific for practical
usefulness
D.
It is flexible that suits to
every human individuals needs
17. The goals of Values Education
that need to be changed include all
the following EXCEPT:
A.
To become aware of the
National Issues and Problems that
beset the country
B.
To encourage OFWs to
continuous work abroad in order to
help the National Economy.

C.
To develop deep sense of
responsibility and accountability
D.
To initiative action-centered
community organizations
18. Which of the following is NOT
an example of economic values?
A.

Hard work and frugality

C. Self-determination
B.

Work values

D. Responsible parenthood
19-20.
Which among the
strategies show negative concerns
for environmental protection?
A.
Campaign against wrong
health and environmental practices
B.
Suggest the use of synthetic
substances instead of organic ones
C.
Link with organizations with
projects on waste management
D.
Cut tall trees and plant
ornamentals to beautify the
surroundings
21. Filipinos love for socials,
fiestas and celebrations is
manifestations of which Filipinos
character?
A.
Spirituality
Superstitious

C.

B.
Family Solidarity D. Joy and
Humor

A. culture shock
change

C. culture

B. severe headache
loneliness

PRACTICE TEST II
1.
An organized and
systematized manner of learning
where expectancies are controlled
by the teacher is termed as:

5.

B.
professionalism
philosophy

D. extreme

What is enculturation?

A.
Handing down of culture by a
teacher

A.
teaching
schooling

C.

B.
Handing down of culture
without any question

B.
education
motivation

D.

C.
Handing of culture from one
generation to the next

2.
A social agency that makes a
child learn to value himself and
eventually others is
A.

home

B.
school
society

C. group
D.

3.
What will surely takes place
whenever society adapts
modernization abruptly?
A. There is peace.
progress.
B. There is harmony.
confusion.

C. There is

D.
Handing down of culture from
one who knows to somebody who
doesnt know
6.
Which of the following
functions cannot be done by the
school?
A.

perpetuation of specie

B.

changing cultural practices

C.

socialization among children

D.
development of attitudes and
skills

D. There is

4.
When a person fails to adjust
to the fast changing world, he is
likely to suffer from

7.
Filipinism could be taken as
having the same connotation as a
sense of
A.
responsibility
nationhood

C.

D.

8.
The process of individual and
group change caused by contact
with various people
A.
acculturation
culturation

C.

B.
enculturation
indoctrination

D.

9.
All people have culture and
therefore share a common
humanity. This shows that
A.

culture is universal

B.

culture is organized

C.

culture is a product of change

D.
culture is the sum total of
human experiences
10. Which of the following
sciences below is more connected
with the study of social traditions?
A.

theology

C. sociology

B.

psychology D. anthropology

11. Which of the following


situations presents a value conflict?
A.
The parent expects her child
to do what he tells him; the child
obeys so.

B.
The parent wants to his child
to be in the Deans list; the child
studies hard.
C.
The child has many friends;
the parent allows him to spend
time with them during week-ends
D.
The parents priority is for
her child to be an A student, the
childs predominant interest

14. When an individual or a


group adapts the culture of others,
practice them and becomes
habitual, this is:
A.

culture lag C. culture shock

B.
culture change D. culture
difference

12. Why cant the school


implement programs for social
reconstruction?

15. The proper sequencing of


grade levels according to
chronological ages of learners is
called

A.
Most of the students are poor
and passive

A.

B.
Some teachers do not like
teaching career

status

B.
organization
institution

C. agency
D.

C.
Some teachers have no
dedication to the profession

16. Divorce is legal in the United


States but is not acceptable in the
Philippines. What does it show?

D.
Most teachers belong to Low
Socio Economic Status (SES)

A. culture is illegal

13. Which of the following best


illustrates a sense of responsibility
that a professional teacher is
expected to have?
A.

Is humble

C. Has pleasant disposition


B.

Makes good use of time

D. Tutors own student for a fee

D.

18. What is the responsibility of a


newspaper editorial in values
processing?
A.
Interprets the news for the
sale of readability
B.
Analyzes the relevance of the
personal experience with the news
material
C.
Explains the message of the
editorial
D.
Introduces socialized
recitation
19. When does the Kanyakanya syndrome become positive?
A.
When one can discover what
he likes in life
B.
When the benefit of others is
viewed as one loss

C. culture is relative

C.
When one protects the
individual interests

B. culture is uniform
D. culture is phenomenon
17. A school operates not only to
cater her students but for all the
people where it is located can be
called a
A.
public school
school

B.
private school
community school

C. societal

D.
When one becomes selfreliant and can stand on his own.
20. Which situation shows a
desirable relationship between
teachers and other groups of
people?

A.
Miss Sison issues report cards
only if she remembers.
B.
Mr. Fajardo never entertains
parents complains; he feels they
are only a pest
C.
As a class adviser, Mr.
Magalang has not had a single
conference with the parents of his
students.
D.
Mrs. Vergara is glad that the
parents of her Grade III class
volunteer to her in certain class
activities

22. When statement can be


considered features of our present
educational system?
A.

Knowledge is power

B.

Knowledge does not change

C.
Knowledge begins in the
senses
D.
Knowledge can be known
only by reason
23. When does a teacher of
Values Education a good facilitator
of learning?

statement which presupposes the


following:
A.
Facilitators of values must
grow in their own personal total
development
B.
Values education is a lifelong
process
C.
The home is the primary
source of basic values
D.
The school has the sole
responsibility to educate the youth

21. Does valuing process used as


a teaching strategy?

A.
Monitors action learning
plans for the students

25. Which is not an


anthropological view of education?

A.
Focus on the affective
processing of value learning

B.
Provides cognitive inputs
such as lecturette

A.
Education is a social
institution

B.
Emphasize on the effective
component of learning

C.
Clarifies the students values
by the use of processing questions

B.
Education is the process of
teaching and learning

C.
Emphasize on the processing
rather than on concept

D.
Imposes her own beliefs on
her students

C.
Education is an agent of
change and modernization

D.
Concentrate on the concept,
process and behavioral
components of value

24. One cannot give what he


does not have is a popular

D.
Education is boundary
breaking between social classes

KEY TO CORRECTION
PRACTICE TEST I
1.

2.

3.

4.

5.

6.

7.

8.

9.

10.

11.

12.

13.

14.

15.

16.

17.

18.

19-20. B, D

11.

21. D

12.

PRACTICE TEST II

13.

1.

14.

2.

15.

3.

16.

4.

17.

D18. C

5.

19.

6.

20.

C21. D

22.

C23. C

7.

24.

8.

25.

9.

10.

Licensure Examination for Teachers


General Education: Social Science
Focus: Economics (Majorship)
Prepared by: Prof. Serafin A. Arviola
Jr.

ECONOMIC ISSUES AND CONCERNS


1.
Poverty and Income
Distribution

Poverty line an amount of


income below which a family is
considered poor

Income Poverty based from


the United Nations, are those
individuals living on an income of
$1 a day or less than $1/day.

Human Development Index


measure of human development
looking at the following indexes:
longevity, income and educational
attainment

Human Poverty Index a


measure of poverty looking at the
income of individuals
2.
Globalization - The
movement of money, goods,

information and people across


nations made possible by the rapid
advances in travel and
communication.

Globalization of Goods
through importation and
exportation

Globalization of Money
through official development
assistance (ODA), investments,
grants, donations, etc.

Globalization of Information
via internet, cable TV, education,
etc.


Globalization of People
through travel, scholarships,
fellowships, job, etc.
3.
a.

Development
Sustainable Development

Development that meets the needs


of the present generation without
compromising the ability of the
future generations to meet that
own needs. United Nations
declared 2005-2014 as the Decade
on Education for Sustainable
Development. It pursues the
following global vision:
The vision of education for
sustainable development is a world
where everyone has the
opportunity to benefit from quality
education and learn the values,
behavior and lifestyles required for
a sustainable future and for
positive societal transformation.
Dimensions of Sustainable
Development

Economic Dimension

Social Dimension

Institutional Dimension

Environmental Dimension

Cultural and Spiritual


Dimension

b.
The 8 Millennium
Development Goals
1.
Eradicate extreme poverty
and hunger

Reduce by half the


production of people living on less
than a dollar a day

Reduce by half the proportion


of people who suffer from hunger
2.
Achieve universal primary
education

Ensure that all boys and girls


complete a full course of primary
schooling
3.
Promote gender equality

Eliminate gender disparity in


primary and secondary education
preferably by 2005, and at all
levels by 2015
4.
Reduce child mortality

Reduce by two thirds the


mortality rate among children
under five
5.
Improve womens
reproductive health

Reduce by three quarters the


maternal mortality ratio
6.
Combat HIV/AIDS, Malaria
and other diseases

Halt and begin to reverse the


spread of HIV/AIDS

Halt and begin to reverse the


incidence of malaria and other
major diseases
7.
Ensure environmental
sustainability

Integrate the principles of


sustainable development into
country policies and programmes;
reverse loss of environmental
resources

Reduce by half the proportion


of people without sustainable
access to safe drinking water

Achieve significant
improvement in lives of at least
100 million slum-dwellers by 2020
8.
Develop a global partnership
for development

Develop further an open


trading and financial system, that
is rule-based, predictable and nondiscriminatory. Includes a
commitment to good governance,
development and poverty
reduction nationally and
internationally

Address the least developed


countries special needs. This
includes tariff-and quota-free
access for their exports; enhanced
debt relief for heavily indebted

poor countries; cancellation of


official bilateral debt; and more
generous official development
assistance for countries committed
to poverty reduction

Address the special needs of


landlocked and small island
developing States
c.
Philippine Initiatives on
Sustainable Development
1.
Philippine Agenda 21 in 1996
2.
Philippine Council for
Sustainable Development in 1997
3.
Major legislations on
Sustainable Development
Clean Air Act of 1999
Ecological Solid Waste
Management Act of 2001

revenue to finance expenditure on


public goods and services.
Purpose of Taxation

To collect revenue for the


government

To redistribute income

To combat inflation

To correct an adverse
balance of payment

To check consumption of
goods which are considered
undesirable

To protect local infant


industries

To influence population trend

To improve unfavorable terms


of trade

To reallocate resources to
create a sense of identity

TAXATION
Sources and Origin of Taxation

It is an inherent power of the


state to impose and collect
revenues to defray the necessary
expenses of the government.

It is compulsory contribution
imposed by a public authority
irrespective of the amount of
services rendered to the payer in
return.

It is compulsory level on
private individuals and organization
by the government to raise

The Constitution

Statutes or Presidential
Degrees

Bureau of Internal Revenue


regulations

Judicial Decision

Provincial, Municipal and


Barrio Ordinances

Observance of International
Agreement

Administrative Ruling and


Opinions

Classification of Tax System

Progressive Income Tax the


Higher the income the higher the
tax rate.

Proportional Tax The tax


rate is constant and unaffected by
the level of income.

Regressive Tax The higher


the income the lower the tax rate.
Classification of Taxes

As to the subject matter


Personal Property, Capitation and
Poll Tax; Property Tax; Excise Tax
As to who bears the burden
Direct Tax and Indirect Tax

As to determination of the
amount Specific and Ad valorem

As to purpose General and


Specific

As to Scope National and


Local
Types of Taxes

Direct Taxes

The burden cannot be shifted


to the third party

Direct taxes are based on


income and wealth

In most cases, direct taxes


are progressive in nature


Direct taxes are compulsory
in nature

Examples: income tax,


residence tax, real state,
immigration tax,
estate/gift/inheritance tax.

Stability tax system must


not be too often or it will encourage
tax payers to withhold tax payment
until a more preferred system is
put in place
AGRARIAN REFORM

Indirect Taxes

The tax burden can be


shifted to the third party

Indirect taxes are based on


expenditure and consumption

All indirect taxes are


regressive in nature

Indirect taxes are optional in


the sense that they can be avoided

Examples: sales tax, import


tax, VAT/EVAT
Characteristics of a Sound Tax
System

Efficiency must generated


revenue greater than the amount
of money the government must
spend to collect taxes.

Equity individual and


groups belonging to the same
income bracket must be taxed
equally while belonging to different
income groups must be taxed
differently.

Convenience to set up
measures and procedures that will
make it more convenient for
taxpayers to pay.

Republic Act No. 6657 The


Comprehensive Agrarian Reform
Law of 1988 which was signed into
law by Pres. Corazon Aquino
Meaning - The redistribution of
lands, regardless of crops or fruits
produced to farmers and regular
farm workers who are landless,
irrespective of tenurial
arrangement to include the totality
of factors and support services
designed to lift their economic
status of the beneficiaries and all
other arrangements alternative to
physical redistribution of lands,
such as production, profit sharing,
labor administration and the
distribution of shares of stocks,
which will allow beneficiaries to
receive a just share of the fruits of
the land they work.
Principles of Agrarian Reform

The policy of the state to pursue a


comprehensive Agrarian Reform
Program (CARP) to:

To promote social justice

To move the nation toward


sound rural development and
industrialization

To establish ownercultivatorship of economic sized


farms as basis of Philippine
agriculture.
Coverage of CARP

All alienable and disposable


lands of the public domain devoted
to or suitable for agriculture

Stability tax system must


not be too often or it will encourage
tax payers to withhold tax payment
until a more preferred system is
put in place
AGRARIAN REFORM
Republic Act No. 6657 The
Comprehensive Agrarian Reform
Law of 1988 which was signed into
law by Pres. Corazon Aquino
Meaning - The redistribution of
lands, regardless of crops or fruits
produced to farmers and regular
farm workers who are landless,
irrespective of tenurial
arrangement to include the totality

of factors and support services


designed to lift their economic
status of the beneficiaries and all
other arrangements alternative to
physical redistribution of lands,
such as production, profit sharing,
labor administration and the
distribution of shares of stocks,
which will allow beneficiaries to
receive a just share of the fruits of
the land they work.
Principles of Agrarian Reform
The policy of the state to pursue a
comprehensive Agrarian Reform
Program (CARP) to:

To promote social justice

To move the nation toward


sound rural development and
industrialization

To establish ownercultivatorship of economic sized


farms as basis of Philippine
agriculture.
Coverage of CARP

All alienable and disposable


lands of the public domain devoted
to or suitable for agriculture

All lands of the public domain


in excess of the specific limits as
determined by the Congress

All other lands owned by the


governments devoted to or suitable
for agriculture

All public lands devoted to or


suitable for agriculture regardless
of the agricultural products raised
or can be raised.
Retention Limits

Five hectares for land owners

Three hectares to be
awarded to each child of the
landowner subject to the following
qualification:
o
At least 15 years old
o
Actually tilling the soil or
directly managing the farm
Beneficiaries

Agricultural lessees and


share tenants

Regular farm workers

Seasonal farm workers

Other farm workers

Actual tillers or occupants of


public lands

Collectives or cooperatives

Other directly working on the


land
Salient Features of CARP

CARP covers all agricultural


lands and not only devoted to rice
and corn

CARP covers not only those


privately owned tenanted lands but

also that of agricultural land owned


by Multinational Corporations and
commercial farms.

Lower retention limits of


three hectares

Rights of indigenous
communities, to their ancestral
lands are protected to ensure their
economic, social and cultural well
being

In determining just
compensation, the cost of
acquisition of the land, the current
value of like properties, its nature,
actual use and income, the sworn
valuation of the owner, the tax
declarations and the assessment
made by the government assessors
shall be considered.

Lands awarded to
beneficiaries shall be paid to the
Land Bank of the Philippines in 30
annual amortization at six percent
interest per annum.
COOPERATIVES
New Cooperative Laws
Cooperative Code of the Philippines
(RA 6938), Cooperative
Development Authority (RA 6939)
and Executive Order 95 and 96
issued by President Fidel Valdes
Ramos.

Definition - A cooperative is:

A free association of persons


voluntarily joined together

With common bond of


interest

Legally constituted

Purpose of conducting an
economic enterprise

Owned, controlled and


administered democratically

Making equitable
contributions to the capital
required

Accepting a fair share of the


risks and benefits

Organized in accordance with


generally accepted principles
Universal Principles of
Cooperativism

Open and Voluntary


Membership - No artificial
discrimination against individuals
because of their race, creed or
political affiliation, freedom of entry
and exit of any member of the
cooperative

Democratic Control In order


for members to gain entry to the
cooperatives, they must purchase
shares of the cooperative, obtain
the right to govern the
organization, voting rights of the

owner are on the basis of one


person, one vote.

Limited Interest on Capital


Capital in a cooperative is like a
loan because the owners of the
capital can expect to received a
rate of return not exceeding that of
the prevailing market interest rates
on investing.

Division on Net Surplus Net


surplus should be distributed as
follows:
Item
%
allocation
General Reserve Fund
At least 10%
Education/Training Fund
At Least
10%
Optional Fund
At Least 10%
Dividend/Patronage Refund
Remaining
Balance of Savings

General Reserve Funds


cover losses in operation

Education/Training Funds
for members and Management
trainings

Optional Funds discretion of


cooperatives for purposes of
acquiring land construction of a
building or community
development

Dividends/Patronage Refunds
the volume of transaction that
members have with the
cooperative

Continuing Membership

Pre-membership education
seminar as required for entry to the
cooperative

Special trainings for the


cooperative leadership and
members

Cooperation Among
Cooperative interlending and
pooling of funds
Typologies of Cooperative
1. According to Level of
Cooperatives

Primary members of which


are natural

Secondary members of
which are primaries

Tertiary members of which


are secondary upward to one or
more apex organization
2. According to Services Rendered

Credits is one, which


promotes thrift among its

members, and creates funds in


order to grant loans for productive
and provident purposes.

Consumer is one wherein


the primary purpose is to procure
and distribute commodities to
members and non-members.

Producers is one which


undertakes joint production
whether agricultural or industrial;

Marketing Cooperative is
one which engages in the supply of
production inputs to members and
in turn market their products.

Service is one engages in


medical and dental care,
hospitalization, transportation,
insurance, housing, labor,
electricity, communications and
other services.

Multipurpose is one which


combines two or more activities o
these different types of
cooperatives
3. According to Scope of
Membership

Institutional Members are


employees of a specific institution
or corporation

Associational Members are


those who have their own
enterprise and belong to specific
sector or organization.

Community-Level Members
are based on a defined
geographical area.

PREBOARD EXAMINATION

PROFESSIONAL EDUCATION

The Teaching Profession, Social


Dimensions for Education
1.
To whom does the word
teacher refer?

2.
Teacher Kevin has not
practiced his profession for the
past five years. Can he go back to
teaching immediately?

4.
Which is true of the periodic
merit exam for teacher provided
for in RA 7836?
I.

Consist of oral exam

a.
Yes, if nobody can take his
place

II.

Consist of written exam

b.
No, unless she has enrolled in
refresher course of 12 units

III.
May serve as additional basis
for merit promotion in addition to
performance rating

c.

No

d.

Yes

IV.
Taken with fee of P 1000 per
examinee
a. I only

Guidance counselors

3.
Is membership to the
accredited professional
organization for teachers
mandatory for all LET passers?

IV.

Librarians

a.

d. II only

V.

Division Superintendent

a.

I, II, and III

b.
Yes, when the teacher is
already teaching

b.

I and III

I.

Full time teachers

II.

Part time teachers

III.

c.
I, II, III, and IV I,II,V BEST
ANSWER
d.

III and IV

c.

No

Yes

d.
Only for LET passers who are
not repeaters

b. I and IV
c. II and III I,II,III BEST ANSWER

5.
Can Manny Pacquiao be
given a special permit to teach
boxing in a special school?
a.
No, he is not a teacher
education graduate
b.
LET

No, he has not passed the

c.

Yes, he is a graduate of ALS

a.

Moral or ethical values

d.
Yes, he has excelled and
gained international recognition

b.
Technical and scientific
competencies

6.
Is it professional for a teacher
to receive gifts from the student
and parents?

c.

Upgraded laboratory facilities

d.

Vocational skills

a.

Not at all

b.
No, especially if done in
exchange for requested
concessions
c.

Yes, if deserved

d.
Yes, in-season and out-ofseason gifts
7.
An Education graduate
without a license is accepted to
teach in a private school? Is this in
violation of RA 7836?
a.
No provided he has taught
for at least 3 years
b.
Yes. No one may teach
without a license
c.
d.

No
Yes

8.
For relevance to business and
industry, what did the First Biennial
National Education on Education
(2008) impose for updating the
Licensure Examination for
teachers?

9.
What does the Teacher
Education Development Program
signify as a prerequisite for
employment of teachers in basic
education schools?
a.
National Standard
Competencies among teachers
b.
Licensure Examination for
Teachers
c.

Induction of new teachers

d.
Job interviews for teacher
applicants
10. Among active participation of
school officials and teachers in the
community, which of the following
is not appropriate due to prevailing
religious sentiments?
a.
Literacy assistance for out of
school children/youths
b.
Household campaign for
healthful practice
c.
Promoting contraceptives for
planned parenthood

d.
Introducing cooperative thrift
practices
11. Which of the following is not
John Deweys contribution to the
sociological foundation of
education?
a.
Facilitating learning along
social conditions of the learner
b.
As a social process,
education begins at birth
c.
True education is
transmission of knowledge
d.
The school is a continuation
of home
12. Of the following, which is
most fundamental to building up a
strong school culture of excellence?
a.
High standards of
performance
b.

Student-centered curriculum

c.

Mission and core values

d.

Student handbook of conduct

13. Among rights of the schools,


which is not provided by the law?
a.
Right for basic education to
determine subjects of the study
b.
Right to enforce
administrative systems

c.
Right to provide proper
governance
d.
Right for institutions of
higher learning to determine
academic grounds for admission
14. What kind of grassroots
model best advances Education for
All as served children of slum city
dwellers?
a.

Mobile education on Kariton

16. What is the cultural trait of


conflicting values that aims to
please people in different venues
and situations rather than abide by
principles?
Crab mentality

b.

Split personality

b.
TEDPA Technical Education
Development Program

c.

Kanya-kanya system

c.

d.

Bahala na mentality

d.
BEC Basic Education
Curriculum

c.

Radio education modules

d.

Educational television

a.

Aptitude for teamwork

15. After the implementation of


NCBTS, results of LET still reveal
low performance among
examinees. What can teacher
education institutions do to
upgrade their graduates LET
performance?

b.

Skills and social behavior

c.

Readiness to take risks

a.
Review curriculum vis--vis
TOS
b.

Intensify Field Study Courses

c.
Hire expensive review
trainers
d.
Implement selective
admission in TEIs

a.
BESRA Basic Education
Sector Reform Agenda

a.

17. Among qualities which


employers look for in the 21st
century workplace, which is the
most challenging and demanding?

b.
Leaf flyers for out-of-school
children

19. Which program directly


embodies both the pre-service and
in-service programs?

d.
Specific competencies for
work
18. In educating the whole
person as demanded by the
Learning to be pillar of the 21st
century education, where does the
concept of meaning, purpose and
engagement belong?
a.

Mind and body

b.

Aesthetic sense

c.

Spiritual values

d.

Personal responsibility

K-12

20. How can the efforts of four


agencies (DepEd, CHED, PRC, CSC)
be best achieved for the training
and development of teachers?
a.

Synchronization

b.

cost-reduction

c.

streamlining

d.

sharing of resources

21. What is the core of the


Teacher Education Development
Program?
a.
high order thinking skills or
HOTS
b.

student-centered learning

c.
National Competency-Based
Teaching Standards
d.
Technology integration in
instruction

22. What is known as a selfappraisal for professional growth


that is acceptable and useful for
recognizing weakness and
strengths for a new beginning
teacher?

a.

familiarity with new culture

a.

master teachers evaluation

d.

b.

students evaluation

c.

principals evaluation

d.

self-evaluation

25. For a school, which of the


following is most significant in
repairing shorelines with depleted
coral reefs?

b.
open-mindedness to new
culture
c.
adaptability to new work
environment
foreign-language policy

23. Among reforms for enhancing


teacher professionalism, which has
been implemented by law in order
to determine whether prospective
teachers have acquired
professional competencies prior to
granting them a permit to teach?

a.
outreach by depositing
rubber tires as artificial coral reefs

a.
accrediting a national
organization for teachers

d.
outreach by educating the
villagers on protection of coral
reefs

b.
setting up centered for
excellence in teacher education
centers
c.

licensure examination

d.
creation of a professional
board for teachers
24. From global competence as
defined by international educators,
which is the most appropriate
characteristic of globally
competent individual?

b.
implement reporting system
against dynamite fishermen
c.
legislative lobby to disallow
tourism in endangered shorelines

26. In a tertiary school, the


President organized a Fun Run for
students, faculty and personnel to
enjoy camaraderie, physical
exertion under the sun, sense of
engagement and achievement.
What does the activity promote?
a.

spiritual vigor

b.

cultural consciousness

c.

national integrity

d.

moral integrity

27. In the Education Act of 1901


which established a free public
education in the Philippines, what
language was imposed under the
one-language policy?
a.

Spanish

b.

English

c.

Tagalog

d.

Filipino

28. Of the following, which is the


most functional intervention in
order to achieve a basic right of
every Filipino Child under the
Constitution and Magna Carta for
Disabled Persons?
a.

Philosophy of education

b.

policy for curricular reform

c.

home study program

d.

structural organization

29. Of the following


interventions, which is directly
aimed at responding to the
transitional gap between academic
achievement and employment?
a.
identification of centers of
excellence
b.

deregulation of tuition fees

c.
school networking with
business and industry
d.
voluntary accreditation of
schools
30. In the formal education
system during Hispanic times in
the Philippines, what was not
implement but which we enjoyed
during the American period?
a.

vocational education

b.

private education

c.

religious education

d.

public education

31. If Dr. Jose Rizal lives in the


21st century, what character
expression and commitment would
have shown our generation?
a.

inventor of techniques

b.

citizen and producer

b.

skills

c.

insights

d.

values

33. Before being able to fully


learn to live and work together
under the pillar of the 21st century
education, what must the learner
attain for himself?
a.

find peace within oneself

b.

attain an altruistic mind

c.

love his fellowmen

d.

become self-actualized

34. The Transparency


Internationals perception that the
Philippines suffers a cultural
malaise of corruption, what
component of our character needs
to be further developed along the
Learning To Be Pillar of education in
the 21st century?

c.
member of family and
community

a.

d.

b.
Physical-economic
component

creative dreamer

32. In the learning to do pillar of


new education, what is the
enabling factor that can make the
learner fully contribute to a
peaceful and just society?
a.

knowledge

Familial-social component

c.
Intellectual-emotional
component
d.

Ethical-spiritual component

35. This powerful European


country supplied arms to

Afghanistan rebels who were


fighting a terrorist war in the
Middle East. What was the principle
of moral discernment applicable in
this case?
a.

Principle of double effect

b.

Principle of lesser evil

c.
Principle of material
cooperation
d.
Principle of moral
cooperation
36. Which of the following best
defines a morally mature person?
a.

Cultural values clarification

b.
Unhampered exercise of
ones right
c.
Transmittal of ones moral
viewpoint
d.
Knowledge and practice of
universal moral values
37. Educated in a religious
school, Sansa goes to confession
every day to be free of any kind of
sin. How do you characterize
Donas moral attitude?
a.

Callous

b.

Pharisaical

c.

Scrupulous

d.

Strict

38. How would you characterize


the moral attitude of Hispanic friars
who taught religion but were
unfaithful to their vow of property
by amassing the land properties of
natives?

c.

Perplexed

a.

Existentialist

d.

Probable

b.

Utilitarianist

c.

Pragmatic

d.

Constructivist

a.

Scrupulous

41. Teacher Slash is of the


thinking that from the very start
students must be made to realize
study is indeed hard work. To which
philosophy does Teacher Susan
adhere?

b.

Strict

a.

Essentialism

c.

Lax

b.

Perennialism

44. Who asserts that teaching is


not just depending knowledge into
the empty minds of the learners? It
is helping students create
knowledge and meaning of their
experiences?

d.

Pharisaical

c.

Progressivism

a.

Constructivist

d.

Reconstructionism

b.

Essentialist

c.

Existentialist

d.

Pragmatist

39. How would you characterize


the moral attitude of prisoners with
criminal minds, who have no
sensitivity to the welfare of other
people?
a.

Pharisaical

b.

Strict

c.

Lax

d.

Callous

40. What was the degree of


moral certitude when U.S.
statement decided to drop the
atomic bombing on Hiroshima and
Nagasaki to prevent mass deaths
by a land invasion of Japan?
a.

Doubtful

b.

Certain

42. If your students appear to be


more interested in a topic outside
your planned lesson for the day,
you set aside your lesson plan for
that day and grasp the opportunity
to discuss the topic of particular
interest to your students. Strike the
iron while it is hot! Which
philosophy governs for your action?
a.

rationalism

b.

empiricism

c.

existentialism

d.

progressivism

43. Students must be taught selfresponsibility is the desire of the


___________ teacher.

SITUATIONAL
In a faculty meeting, the principal
told his teachers: We need to
improve our school performance in
the National Achievement Test.
What should we do? The teachers
gave varied answers as follows:
1.
Lets give incentives and
rewards to students who get a
rating of 85%
2.
Lets teach them to accept
complete responsibility for their
performance

3.
Lets make the school
environment conducive for learning
4.
Lets make use of the
experiential methods of teaching
45.
On which educational
philosophy is response #1
anchored?
a.

Behaviorism

b.

Progressivism

c.

Existentialism

d.

Essentialism

46. Which response/s come/s


from a behaviorist?
a.

1 and 3

b.

2 and 4

c.

1 and 2

d.

3 and 4

47. If you lean toward a


progressivist philosophy, with
which response do you agree?

about him/her. In a faculty


recollection, the teachers were
asked to share their thoughts of
the learner, their primary customer.
What follows are the gists of what
were shared:

Teacher A The learner is a product


of his environment. Sometime he
has no choice. He is determined by
his environment.

a.

Progressivist

b.

Perrenialist

c.

Essentialist

d.

Rationalist

50. Whose response denies


mans freewill?
a.

Teacher As

Teacher B The learner can choose


what he can become despite his
environment.

b.

Teacher Cs

c.

Teacher Bs

Teacher C The learner is a social


being who learns well though an
active interplay with others

d.

Teacher Ds

Teacher D The learner is a


rational being. Schools should
develop his rational and moral
powers

a.

48. Whose philosophical concept


is that of Teacher A?

b.

a.

c.

d.

How a teacher relates to his/her


pupils depends on his/her concepts

49. If you agree with Teacher C,


you are more of a/an

Human Growth and Development,


Facilitating Learning,
Developmental Reading
51. From a broad vantage view of
human development, who has the
primary duty to educate the youths
or children?
I.

Parents

Behaviorists

II.

Teachers

b.

Existentialists

III.

the state

c.

Progressivists

IV.

the schools

d.

Rationalists

52. Of the three aspects of


learning, which is not mentioned as

needed so that the individual


learner in the 21st century can
learn how to learn?
a.

Ability to think

b.

Mathematical skills

c.

Memory skills

d.

Concentration

53. Which of the following


belongs to the more sophisticated
learning-to-learn skills for the
individual learner?
a.

To ask and gather data

b.

To listen and observe

c.
To process and select
information
d.

To read with understanding

54. Of the following effects on


learning, what is the effect of
simulations that make students feel
and sense experience in the
classroom?
a.

Reinforcing learning

b.
Providing experiences that
otherwise might not be had
c.

Motivating students

d.
Changing attitudes and
feelings

55. Of the following effects on


learning, what is the effect of
assigning various sections of the
newspaper, and allowing choice
depending on the learners choice?
a.

Encouraging participation

b.

Reinforcing learning

c.

Allowing different interests

d.
Changing attitudes and
feelings

58. Of comprehension or thinking


strategies, which is relating one or
two items, such as nouns and
verbs?
a.

Basic elaboration strategies

b.

Complex rehearsal strategies

c.
Complex elaboration
strategies
d.

Affective strategies

56. A young mother observes her


seven year old girl glued to her
computer games. What aspect of
the family life may suffer due to
obsession of the young with
technology gadgets?

59. Of skills teacher should


understand and students need to
acquire, which is the ability to
integrate complex information into
categories through its attributes
(characteristics, principles or
functions)?

a.

Family social life

a.

Scanning

b.

Family economic life

b.

Complex cognitive

c.

Discipline and obedience

c.

Sharpening-leveling

d.

Parent-child relationship

d.

Complexity-simplicity

57. Which of the following is not


an advanced process of metacognition among learners?
a.
Learning how to recognize
thoughts
b.

Acquisition of new knowledge

c.

Assessing own thinking

d.

Learning how to study

60. Inculcating moral maturity


among students, which of the
following relates to belief and
ideals?
a.

Promoting human equality

b.
Refraining from prejudiced
action

c.
Avoiding deception and
dishonesty
d.
Respecting freedom of
conscience
61. Research studies showed
that children in slums generally
have lower reading achievement
then children in urban schools.
What factor is shown to affect
reading achievement?

63. What is the main


organization and orientation of
science and social studies reading
materials?
a.

Expository

b.

Descriptive

c.

Narrative

d.

Argumentative

c.

Socio-economic status

64. In his History class, teacher


Naomi used a current events IQ
contest to determine champions in
identifying people, places, and
events. What learning objective
outcome does she aim to achieve?

d.

Listening comprehension

a.

Knowledge or recall

b.

Perpetual abilities

c.

Application

d.

Responding

a.

Mobility

b.
Personality and emotional
factors

62. When preacher Xian read the


Genesis story on creation, he
explained that God is so powerful
he created the universe in only
seven days. What level of reading
comprehension did preacher John
apply?
a.
Evaluative reading on
character, plot or style
b.

Literal reading the lines

c.
Applied reading beyond the
lines
d.
Interpretative reading
between the lines

65. In Eriksons stage theory of


development questionnaire, which
affirmation does not belong to the
stage of initiative vs. guilt?
a.

People can be trusted

b.

In difficulty, I will not give up

c.
I feel what happens to me is
the result of what I have done
d.

I am prepared to take a risk

66. For cognitive learning, what


are sets of facts, concepts, and
principles that describe underlying
mechanism that regulate human
learning, development and
behavior?
a.

Facts

b.

Concepts

c.

Theories

d.

Hypothesis

67. Literature teacher Kim


introduced figures of speech in
poetry to improve ability of her
students to interpret verses. What
kind of thinking is she developing
in her students?
a.

Critical thinking

b.

Metaphoric thinking

c.

Convergent thinking

d.

Divergent thinking

68. Of clusters of meaningful


learning activities, which does not
belong to spatial learning
activities?
a.

Visualization

b.

Concept-mapping

c.

Peer tutoring

d.

Art projects

69. From cluster of meaningful


learning activities, which does not
belong to verbal-linguistic
intelligence learning?
a.

Ecological field trip

b.

Debates

c.

Journal writing

d.

Reading

70. Which of the following


violates the principle that each
childs brain is unique and vastly
different from one another?
a.
Giving ample opportunity for
a pupil to explore rather than
simply dish out information
b.
Employing principles in
multiple intelligence in teaching
c.
Making a left-handed pupil
write with her right hand as this is
better
d.
Allowing open dialogue
among students of various cultural
backgrounds
71. Of the following which is
normally expected of Grade VI
pupils?
a.
Getting along with
classmates
b.

Being independent of parents

c.

Showing class leadership

a.

Reasoning

d.
Displaying a male or feminine
social role

b.

Imagination

c.

Application

d.

Motivation

72. From categories of


exceptionalities in the young child
and adolescents what involves
difficulties in specific cognitive
processes like perception,
language, memory due to mental
retardation, emotional/behavioral
disorder, or sensory impairment?
a.

Learning disabilities

b.
Speech and communication
disorders
c.

Emotional/conduct disorders

d.

Autism

73. Of the following, which is


most true of adolescents?
a.

Hormonal changes

b.

Last splurge of dependence

c.

Unruly behavior

d.

Defiance of peer group

74. Research says, people tend


to attribute successes to internal
causes and their failures to
external causes. What does this
imply as a most potent key to
success?

75. From Kohlbergs theory of


moral development, what is the
moral reasoning or perspective of
Mother Teresa who pledged her life
to serve the sick and very old?
a.

Social contract

b.

Universal principles

c.

Obedience

d.

Law and order

76. Blind cyclist and teacher


Maria Bunyan won 8th place in the
able-bodied Sydney 2000
Olympics. Of the following, which is
the central and fundamental
quality she displayed by never
thinking that blindness is an
impediment to becoming a great
athlete?
a.

Perseverance

b.

Passion

c.

Dedication

d.

Self-belief

77. How can new information be


made more meaningful to
students?
a.
Relating it to knowledge they
already know
b.

Valuing new knowledge

children with disabilities, ages 0 to


3 years old?
a.
Ensuring inclusion for special
children
b.
lag

Early growth development

c.
Demonstrating novelty of
new knowledge

c.
Identifying strengths and
weaknesses in special children

d.
Increasing retention of new
knowledge

d.
Preventing labeling of
disabled children

78. Under the domains of


learning, to what domain do Reflex
movements, perceptual abilities,
and non-discursive communication
belong?

81. What is the degree of moral


certitude of Jade Althea who
entered into marriage only out of
obedience to her parents, but
uncertain whether she wanted
marriage at all?

a.

Psychomotor

b.

Affective

c.
d.

Cognitive
Reflective

79. In what development stage is


the pre-school child?
a.

Early childhood

b.

Babyhood

c.

Infancy

d.

Late childhood

80. What is mainly addressed by


early intervention program for

a.

Certain

b.

Lax

c.

Probable

d.

Doubtful

82. On categories of
exceptionality in the young, what is
difficulty in focusing and
maintaining attention, and/or
recurrent hyperactive and
impulsive behavior?
a.

ADHD

b.

Emotional/conduct disorders

c.

Autism

d.
Speech and communication
disorders
83. What kinds of skills are
commonly dominant in subjects
like Computer, PE, Music, and the
like?
a.

Problem-solving skills

b.

Manipulative skills

c.

Affective skills

d.

Thinking skills

84. How is the disorderly


behavior of children classified when
they tell lies?
a.

Moral

b.

Intellectual

c.

Social

d.

Psychological

85. Which of the following is not


among the major targets of the
child-friendly school system
(CFSS)?
a.
All school children are
friendly
b.
All children complete their
elementary education within six
years

c.
All children 6-12 years old
are enrolled in elementary schools

religion (E.g. Christian, Buddhist,


Jewish, etc.)?

to a previously created cognitive


structure or schema?

d.
All grade six students pass
the division, regional, and national
tests

a.

Authoritarian values

a.

Assimilation

b.

Universal values

b.

Schema

c.

Creedal values

c.

Accommodation

d.

Sectarian values

d.

Equilibrium

86. Research studies that reading


power affects college students who
have insomnia, conflicts with
parents, poor rapport with other
people. What factor(s) is shown to
effect reading achievement?
a.

Home conditions

89. Among models of reading


strategies, what did student Jk
adopt when she reads back and
forth, attending to both what is in
her mind and whats on the page?

b.

Socio-economic status

a.

Bottoms-up

b.

Interactive

c.

Down-top

d.

Top-down

c.
Personality and emotional
factors
d.
Perception and
comprehension
87. Among the following, which is
the abstract form of learning,
parents teach their children?
a.
Tumulong ka sa paglinis ng
bahay
b.

Magbasa ka ng libro

c.

Palagi kang magdasal

d.

Mapakabuti ka

88. What characteristic


differentiate spiritual intelligence or
spiritual quotient as developed by
Harvard University, from sectarian

90. Of the following, how can


self-esteem be best developed
among learners?
a.
Doing fair share in
community work
b.

Fulfilling commitments

c.
Through relationships with
others
d.

Displaying self-control

91. Of Piagets Cognitive


Concepts, which refer to the
process of fitting a new experience

92. In Piagets stages of cognitive


development, which is the
tendency of the child to only see
his point of view and to assume
that everyone has the same point
of view?
a.

Reversibility

b.

Egocentrism

c.

Symbolic function

d.

Centration

93. Which is the most basic in


Maslows hierarchy of needs?
a.

Socialization

b.

Actualization

c.

Self-esteem

d.

Altruism

94. Which aspect of multiintelligence is enhanced by asking


students to work on a physical
model of the atom after a teachers

discussion on the subject of the


atom?
a.

Interpersonal

b.

Linguistic

c.

Kinesthetical

d.

Mathematical

95. Among specialist in reading,


who are mainly concerned about
reading as a thinking process that
involves the recognition of printed
or written symbols which serve as
thought stimuli?
a.

Semantics

b.

Psychologists

c.

Linguists

d.

Sociologists

96. Sequence the following


events on the historical
development of reading:
I.
Greek letters and the Roman
alphabet were developed
II.
Through the Semites
ingenuity, sounds, and symbols
gave rise to the Phoenician
alphabet
III.
People used pictures and
characters to convey messages

IV.
Researchers showed the
processes of reading,
comprehension, and interpretation
a.

I, II, III, and IV

b.

I, II, IV and III

c.

III, II, I and IV

d.

IV, II, I and III

97. How is the disorderly


behavior of children classified when
they dont focus and lack
attention?
a.

Intellectual

b.

Social

c.

Moral

d.

Psychomotor

98. How do you describe transfer


of learning across subject matter,
e.g value of thrift in Economic and
Social Science?
a.

Horizontal

b.

Spiral

c.

Vertical

d.

Cyclic

99. What broad learning is


needed for a learner to desire to
learn throughout life?
a.

Four basic Rs

b.

Basic education

c.

General education

d.

Pre-school system

100. What observation attests to


the fact that the sudden students
motivation vary according to sociocultural background?
a.
Females mature earlier than
boys
b.
Children from low-income
household meet more obstacle in
learning
c.
Genetic endowments may
show gifted endowments among
the young
d.
Brains of boys are bigger and
better than those of females
Assessment of Learning, Field
Study, Practice Teaching
101. Of the types of validity tests,
what is concerned with the relation
of test scores to performance at
some future time, e.g. Freshmen
college test can show success in
college?
a.

Curriculum validity

b.

Criterion validity

c.

Content validity

d.

Predictive validity

102. The test questions in Teacher


Dae Daes test were confusing and
subject to wrong understanding,
especially to poorer students. What
was wrong with the test?
a.
Inappropriate level of difficult
of items
b.

Unclear directions

c.

Ambiguity

d.
Test items inappropriate for
outcomes being measured
103. Of the following, which
exemplifies the best example of
cooperation and voluntarism in the
Parent-Teacher Associations?
a.
Helping hands after a natural
crisis, e.g. devastating storm
b.

Attending regular meetings

c.

Fund raising for PT funds

d.

Running the school canteen

104. Among standardized tests,


which reveals strengths and
weaknesses for purposes of
placement and formulating an
appropriate instructional program?
a.

Personality tests

b.

Achievement tests

c.

Diagnostic tests

c.

Science

d.

Competency tests

d.

Mathematics

105. Among standardized tests,


which can show how students
perform in comparison with each
other and to students in other
schools?
a.

Competency tests

b.

Subject exit tests

c.

Achievement tests

d.

Diagnostic tests

106. Teacher Bea Bunana makes


her tests easy for students to
understand, easy to administer and
score and suitable to test
conditions, e.g. time. What is she
achieving for her tests?
a.

Efficiency

b.

Usability

c.

Reliability

d.

Validity

107. Of the following subjects,


which does not belong to
performance-based subjects in
which direct instruction is
effectively used?
a.

Values education

b.

Music

108. Which of these approaches


would reform assessment
outcomes?
a.
Apply sanctions on low
performing schools
b.
Focus on testing without
investing the learners needs
c.
Use understanding as means
of giving feedback on students
learning
d.
Compare results of
performance of all schools
109. Using extrinsic motivational
assessment, what could be the
most noble motive in students
pursuing a lifetime work and
mission for the teaching
profession?
a.
Promise of high rank and
prestige
b.
Social service to upcoming
generations
c.
Economic security and
welfare
d.

Respected position in society

110. To what process of evaluation


does determining the extent
objectives are met belong?
a.

Authentic

b.

Formative

c.

Criterion-referenced

d.

Norm-referenced

of her students through


questioning in an open class?

d.
He got 60% of the items
correctly

a.

Oral questioning

b.

Performance test

116. Which of the following may


not be adequately assessed by a
paper and pencil test?

c.

Product rating scale

d.

Observation and self-report

111. Which form of the foundation


of all cognitive objects without
which the next level of higher
thinking skills cannot be attained?

114. In the context of the 6 facets


of understanding cited by Wiggins
and McTIghe, what is a proof of a
students understanding a
principle?

a.

Knowledge

a.

b.

Synthesis

c.

Application

b.
Repeating it as given by the
teacher

d.

Analysis

112. What primary response factor


is considered by Essay questions?
a.

Factual information

b.

Wide sampling of ideas

c.

Originality

d.
Less time for construction
and scoring
113. Among written categories of
assessment methods, what did
teacher Maggie Lagid use when
she assessed the stock knowledge

Stating given examples

c.
Applying it to solve his
problem
d.
Retaining it in memory for a
long period of time
115. What does it mean if student
Pete got a 60% percentile rank in
class?
a.
He scored better than 60% of
the class
b.
He scored less than 60% of
the class
c.
He got 40% of the test
wrongly

a.

Sight reading in music

b.

Multiplication skills

c.

Subject-verb agreement

d.

Vocabulary meaning

117. What should be done with


test item whose difficulty index is .
98?
a.

Revise it

b.

Retain it

c.

Reject it

d.
Reserve it for another group
of students
118. What is known as the scoring
guides for rating open-ended
questions?
a.

Rubrics

b.

Outcomes

c.

Scales

d.

Outputs

119. What does it mean to say


that the facility index of a test item
is .50?

a.

It is reliable

b.

It is valid

c.

It is moderate in difficulty

d.

It is very easy

120. With the mode of answering


as a point of reference, which of
the following does not belong to
this test group?
a.

Completion

b.

Essay

c.

Problem-solving

d.

Matching

121. One half of the class scored


very low. Teacher Janus gave
another tests to determine where
were the students were weakest.
What type of test is this?
a.

Aptitude test

b.

Remedial test

c.

Diagnostic test

d.

Readiness test

122. On what is normative


marking based?

c.
Normal curve of standard
distribution

126. What does a negative


discrimination index mean?

d.
Student achievement relative
to other students

a.
The test item has low
reliability

123. What cognitive domain is


involved in the students clarifying
information from conclusion?

b.
More from the lower group
answered the test item correctly

a.

Synthesis

b.

Evaluation

c.

Analysis

d.

Application

124. Which of the following


indicates a strong negative
correlation?
a.

-75

b.

-15

c.

-10

d.

-25

125. What is the graphic


illustration for the relationship
between two variables?
a.

Histogram

b.

Normal curves

a.

High marks of few students

c.

Frequency polygons

b.

Failure of some students

d.

Scatter diagram

c.
More from the upper
answered the test correctly
d.
The test could not
discriminate between the upper
and lower group
127. What is the deviation from a
standard or desired level of
performance?
a.

A problem

b.

A deficit

c.

A defect

d.

A gap

128. How does a students 80


percentile score interpreted?
a.
High in all the skills being
tested
b.
Higher than 80% of the
members of the group
c.
Better relative to the
competencies targeted
d.

80% of the specified content

129. Of the types of validity for


tests, what is focused on the extent
to which a particular tests
correlates with acceptable measure
of performance?
a.

Curricular validity

b.

Content validity

c.

Criterion validity

d.

Predictive validity

130. Among general categories of


assessment methods, what
instruments did pre-school teacher
Justine use when he rated the
handwriting of his students using a
prototype handwriting model?
a.

Product rating scale

b.

Performance test

c.

Written response instruments

d.

Observation and self-reports

131. On what should teachers


evaluation of a learners work be
based?

a.

I and II

b.

II, III, and IV

c.

II and III

d.

I, II, III, and IV

132. Self-evaluation can be done


in various ways, but this is not one
of them:
a.
Use of an evaluation
instrument

d.
How are the test scores to be
tabulated?

c.
Self-videotape of class
performance

a.
[Test scores = transmutation
table] x 100

d.

b.
[Highest score + Lowest
possible score] x 100

Per feedback session

133. In her test, Teacher Marian R


unknowingly gave clues to the
answers that reduce usability of
the test. What was wrong with the
test?
a.

Ambiguity

b.

Unclear directions

c.

Poorly constructed test items


Test too short

d.

ii.

Merit

134. In preparing classroom tests,


which of the following checklists is
the LAST among steps in tests
preparation?

Behavior in class

c.
How I have prepared a table
of specifications?

Written reflection

b.

Attendance

iv.

b.
How are the test results to be
reported?

135. What formula is used to total


and compute test scores at the end
of the year?

i.

iii.
Quality of academic
performance

a.
How are the objective items
to be scored?

c.

[Students score x 100]

d.
[Students score + Highest
possible score] x 100
136. What can be said of student
performance in a positively skewed
score distribution?
a.
A few students performed
excellently
b.

Most students performed well

c.
Almost all students had
average performance
d.
Most students performed
poorly

137. Which is true when the


standard deviation is small?
a.
Scores are toward both
extremes
b.

Scores are spread apart

c.
Scores are tightly bunched
together
d.

The bell curve is relatively fat

138. In her tests, Teacher Tomden


made tests that were either too
difficult or too easy. What was
wrong with her tests?
a.

Unclear directions

b.
Inappropriate level of
difficulty of the test items
c.

Ambiguity

d.
Identifiable patterns of
answers
139. What is an alternative
assessment tool that consists of a
collection of work artifacts or in
progress accomplishment by a
targeted clientele?
a.

Evaluation instrument

b.

Rubric

c.

Achievement test

d.

Portfolio

140. What computation did


teacher Panny use in getting the
difference between the highest and
lowest scores in each class?
a.

Mean

b.

Range

c.

Standard deviation

d.

Median

141. Which measure of central


tendency is most reliable when
scores are extremely high and low?
a.
Cannot be identified unless
individual scores are given
b.

Median

c.

Mode

d.

Mean

142. Which measure of central


tendency is most reliable to get a
picture of the class performance
whose raw scores in a quiz are: 97,
95, 85, 86, 77, 75, 50, 10, 5, 2, 1?

143. Self-evaluation has become


an important kind of performance
assessment among teachers,
useful as an honest self-criticism
and a starting point to removal
evaluation by supervisors, peers, or
students. How is self-evaluation
described?
a.
Evidence of teaching
performance
b.
Substitute to supervisors
rating
c.

Guide for self-adjustment

d.

Tool for salary adjustment

144. What is the common


instrument used in measuring
learning in the affective domain?
a.

Multiple choice

b.

Checklist

c.

Scaling

d.

Questionnaire

b.
None. It is best to look at
individual scores

145. On the test givers list of


Dos, which of the following is not
relative to motivating students to
do their best?

c.

Mean

a.

d.

Median

b.
Reduce test anxiety, e.g.
Take a deep breath.

a.

Mode

Read test directions

c.
test

Explain the purpose of the

d.
Tell students: I will be proud
of you if you perform well.
146. What is the range if the score
distribution is: 98, 93, 93, 93, 90,
88, 87, 85, 85 , 85, 70, 51, 34, 34,
34, 20, 18, 51, 12, 9, 8, 6, 3, 1?
a.

93

b.

85

c.

97

d.

Between 51 and 34

147. What does the test mean if


the difficulty index is 1?
a.

Very difficult

b.

Missed by everyone

c.

Very easy

d.

A quality item

148. What is the meaning of a


negative correlation between
amount of practice and number of
errors in tennis?
a.
The increase in the amount
of practices does not at all affect
the number of errors
b.
As the amount of practice
increases, the number of errors
decreases

c.
The decrease in the amount
of practice sometimes affects the
number of errors
d.
Decrease in the amount of
practice goes with decrease in the
number of errors

teachers and become an obstacle


for adapting well to the school
environment. What should be the
policy for assigning said physical
facilities?
a.

needs of students basis

149. An entering college would


like to determine which course is
best suited for him. Which test is
appropriate for this purpose?

b.

position ranking basis

c.

first-come, first-served basis

d.

service seniority basis

a.

Aptitude test

b.

Intelligence test

c.

Achievement test

d.

Diagnostic test

152. There are various functions a


fellow teacher or peer coach can
help new teachers. What role does
a peer coach play by being
present/available to share ideas,
problems and success with a new
teacher?

150. Which of the following


criteria is the basis for selecting
tests that yield similar results when
repeated over a period of time?
a.

Efficiency

b.

Validity

c.

Usability

d.

Reliability

Principles and Methods of Teaching,


Educational Technology, Curriculum
Development
151. Facilities such as classrooms,
fixtures, and equipment can often
damage the morale of new

a.
a provider of technical
feedback
b.

a facilitator of strategies

c.

an analyzer of teaching job

d.

a close peer or companion

153. Teacher Princess sees to it


that her classroom is clean and
orderly so her pupils will less likely
disarrange seats and litter on the
floor. On which thought is her
action based?
a.

existentialism

b.

progressivism

c.

behaviorism

d.

reconstructionism

154. Teacher Nancy is directed to


pass an undeserving student with a
death threat. Which advise will a
utilitarian give?
a.
Dont pass him. You surely
will not like someone to give you a
death threat in order to pass
b.
Pass the student. That will be
off use to the student, his parents
and you.
c.
Pass the student. Why suffer
the threat?
d.
Dont pass him. Live by your
principle of justice. You will get
reward, if not in this life, in the
next!
155. In what setting is
differentiated and multi-lingual
teaching most effective?
a.

special children with classes

b.

multi-grade classes

c.
children with diverse cultural
backgrounds
d.

pre-school children

156. After the embarrassing


incident, Teacher Kevin vowed to
himself to flunk the student at the

end of the school term. What has


Dante done that is against the
guidelines for using punishment?
a.
Punishing immediately in an
emotional state
b.
Using double standards in
punishing
c.

Doing the impossible

d.
Holding a grudge and not
starting with a clean slate
157. Following the principles for
punishing students, which of the
following is the LEAST desirable
strategy for classroom
management?
a.
Punishing while clarifying
why punishment is done
b.

Punishing while angry

c.
Punishing the erring student
rather than the entire class
d.

Give punishment sparingly

158. According to the guidelines


on punishment, what does it mean
that the teacher should give the
student the benefit of the doubt?
a.
Make sure facts are right
before punishing
b.
Doubt the incident really
happened

c.
Dont punish and doubt
effectiveness of punishment
d.
Get the side of the students
when punishing
159. Which of the following
guidelines for punishment may be
done?
a.
Dont punish students
outside of school rules on
punishment
b.

Dont threaten the impossible

c.
Dont use double standards
for punishing
d.

Dont assign extra homework

160. For group guidance in


classroom management, what
element is lacking when there is
too much competitiveness and
exclusiveness with the teacher
being punitive and partial to some
students?
a.
Dissatisfaction with
classroom work
b.

Poor interpersonal relations

c.

Poor group organization

d.

Disturbance in group climate

161. To demonstrate here


authority Teacher Kokeyni made an
appeal to undisciplined students.

What kind of appeal did she make


by saying, Ladies and gentlemen,
dont engage in that kind of
behavior, you can do much
better?
a.

Invoke peer reaction

b.

Exert authority

c.
Internalizing students image
of themselves
d.

Teacher-student relationship

162. What is the term for the leap


from theory to practice in which
the teacher applies theories to
effective teaching methods and
theories?
a.

Integration process

b.

Informational process

c.

Conceptualization process

d.

Construction process

163. Of subcategories of
movement behavior, what is
happening when the teacher ends
an activity abruptly?
a.

Thrust

b.

Truncation

c.

Stimulus-bounded

d.

Flip-flop

164. Of subcategories of teacher


movement behavior, what is
happening when the teacher goes
from topic or activity to other topic
or activities, lacking clear direction
and sequence of activities?
a.

Truncation

b.

Dangle

c.

Thrust

d.

Flip-flop

165. Of subcategories of teacher


movement behavior, what is
happening when the teacher is too
immersed in a small group of
students or activity, thus ignoring
other students or activity?

d.
Shaming erring student
before the class
167. Among mistaken goals in the
Acceptance Approach to discipline,
what happens when students defy
adult by arguing, contradicting,
teasing, temper tantrums, and low
level hostile behavior?
a.

Power seeking

b.

Withdrawal

c.

Revenge seeking

d.

Attention getting

a.

Truncation

168. Teacher Ann Patuan dealt


effectively with a minor infraction
of whispering by a student to a
neighbor during class. Which of the
following did she do?

b.

Flip-flop

a.

c.

Stimulus-bounded

d.

Thrust

b.
Continue to teach and ignore
infraction

166. From classroom management


strategies applied on erring
students, which of the following
should not be done?
a.

Surprise quiz

b.
Communicating problems to
parents
c.

Parent-principal conference

Reprimand quietly

c.
Reprimand student after
class
d.
Use nonverbal signals
(gesture or facial expression)
169. What mistake is teacher
Senemin Basic trying to avoid by
never ignoring any student or
group of students in her
discussions and other activities?

a.

Non-direction

b.

Dangled activity

c.

Divided attention

d.

Abrupt end

170. Teacher Dra D Explorer is a


great lecturer and so she is invited
to speak and represent the school
on many occasions. What is one
quality of her lecturers when she
follows a planned sequence, not
diverting so as to lose attention of
her listeners?

content. What quality of content


did he achieve when she made
certain her information came with
the information explosion which
she got in the Internet, such as
how to effectively teach phonetics?
a.

Learnability

b.

Significance

c.

Balance

d.

Interest

a.

Explicit explanations

b.

Continuity

c.

Inclusion of elements

173. Teacher Kevin made certain


his lesson content can be useful to
his students, taking care of their
needs in a student-centered
classroom. What is this kind of
quality content?

d.

Fluency

a.

Utility

b.

Balance

c.

Self-sufficiency

d.

Interest

171. Teacher Aldub makes certain


content interesting to his students.
Focusing on learners, he also uses
many simple examples, metaphors
and stories. What is this quality of
lesson content?
a.

Interest

b.

Feasibility

c.

Self-sufficiency

d.

Balance

172. Teaching English, teacher


Krizzy is careful about her lesson

174. In the implementation of the


curriculum at the classroom level,
effective strategies are called
Green. Which of the following
belongs to the Green Flag?
a.
Homogenous students
grouping
b.
Content delivery based on
lessons

c.

Excess in chalkboard talk

d.
Student interest and teacher
enthusiasm
e.

Rigidity if movement

175. In the implementation of the


curriculum at the classroom level,
ineffective strategies are called
Red. Which of the following
belongs to the Red Flag?
a.
Content applied to real-life
situations
b.
Overemphasis on drill and
practice
c.
Available enrichment
activities
d.
Integration of problem
solving
176. Teacher Maggie explains by
spicing her lectures with examples,
descriptions and stories. What is
this quality in her lectures?
a.

Planned sequence

b.

Elaboration through elements

c.

Use of audiovisuals

d.

Simple vocabulary

177. Can technology take the


place of the teacher in the

classroom? Select the most


appropriate answer:
a.
No. It is only an instrument or
a tool
b.
Yes, when they hire less
teachers and acquire more
computers
c.
Yes. When teachers are not
competent
d.
Yes, such as in the case of
Computer-assisted instruction (not
teacher-assisted instruction)
178. What kind of tool is
technology as evidenced by its use
in word processing databases,
spreadsheets, graphics design and
desktop publishing?
a.

Analyzing tool

b.

Encoding tool

c.

Productivity tool

d.

Calculating tool

179. In avoiding implying sickness


or suffering, which of the following
is the most preferable way to refer
to those with disabilities like polio?
a.

Is polio-stricken

b.

Had polio

c.

Polio victim

d.

Suffers from polio

180. If threat of punishment is


necessary on erring students, how
should this best be done?
a.
Make the threat and reinforce
with warning
b.
Make the threat with
immediate punishment
c.
Ward and threat at the same
time
d.
First a warning before the
threat
181. Among cognitive objectives,
what is also known as an
understanding and is a step higher
than more knowledge of facts?

c.
Causal and logical
relationships
d.

Continuing sequence

183. In determining the materials


and media to use, what
consideration did Teacher Ina A.
Mag adopt when he chose
materials that can arouse and
sustain in curiosity?
a.

Satisfaction

b.

Interest

c.

Expectancy

d.

Relevance

184. Which of the following is true


of a democratic classroom?
a.
Teacher acts as firm decision
maker

a.

Comprehension

b.

Analysis

c.

Synthesis

b.
Students decide what and
how to learn

d.

Application

c.

182. What is the quality of teacher


Pining Garcias lecture when she
makes use of various pictures,
charts, graphs, videos to support
her lectures?
a.

Simplified vocabulary

b.
Enrichment through visual
aids

Consultation and dialogue

d.
Suggestions are sent to
higher officials for decisions
185. This is appropriate use of
technology which can unite people
of the world rather than exploit
them?
a.

For pornography

b.

For social media

c.

For financial fraud

d.

For propaganda

186. From structures in


Multifunctional Cooperative
Learning, which involves each
student writing in turn one answer
as a paper and pencil is passed
around the group?
a.

Jigsaw

b.

Inside-outside circle

c.

Roundtable

d.

Partners

187. How does the humaneness


of the teacher best described when
he/she is full interest and
enthusiasm in the work of
teaching?
a.

Responsiveness

b.

Perceptiveness

c.

Knowledge

d.

Sensitivity

188. Teacher Lester Cruz Valdez


gets more information about how
his students learn in order to
upgrade his pedagogy. What
principle is he following?
a.
Teachers should keep track of
learning outcomes

b.
Teachers should value
information

I.
Provide motivation and draw
commitments

c.
Teachers should document
information data on students

II.
Explain rationale and
objectives

d.
Teachers should teach and
test learning

III.

Provide feedback

IV.

Practice for mastery

a.

II, I, IV, and III

b.

IV, I, II and III

c.

I, IV, III, and II

d.

I, II, IV and III

189. In order to assist new


teacher, which is the most effective
way to clarify the schools goals
and responsibilities early in the first
year?
a.

Students handbook

b.

Orientation

c.

Principals memorandum

d.

School curriculum

190. Of components of direct


instruction, which involves teachers
and students working together on a
skill or task and figuring out how to
apply the strategy?

192. Teacher JanJan made certain


his lesson content is within the
capacity of his young forum grade
learners. What is the quality of
Johns lesson content when he fits
lesson to learners capacity to
absorb lesson content?
a.

Learnability

b.

Balance

a.

Consolidation

c.

Validity

b.

Guided practice

d.

Interest

c.

Application

d.

Modeling

193. From structures of


Multifunctional Cooperative
Learning, which makes each group
to produce a group product to
share with the whole class?

191. In direct/expositive
instruction, what is the logical
pattern of procedures in a lesson
adopted?

a.

Coop-coop

b.

Think-pair-share

c.

Team Word-Webbing

d.

Partners

194. This is the more appropriate


understanding of technology in
education?

196. In the guided exploratory


approach to learning, which is not
the term used for Inquiry learning?
a.

Heuristic learning

b.

Problem-solving learning

c.

Discovery learning
Expository learning

a.

Methods and process

d.

b.

Inventions and equipment

c.

Channels and instruments

197. What is another quality of


teacher Lassie Pecsons lectures
when she used words that are
within the grasp of her listeners,
avoiding technical terms and
jargons?

d.
Hardware, designs, and
environment
195. A teacher introduces herself
as teacher only. What does this
imply?
a.
She must have been forced
to pursue a career in teaching.
b.
The teaching profession is
not a very significant one
c.
The teaching profession is
the lowest paid profession
d.
She takes no pride in the
teaching profession

a.
Use of specific descriptions
and examples
b.

Enriched audiovisuals

c.

Normal vocabulary

d.

Planned sequence

198. In delivering her lessons,


teacher Blackie Lou Blanco is
careful that no topic is extensively
discussed at the expense of other
topics. That guiding principle in
selection and organization of
lesson content is she following?

a.

Significance

b.

Self-sufficiency

c.

Feasibility

d.

Balance

199. In determining materials and


media to use, what consideration
did Teacher Grachie adopt when
she gave importance to the level of
outcome and the learners sense of
fulfillment in performing the task?
a.

Expectancy

b.

Satisfaction

c.

Interest

d.

Relevance

200. In the inductive approach to


learning, what is not among the
facilitating skills needed on the
part of the teacher?
a.
Teacher giving generalization
of principles
b.
Commenting to pave way for
generalizations or principles
c.

Organizing answers

d.

Asking the right questions

FINAL REVIEW

1. A guest speaker in one


graduation rites told his audience:
"Reminder, you are what you
choose to be." The guest speaker is
more of a/an
.
a.

Realist b. Pragmatist

c.

Idealist d. Essentialist

2. A teacher who equates


authority with power does NOT
.
a. Shame b. develop self-respect
in every pupil
c.

retaliate

d. intimidate

3. Based on Edgar Dale's Cone of


Experience, which activity is
farthest from the real thing?
a. Read
images

b. Hear
c. View
d. Attend exhibit

4. Based on Piaget's theory, what


should a teacher provide for

children in the concrete operational


stage?

A. Physics and Math b. English


c. Physics
d. Math

a. Activities for hypothesis


formulation.

6. Who among the following puts


more emphasis on core
requirements, longer school day,
longer academic year and more
challenging textbooks

b. Learning activities that involve


problems of classification and
ordering.
c. Games and other physical
activities to develop motor skills.
d. Stimulating environment with
ample objects to play with.
5. Study this group of tests
which was administered with the
following results, then answer the
question.
Ronnels

A. Perennialist
Essentialist
C. Progressivist

B.
D. Existentialist

7. I drew learners into several


content areas and encouraged
them to solve a complex question
for Inter-disciplinary teaching.
Which strategy did I use?
A. Problem-centered learning

Subject
Score

Mean SD

Math

56

10

43

C. Reading-writing activity

Physics

41

31

D. Thematic instruction

English

80

16

109

8. The concepts of trust vs.


maturity, autonomy vs. self-doubt,
and initiative vs. guilt are most
closely related with the works of

B. Unit method

In which subject(s) did Ronnel


perform best in relation to the
group's performance?

A. Erikson

B. Piaget
D. Jung

C. Freud

9. I want to teach concepts,


patterns and abstractions. Which
method is most appropriate?
A. Indirect instruction
Discovery

B.

C. Direct instruction
solving

D. Problem

12. Principal B tells her teachers


that training in the humanities is
most important. To which
educational
philosophy does he adhere?
A. Existentialism B. Perennialism

10. Which behavioral term


describes a lesson outcome in the
highest level of Bloom's cognitive
domain?
A. Create B. Evaluate C. Analyze
D. Design
11. A sixth grade twelve-year old
boy comes from a dysfunctional
family and has been abused and
neglected. He has been to two
orphanages and three different
elementary schools. The student
can decide on the second grade
level, but he can comprehend
orally material at the fourth or fifth
grade level. The most probable
cause/s of this student's reading
problem is/are.

C. Progressivism D. Essentialism
13. Which is the first step in
planning an achievement test?
A. Define the instructional
objective.
B. Decide on the length of the test.
C. Select the type of test items to
use.

writes an anonymous letter against


Teacher B accusing her of
fabricated lies Teacher Q mails this
anonymous letter to the Schools
Division Superintendent. What
should Teacher Q do if she
has to act professionally?
A. Submit a signed justifiable
criticism against Teacher B, if there
is any.
B. Go straight to the Schools
Division Superintendent and gives
criticism verbally.
C. Hire a group to distribute poison
letters against Teacher B for
information dissemination.

D. Build a table of specification.

D. Instigate student activists to


read poison letters over the
microphone.

14. In the Preamble of the Code of


Ethics of Professional Teachers,
which is NOT said of teachers?

16. As a teacher, what do you do


when you engage yourself in major
task analysis?

A. LET passers

A. Test if learning reached higher


level thinking skills.

B. Duly licensed professionals


C. Possess dignity and reputation

A. emotional factors
teaching

B. poor

D. With high-moral values as well


as technical and professional
competence

C. neurological factor
Immaturity

D.

15. Teacher Q does not want


Teacher B to be promoted and so

B. Breakdown a complex task into


sub-skills.
C. Determine the level of thinking
involve
D. Revise lesson objectives

17. Which is a sound classroom


management practice?

D. Use the objectives for the units


as guide in your test construction.

A. Avoid establishing routines

20. Teacher M's pupils are quite


weak academically and his lesson
is already far behind his time table.

B. Establish routines for all daily


needs and tasks.
C. Apply rules and policies on a
case to case basis.
D. Apply reactive approach to
discipline.
18. What does extreme
authoritarianism in the home
reinforce in learners?
A. Doing things on their own
initiative
B. Ability to direct themselves.
C. Dependence on others for
direction.
D. Creativity in work.
19. In a criterion-referenced
testing, what must you do to
ensure that your test is fair?
A. Make all of the questions true or
false.
B. Ask each student to contribute
one question.
C. Make twenty questions but ask
the students to answer only ten of
their choice.

How should Teacher M proceed with


his lesson?
A. Experientially B. Inductively
C. Logically

D. Deductively

21. Soc exhibits fear response to


freely roaming dogs but does not
show fear when a dog is on a leash
or confined to a pen. Which
conditioning process is illustrated
A. Generalization B. Extinction
C. Acquisition
Discrimination

D.

22. Teacher P wants to develop the


skill of synthesizing in her pupils.
Which one will she do?
A. Ask her students to formulate a
generalization from the data shown
in graphs.

D. Directs her students to ask


questions on the parts of the lesson
not understood.
23. Studies in the areas of
neurosciences disclosed that the
human brain has limitless capacity.
What does this imply?
A. Some pupils are admittedly not
capable of learning.
B. Every pupil has his own native
ability and his learning is limited to
this native ability.
C. Every child is a potential genius.
D. Pupils can possibly reach a point
where they have learned
everything.
24. What should you do if a parent
who is concerned about a grade his
child received compared to another
student's grade, demands to see
both students' grades?
A. Refuse to show either record.
B. Show both records to him.

B. Ask her students to answer


questions beginning with What if ...

C. Refuse to show any record


without expressing permission from
principal.

C. Tell her pupils to state data


presented in graphs.

D. Show only his child's records.


25. Rights and duties are
correlative. This means that
.

A. rights and duties regulate the


relationship of men in society

reward, if not in this life, in the


next!

covert responses. Which one did


she NOT do?

B. rights and duties arise from


natural law

D. Pass the student. That will be of


use to the student, his parents and
you.

A. She had the students write their


response privately.

C. each right carries with it one or


several corresponding duties
D. rights and duties ultimately
come from God
26. If you agree with Rizal on how
you can contribute to our nation's
redemption, which should you work
for?
A. Opening our doors to foreign
influence
B. Upgrading the quality of the
Filipino through education
C. Stabilizing the political situation
D. Gaining economic recovery
27. Teacher A is directed to pass an
undeserving student with a death
threat. Which advice will a hedonist
give?
A. Pass the student. Why suffer the
threat?
B. Don't pass him. You surely will
not like someone to give you a
death threat in order to pass.
C. Don't pass him. Live by your
principle of justice. You will get

28. Are percentile ranks the same


as percentage correct?
A. It cannot be determined unless
scores are given.
B. It cannot be determined unless
the number of examinees is given.
C. No

D. Yes

29. Teacher W wants to review and


check on the lesson of the previous
day? Which one will be most
reliable?
A. Having students identify difficult
homework problems.
B. Having students correct each
other's work.
C. Sampling the understanding of a
few students.
D. Explicitly reviewing the taskrelevant information necessary for
the day's lesson.

30. To elicit more students


response, Teacher G made use of

B. She showed the correct


answers on the overhead after
the students have written their
responses.
C. She had the students write their
responses privately then called
each of them.
D. She refrained from judging on
the student's responses.
31. Which is a form of direct
instruction?
A. Discovery process
Problem solving

B.

C. Programmed instruction
Inductive reasoning

D.

32. Which test has broad sampling


of topics as strength?
A. Objective test
answer test

B. Short

C. Essay test
type

D. Problem

33. Teacher B uses the direct


instruction strategy. Which
sequence of steps will she follow?
I. Independent practice

II. Feedback and correctiveness


III. Guided student practice
IV. Presenting and structuring
V. Reviewing the previous day's
work
A. V-II-IV-III-I
V

B. III-II-IV-I-

C. V-lV-III-II-I
IV

D. I-V-II-III-

35. Which illustrates a


developmental approach in
guidance and counseling?
A. Spotting on students in need of
guidance
B. Teaching students how to
interact in a positive manner

D. guesses, data and conclusions


38. Which group of philosophers
maintains that truth exists in an
objective order that is independent
of the knower?
A. Idealist
C. Existentialists

B. Pragmatists
D. Realist

C. Acting as a mediator

34. Teacher H and Teacher I are


rivals for promotion. To gain the
favor of the promotional staff,
Teacher I offers her beach resort for
free for members of the
promotional staff before the
ranking. As one of the contenders
for promotion, is this becoming of
her to do?
A. Yes. This will be professional
growth for the promotional staff.
B. No. This may exert undue
influence ori the members of the
promotional staff and so may fail to
promote someone on the basis of
merit.
C. Yes. The rare invitation will
certainly be welcomed by an
overworked promotional staff.
D. Yes. There's nothing wrong with
sharing one's blessings.

D. Making the decision for the


confused student
36. The test item "Group the
following items according to shape"
is a thought test item on
.
A. creating
C. generalizing
comparing

B. classifying
D.

37. Direct instruction is for facts,


rules, and actions as indirect
instruction is for
,
,
,.
A. hypotheses, verified data and
conclusions
B. concepts, patterns and
abstractions
C. concepts, processes and
generalizations

39. On whose philosophy was A. S.


Neil's Summerhill, one of the most
experimental schools, based?
A. Rousseau
Pestalozzi

B.

C. Montessori
Locke

D. John

40. Teacher H strives to draw


participation of every student into
her classroom discussion. Which
student's need is she trying to
address? The need
A. to show their oral abilities to the
rest of the class
B. to be creative
C. to feel significant and be part of
a group
D. to get everything out in the
open

41. Out of 3 distracters in a


multiple choice test item, namely
B, C, and D, no pupil chose D as
answer.
This implies that D is
A. an ineffective distracter
vague distracter

B. a

C. an effective distracter
D. a plausible distracter
42. Which describes normreferenced grading?
A. The performance of the group
B. What constitutes a perfect score
C. The students' past performance
D. An absolute standard
43. Which Filipino trait works
against the shift in teacher's
role from teacher as a fountain
of information to teacher as
facilitator?
A. Authoritativeness
Authoritarianism

B.

C. Hiya
Pakikisama

D.

A. Test
Appraise

B. Assess
D. Theorize

C.

45. From whom do we owe the


theory of deductive interference as
illustrated in syllogisms?
A. Plato
Aristotle

B. Socrates
C.
D. Pythagoras

46. To come closer to the truth we


need to go back to the things
themselves. This is the advice of
the
A. behaviorists
phenomenologists
C. idealists

B.

D. pragmatists

47. Which is NOT a sound purpose


for asking questions?
A. To probe deeper after an answer
is given.
B. To discipline a bully in class.
C. To remind students of a
procedure.
D. To encourage self-reflection.

44. Teacher G's lesson objective


has something to do with the skill
of synthesizing? Which behavioral
term is most appropriate?

48. Your teacher is of the opinion


that the world and everything in it
are ever changing and so teaches
you the skill to cope with change.
What is his governing philosophy?
A. Idealism

B. Existentalism

C. Experimentalism

D. Realism

49. Research tells that teachers ask


mostly content questions. Which of
the following terms does NOT
refer to content question?
A. Closed
Concept

B. Direct
C.
D. Convergent

50. Whose teaching is in support of


Education for All (EFA), he asserted
that in teaching there should be no
distinction of social classes.
A. Sun Yat Sen
Confucius
C. Mencius

B.
D. Lao Tsu

51. As a teacher, you are a


Reconstructionist. Which among
these will be your guiding
principle?
A. I must teach the child every
knowledge, skill, and value that he
needs for a better future.
B. I must teach the child to develop
his mental powers to the full.
C. I must teach the child so he is
assured of heaven.
D. I must teach the child that we
can never have real knowledge of
anything.

52. If a teacher plans a


constructivist lesson, what will he
most likely do? Plan how he can
A. do evaluate his students' work
B. do reciprocal teaching
C. lecture to his students
D. engage his students in
convergent thinking
53. Based on Piaget's theory, what
should a teacher provide for
children in the sensorimotor stage?
A. Games and other physical
activities to develop motor skill.
B. Learning activities that involve
problems of classification and
ordering.
C. Activities for hypothesis
formulation.
D. Stimulating environment with
ample objects to play with.
54. Teacher H gave her first-grade
class a page with a story in which
pictures take the place of some
words. Which method did she use?
A. The whole language approach
B. The Spaulding method
C. The rebus method

D. The language experience


approach
55. A stitch on time saves nine, so
goes the adage.. Applied to
classroom management, this
means that we
A. may not occupy ourselves with
disruptions which are worth
ignoring because they are minor
B. must be reactive in our approach
to discipline
C. have to Resolve minor
disruptions before they are out of
control
D. may apply 9 rules out of 10
consistently
56. The attention to the
development of a deep respect and
affection for our rich cultural past is
an influence of
A. Confucius
Hegel

B.

C. Teilhard de Chardin
Dewey

D.

57. In what way can teachers


uphold the highest possible
standards of quality education?

A. By continually improving
themselves personally and
professionally
B. By wearing expensive clothes to
change people's poor perception of
teachers
C. By working out undeserved
promotions
D. By putting down other
professions to lift the status of
teaching
58. Student B claims: I cannot see
perfection but I long for it. So it
must be real.Under which group
can he be classified?
A. Idealist

B. Empiricist

C. Realist

D. Pragmatist

59. In Krathwohl's taxonomy of


objectives in the affective, which is
most authentic?
A. Characterization
Organization

B.

C. Responding
Valuing

D.

60. NSAT and NEAT results are


interpreted against set mastery
level. This means that NSAT and
NEAT
fall under

A. intelligence test
aptitude test

B.

C. criterion-referenced test
norm-referenced test

D.

61. Each teacher is said to be a


trustee of the cultural and
educational heritage of the nation
and is, under obligation to transmit
to learners such heritage. Which
practice makes him fulfill such
obligation?
A. Use the latest instructional
technology.
B. Observe continuing professional
education.

63. In which competency do my


students find the greatest
difficulty? In the item with a
difficulty index of
A. 0.1

B. 0.9

C. 0.5

A. Socratic method
Cooperative learning

B.

C. Mastery learning
Indirect instruction

D.

A. Buddha
Confucius

65. Why should a teacher NOT use


direct instruction all the time?

D. Study the life of Filipino heroes.

B. It requires use of many


supplementary materials.

B. lt brings together the


information that has been
discussed
C. it makes provisions for full
participation of students.
D. it clinches the basic ideas or
concepts of the lesson.

B.

67. Whose influence is the


education program that puts
emphasis on self-development.
through the classics, music, and
rituals?

64. Which method has been proven


to be effective in courses that
stress acquisition of knowledge?

A. It requires much time.

A. it links the parts of the lesson

A. Perennialism
Essentialism

C. Existentialism D. Progressivism

D. 1.0

C. Use interactive teaching


strategies.

62. A teacher's summary of a


lesson serves the following
functions, EXCEPT

From which philosophy is this


thought based?

C. It is generally effective only in


the teaching of concepts and
abstractions.
D. It reduces students
engagement in learning.
66. Principal C shares this thought
with his teachers: Subject matter
should help students understand
and appreciate themselves as
unique individuals who accept
complete responsibility for their
thoughts, feelings, and actions.

B. Mohammed
D. Lao Tsu

C.

68. Teacher A is a teacher of


English as a Second Language. She
uses vocabulary cards, fill-in-theblank sentences, dialogues,
dictation and writing excercises in
teaching a lesson about grocery
shopping. Based on this
information, which of the following
is a valid conclusion?
A. The teacher is applying Bloom's
hierachy of cognitive learning.
B. The teacher is teaching in a
variety of ways because not all
students learn in the same manner.
C. The teacher wants to make
herteachirig easier by having less
talk.
D. The teacher is emphasizing
reading and writing skills.

69. Which one may support


equitable access but may sacrifice
quality?

C. Both students are functioning in


the average range of intellectual
ability

75. Which criterion should guide a


teacher in the choice of
instructional devices?

A. Open admission
School accreditation

D. Another IQ test should be given


to truly assess their intellectual
potential

A. Attractiveness

B.

C. Deregulated tuition fee hike


D. Selective retention
70. Teacher A discovered that his
pupils are very good in
dramatizing. Which tool must have
helped him discover his pupils'
strength?

A. Portfolio assessment
B. Performance test
C. Journal entry
D. Paper-and-pencil test
71. Two students are given the
WISE II. One has a full scale IQ of
91, while the other has an IQ of
109.
Which conclusion can be drawn?
A. The second student has
significantly higher intellectual
ability
B. The first student is probably
below average, while the second
has above average potential

72. Helping in the development of


graduates who a maka-Diyos is an
influence of
A. naturalistic morality B. classical
Christian morality
C. situational morality D.
dialectical morality
73. Teacher B clears his throat to
communicate disapproval of a
student's behavior. Which specific
influence technique is this?
A. Signal interference
Direct appeal

B.

C. Interest boosting
Proximity control

D.

74. In the context on the theory on


multiple intelligences, what is one
weakness of the paper-pencil test?
A. It is not easy to administer.
B. It puts the non-linguistically
intelligent at a disadvantage.
C. It utilizes so much time.
D. It lacks reability.

B. Cost

C. Novelty
D.
Appropriateness
76. What is most likely to happen
to our economy when export
continuously surpasses import is a
thought question on
.
A. creating
and-effect
C. synthesizing
predicting

B. relating causeD.

77. The teacher's first task in the


selection of media in teaching is to
determine the
.
A. choice of the students
availability of the media

B.

C. objectives of the lesson


technique to be used

D.

78. All men are pretty much alike.


It is only by custom that they are
set apart, said one Oriental
philosopher. Where can this
thought be most inspiring?
A. In a multi-cultural group of
learners

B. In multi-cultural and
heterogeneous groups of learners
and indigenous peoples' group
C. In a class composed of
indigenous peoples
79. If teacher has to ask more
higher-order questions, he has to
ask more
questions.
A. closed

B. fact
C. concept
D. convergent

80. Student Z does not study at all


but when the Licensure
Examination for Teachers (LET)
comes, before he takes the LET, he
spends one hour or more praying
for a miracle, i.e. to pass the exam.
Which attitude towards religion or
God is displayed?
A. Religion as fake
B. Religion as magic
C. Religion as authentic
D. Religion as real
81. Teacher T taught a lesson
denoting ownership by means of
possessives. He first introduced the
rule, then gave examples, followed
by class exercises, then back to the
rule before he moved into the
second rule. Which presenting
technique did he use?

A. Combinatorial B. Comparative
C. Part-whole
Sequence

D.

82. All subjects in Philippine


elementary and secondary schools
are expected to be taught using
the integrated approach. This came
about as a result of the
implementation of
.
A. Program for Decentralized
Education
B. School-Based Management
C. Basic Education Curriculum
D. Schools First Initiative
83. Which technique should a
teacher use to encourage response
if his students do not respond to his
question?
A. Ask a specific student to
respond, state the question, and
wait a response.
B. Tell the class that it will have
detention unless answers are
forthcoming.
C. Ask another question, an easier
one.
D. Wait for a response.
84. Standard deviation is to
variability as mean is to

A. coefficient of correlation
central tendency

B.

C. discrimination index
level of difficulty

D.

85. Quiz is to formative test while


periodic is to
A. criterion-reference test
B. summative test
C. norm-reference test
D. diagnostic test
86. Which teaching activity is
founded on Bandura's Social
Learning Theory?
A. Lecturing

B. Modeling

C. Questioning
Inductive Reasoning

D.

87. In mastery learning, the


definition of an acceptable
standard of performance is called a
A. SMART

B. criterion measure

C. behavior D. condition
88. Students' scores on a test were:
72, 72, 73, 74, 76, 78, 81, 83, 85.
The score 76 is the
.
A. mode
B. average
mean
D. median

C.

89. Test norms are established in


order to have a basis for
.
A. establishing learning goals

92. A student passes a research


report poorly written but ornately
presented in a folder to make up
for the poor quality of the book
report content. Which Filipino trait
does this practice prove? Emphasis
on
.

95. Teacher E discussed how


electricity flows through wires and
what generates the electric charge.

D. identifying pupils' difficulties

A. art over academics B.


substance over porma

Then she gave the students wires,


bulbs, switches, and dry cells and
told the class to create a circuit
that will increase the brightness of
each bulb. Which one best
describes the approach used?

90. Which behavior is exhibited by


a student who is strong in
interpersonal intelligence?

C. art over science


over substance

A. It used taxonomy of basic


thinking skills

B. interpreting test results


C. computing grades

D. porma

B. Keeps interest to himself/herself.

93. The principle of individual


differences requires teachers to
.

C. Seeks out a classmate for help


when problem occurs.

A. give greater attention to gifted


learners

D. Spends time meditating.

B. provide for a variety of learning


activities

A. Works on his/her own.

91. All of the following describe


the development of children
aged eleven to thirteen EXCEPT

C. treat all learners alike while in


the classroom

A. they shift from impulsivity to


adaptive ability

D. prepare modules for slow


learners in class

B. sex differences in IQ becomes


more evident

94. Value clarification as a strategy


in Values Education classes is
anchored on which philosophy?

C. they exhibit increase objectivity


in thinking
D. they show abstract thinking and
judgement

A. Existentialism
philosophy
C. Idealism

B. Christian
D. Hedonism

B. It was contructivist
C. It helped students understand
scientific methodology
D. It used cooperative learning
96. Rodel is very aloof and cold in
his relationships with his
classmates. Which basic goal must
haye not been attained by Rodel
during his developmental years,
according to Erikson's theory on
psychological development?
A. Autonomy
C. Initiative

B. Trust
D. Generativity

97. Which type of report refers


toon-the-spot description of some
incident, episode or occurrence
that is being observed and
recorded as being of possible
significance?

A. Autobiographical report

101. The first thing to do in


constructing a periodic test is for a
teacher to

B. Biographical report
C. Value and interest report

A. decide on the number of items


for the test

D. Anecdotal report

B. go back to her instructional


objectives

98. The main purpose of


compulsory study of the
Constitution is to

C. study the content

A. develop students into


responsible, thinking citizens

D. decide on the type of test to


construct

B. acquaint students with the


historical development of the
Philippine Constitution

D. prepare students for law-making


99. The following are used in
writing performance objectives,
EXCEPT
B. diagram

C. integrate

D. comprehend

100. Which can effectively measure


students' awareness of values?

D.

102. Teacher F is convinced that


whenever a student performs a
desired behavior, provided
reinforcement and soon the
student will learn to perform the
behavior on his own. On which
principle is Teacher F's conviction
based?
A. Cognitivism
Environmentalism

A. delineate

C. Likert scales
Anecdotal record

A. Scorability
B. Reliability
Objectivity D. Validity

C.

105. In self-directed learning, to


what extent should a teacher's
scaffolding be?
A. To a degree the student needs it.

C. make constitutional experts of


the students

A. Projective techniques
Moral dilemma

104. Teacher F wanted to teach


the pupils the skill to do cross
stitching. Her checkup quiz was a
written test on the steps of cross
stitching. Which characteristic of a
good test does it lack?

B.

C. Behaviorism

B.

D. Constructivism

103. Teacher U teaches to his


pupils that pleasure is not the
highest good. Teacher's teaching is
against what philosophy?
A. Realism
C. Epicureanism
Empiricism

B. Hedonism
D.

B. None, to force the student to


learn by himself.
C. To the minimum, to speed up
development of student's sense of
independence.
D. To the maximum, in order to
extend to the student all the help
he needs.
106. What can be said of Peter
who obtained a score of 75 in a
Grammar objective test?
A. He answered 75 items in the test
correctly.
B. He answered 75% of the test
items correctly.
C. His rating is 75.
D. He performed better than 5% of
his classmates.

107. Which applies when skewness


is zero?

D. By giving your students a sense


of belonging and acceptance.

A. Mean is greater than the median

111. Which types of play is most


characteristic of a four to six-year
old child?

B. Median is greater than mean


C. Scores have three modes

A. Solitary and onlooker plays

D. Scores are normally distributed


108. Which measure(s) of central
tendency separate(s) the top half
of the group from the bottom half?
A. Median
C. Median and Mean
Mode

B. Mean
D.

B. Associative and cooperative


plays
C. Associative and onlooker plays
D. Cooperative and solitary plays
112. For which may you use the
direct instruction method?

109. What was the most prominent


educational issue of the mid
1980s?

A. Become aware of the pollutants


around us.

A. Bilingual Education
Values Education

B.

B. Appreciate Milton's Paradise


Lost.

C. Accountability
Mainstreaming

D.

C. Use a microscope properly.

110. How can you exhibit expert


power on the first day of school?
A. By making them feel you know
what you are talking about.

D. Distinguish war from aggression.


113. Read the following then
answer the question

B. By making them realize the


importance of good grades.

TEACHER: IN WHAT WAYS OTHER


THAN THE PERIODIC TABLE MIGHT
WE PREDICT THE UNDISCOVERED
ELEMENTS?

C. By reminding them your


students your authority over them
again and again.

BOBBY: WE COULD GOTO THE


MOON AND SEE IF THERE ARE
SOME ELEMENTS THERE WE DON'T

HAVE. BETTY: WE COULD DIG


DOWN INTO THE CENTER OF THE
EARTH AND SEE IF WE FIND ANY OF
THE
MISSING ELEMENTS
RICKY: WE COULD STUDY DEBRIS
FROM THE METEORITES IF WE CAN
FIND ANY.
TEACHER: THOSE ARE ALL GOOD
ANSWERS. BUT WHAT IF THOSE
EXCURSIONS TO THE MOON, TO
THE CENTER OF THE EARTH, OR TO
FIND METEORITES WERE TOO
COSTLY AND TIME CONSUMING?
HOW MIGHT WE USE THE
ELEMENTS WE ALREADY HAVE
HERE ON EARTH TO FIND SOME
NEW ONES?
Question: The Teacher's questions
in the above exchange are
examples of
questions.
A. fact
direct

B. concept
D. closed

114. The following are sound


specific purposes of questions
EXCEPT
A. to call the attention of an
inattentive student
B. to teach via student answers

C.

C. to stimulate leamers to ask


questions

to your class? This is a question


that

D. to arouse interestand curiosity

A. directs
B. leads the
student to evaluate

115.Teacher B engages her


students with information for
thorough understanding for
meaning and for competent
application. Which principle
governs Teacher B's practice?
A. Contructivist

B. Gestalt

C. Behaviorist
Cognitivist

D.

116. In a study conducted, the


pupils were asked which nationality
they preferred, if given a choice.
Majority of the pupils wanted to be
Americans. In this case, in which
obligation relative to the state, do
schools seem to be failing? In their
obligation to
A. respect for all duly constituted
authorities
B. promote national pride

C. assesses cognition
creative thinking

challenge them

D. probes

118. A mother gives his boy his


favorite snack everytime the boy
cleans up his room. Afterwards, the
boy cleaned his room every day in
anticipation of the snack. Which
theory is illustrated?
A. Associative Learning B. Classical
Conditioning
C. Operant Conditioning
Pavlonian Conditioning

D. Give them work on the level of


the other students and work a little
above the classmates level to

D.

119. What should a teacher do for


students in his class who are not on
grade level?
A. Give them materials on their
level and let them work at a pace
that is reasonable for them, trying
to bring them up to a grade level.

C. promote obedience to the laws


of the state

B. Give them the same work as the


other students, because they will
absorb as much as they are

D. instill allegiance to the


Constitution

capable of.

117. Read this question: How will


you present the layers of the earth

C. Give them the same work as the


other students, not much, so that
they won't feel embarrassed.

120. Which holds true to


standardized tests?
A. They are used for comparative
purposes
B. They are administered
differently
C. They are scored according to
different standards
D. They are used for assigning
grades
121. Shown a picture of children in
sweaters inside the classroom, the
students were asked this question:
"In what kind of climate do these
children live?" This is a thought
question on
A. inferring
B. applying
creating
D. predicting

C.

122. In his second item analysis,


Teacher H found out that more from
the lower group got the test item #
6 correctly. This means that the
test item
.
A. has a negative discriminating
power
B. has a lower validity

C. has a positive discriminating


power
D. has a high reability
123. Teacher A knows of the illegal
activities of a neighbor but keeps
quiet in order not to be involved in
any investigation. Which
foundational principle of morality
does Teacher A fail to apply?
A. The end does not justify the
means.
B. The principle of double-effect
C. Always do what is right.
D. Between two evils, do the lesser
evil.
124. As a teacher, you are a
rationalist. Which among these will
be your guiding principle?
A. I must teach the child that we
can never have real knowledge of
anything.
B. I must teach the child to develop
his mental powers to the full.

125. Which guideline in test


construction is NOT observed in
this test item: Jose Rizal wrote.
A. The central problem should be
packed in the stem
B. There must be only one correct
answer.

A. View images

B. Attend exhibit

C. Watch a demo

D. Hear

129. Teacher Y does normreferenced interpretation of scores.


Which of the following does she
do?

C. Alternatives must have


grammatical parallelism.
D. The alternates must be
plausible.
126. I combined several subject
areas in order to focus on a single
concept for inter-disciplinary
teaching. Which strategy/method
did I use?
A. Problem-entered learning B.
Thematic instruction
C. Reading-writing activity
Unit method

128. Based on Edgar Dale's Cone of


Experience, which activity is
closest to the real thing?

D.

127. Which is a major advantage of


a curriculum-based assessment?
A. It is informal in nature.

C. I must teach the child so he is


assured of heaven.

B. It connects testing with


teaching.

D. I must teach the child every


knowledge, skill, and value that he
needs for a better future.

C. It tends to focus on anecdotal


information on student progress.
D. It is based on a norm-referenced
measurement model.

A. She describes group


performance in relation to a level of
mastery set.
B. She uses a specified content as
its frame of reference.
C. She compares every individual
students scores with others'
scores.
D. She describes what should be
their performance.
130. The typical autocratic teacher
consistently does the following
EXCEPT
A. encouraging students.
B. shaming students.
C. ridiculing students.
D. intimidating students.

131. Which is one role of play in


the pre-school and early childhood
years?
A. Develops competitive spirit.
B. Separates reality from fantasy.
C. Increases imagination due to
expanding knowledge and
emotional range.
D. Develops the upper and lower
limbs.
132. During the Spanish period,
what was/were the medium/media
of instruction in schools?
A. The Vernacular B. English
C. Spanish
the Vernacular

D. Spanish and

A. Major
Enabling

B. Terminal
D. Primary

C.

135. Which is/are the sources of


man's intellectual drives, according
to Freud?
A. Id
Id and ego

B. Superego
D. Ego

C.

136. Which is an appropriate way


to manage off-task behavior?
A. Make eye contact.
B. Stop your class activity to
correct a child who is no longer on
task.
C. Move closer to the child.

133. An effective classroom


manager uses low-profile
classroom control. Which is a lowprofile classroom technique?
A. Note to parents
school detention

nominator.What kind of objective is


the latter?

B. After-

D. Redirect a child's attention to


task and check his progress to
make sure he is continuing to work.
137. John Watson said: Men are
built not born. What does this
statement point to?

C. Withdrawal of privileges D.
Raising the pitch of the voice

A. The ineffectiveness of training


on a person's development.

134. The primary objective of my


lesson is:To add similar fractions
correctly.Before I can do this I must
first aim at this specific objective:To
distinguish a numerator from a

B. The effect of environmental


stimulation on a person's
development.
C. The absence of genetic influence
on a person's development

D. The effect of heredity.


138. Which does NOT belong to the
group of alternative learning
systems?
A. Multi-grade grouping
B. Multi-age grouping
C. Graded education
D. Non-graded grouping
139. Which activity should a
teacher have more for his students
if he wants them to develop logicalmathematical thinking?

A. Problem solving
reading

B. Choral

C. Drama
Storytelling

D.

140. A goal-oriented instruction


culminates in
.
A. planning of activities
B. evaluation
C. identification of topics
D. formulation of objectives
141. A teacher/student is held
responsible for his actions because
s/he
_.

A. has instincts
mature

B. is

C. has a choice
reason

D. has

142. The burnout malady gets


worse if a teacher doesn't
intervene to change whatever
areas he or she can control. Which
one can renew a teacher's
enthusiasm?
A. Stick to job
B. Initiate changes in jobs
C. Judge someone else as wrong
D. Engage in self-pity
143. With indirect instruction in
mind, which does NOT belong to
the group?
A. Problem solving
recitation

B. Lecture-

C. Inductive reasoning D.
Discovery
144. History books used in schools
are replete with events portraying
defeats and weaknesses of the
Filipino as a people. How should
you tackle them in the classroom?
A. Present them and express your
feelings of shame.

B. Present facts and use them as


means in inspiring your class to
learn from them.
C. Present them and blame those
people responsible or those who
have contributed.
D. Present them as they are
presented, and tell the class to
accept reality.
145. Teachers often complain of
numerous non-teaching
assignments that adversely, affect
their teaching. Does this mean that
teachers must be preoccupied only
with teaching?
A. Yes, if they are given other
assignments, justice demands that
they be properly compensated.
B. Yes, because other community
leaders, not teachers, are tasked to
leadin community activities.
C. NO, because every teacher is
expected to provide leadership and
initiative in activities for
betterment of communities.
D. Yes, because teaching is enough
full time job.
146. "In the light of the facts
presented, what is most likely to
happen when ... ?" is a sample
thought question on

A. inferring

B. generalizing

C. synthesizing

D. justifying

147. Which is a true foundation of


the social order?
A. Obedient citizenry
B. The reciprocation of rights and
duties
C. Strong political leadership
D. Equitable distribution of wealth
148. For maximum interaction, a
teacher ought to avoid
questions.
A. informational
rhetorical
C. leading

B.
D. divergent

149. It is not wise to laugh at a


two-year old child when he utters
bad word because in his stage he is
learning to
.
A. consider other's views
distinguish sex differences
C. socialize
right from wrong

B.

D. distinguish

150. Which guideline must be


observed in the use of prompting
to shape the correct performance
of your students?

A. Use the least intrusive prompt


first.
B. Use all prompts available.
C. Use the most intrusive prompt
first.
D. Refrain from using prompts.
151. Bruner's theory on intellectual
development moves from enactive
to iconic and symbolic stages.

Sundays. Is this in accordance with


the Code of Ethics of Professional
Teachers?
A. Yes. What he does is values
education.
B. No. A teacher should not use his
position to proselyte others.
C. Yes. In the name of academic
freedom, a teacher can decide
what to teach.

In which stage(s) are diagrams


helpful to accompany verbal
information?

D. Yes.What he does strengthens


values education.

A. Enactive and iconic


Symbolic

154. Which does Noam Chomsky,


assert about language learning for
children?

B.

C. Symbolic and enactive


D. Iconic
152. If your Licensure Examination
Test (LET) items sample adequately
the competencies listed in the
syllabi, it can be said that the LET
possesses
validity.
A. concurrent
construct
C. content

B.
D. predictive

153. Teacher F is newly converted


to a religion. Deeply convinced of
his new found religion, he starts
Monday classes by attacking one
religion and convinces his pupils to
attend their religious services on

I. Young children learn and apply


grammatical rules and vocabulary
as they are exposed to them.
II. Begin formal teaching of
grammatical rules to children as
early as possible.
III. Do not require initial formal
language teaching for children.
A. I and III B. II only
D. I and II

C. I only

155. In a social studies class,


Teacher I presents a morally
ambiguous situation and asks his
students what they would do. On

whose theory is Teacher I's


technique based?
A. Kohlberg
C. Piaget

B. Bandura
D. Bruner

156. You arrive at knowledge by rethinking of latent ideas. From


whom does this thought come?
A. Experimentalist
C. Idealist

B. Realist

D. Existentialist

157. A child who gets punished for


stealing candy may not steal again
immediately. But this does not
mean that the child may not steal
again. Based on Thorndike's theory
on punishment and learning, this
shows that
A. punishment strengthens a
response
B. punishment removes a response
C. punishment does not remove a
response
D. punishment weakens a response
158. After giving an input on a
good paragraph, Teacher W asks
her students to rate a given
paragraph along the elements of a
good paragraph. The students' task
is in level of

A. application

B. analysis

B. Short answer

C. evaluation
synthesis

D.

C. Essay

159. Which is most implied by a


negatively skewed score
distribution?
A. The scores are evenly
distributed from left to the right
B. Most pupils are achievers
C. Most of the scores are low
D. Most of the scores are high
160. How can you exhibit referent
power on the first day of school?

A. By making them feel you know


what you are talking about.
B. By telling them the importance
of good grades.
C. By reminding your students your
authority over them again and
again.
D. By giving your students a sense
of belonging and acceptance.
161. If teacher wants to test
students' ability to organize
ideas, which type of test should
she formulate?
A. Multiple-choice type

A. Using common conversion table


for translating test scores in to
ratings

D. Technical problem
162. Ruben is very attached to his
mother and Ruth to her father. In
what developmental stage are they
according to Freudian psychological
theory?
A. Oedipal stage
stage

B. Latent

C. Anal stage
genital stage

D. Pre-

163. Behavior followed by pleasant


consequences will be strengthened
and will be more likely to occur in
the future. Behavior followed by
unpleasant consequences will be
weakened and will be less likely to
be repeated in the future. Which
one is explained?
A. Freud's Psychoanalytic Theory
B. Thorndike's Law of Effect
C. B. F. Skinner's Operant
Conditioning Theory
D. Bandura's Social Learning
Theory
164. Which one can enhance the
comparability of grades?

B. Formulating tests that vary from


one teacher to another
C. Allowing individual teachers to
determine factors for rating
D. Individual teachers giving
weights to factors considered for
rating
165. Based on Freud's
psychoanalytic theory which
component(s) of personality is (are)
concerned with a sense of right and
wrong?
A. Super-ego
Ego C. Id

B. Super-ego and
D. Ego

166. Which assumption underlies


the teacher's use of performance
objectives?
A. Not every form of learning is
observable.
B. Performance objectives assure
the learrier of learning.
C. Learning is defined as a change
in the learner's observable
performance.
D. The success of learner is based
on teacher performance.

167. Who among the following


needs less verbal counseling but
needs more concrete and
operational forms of assistance?
The child who
.
A. has mental retardation
B. has attention-deficit disorder
C. has learning disability
D. has conduct disorder
168. We encounter people whose
prayer goes like this: "O God, if
there is a God; save my soul, if I
have a soul" From whom is this
prayer?
A. Stoic
Empiricist
C. Agnostic

B.
D. Skeptic

169. The best way for a


guidance counselor to begin to
develop study skills and habits
in underachieving student would be
to
.

A. has these underachieving


students observe the study habits
of excelling students
B. encourage students to talk
about study habits from their own
experiences

C. has them view film strips about


various study approaches
D. gives out a list of effective study
approaches
170. To promote effective practice,
which guideline should you bear in
mind? Practice should be
A. done in an evaluative
atmosphere
B. difficult for students to learn a
lesson
C. arranged to allow students to
receive feedback
D. take place over a long period of
time
171. In Krathwohl's affective
domain of objectives, which of the
following is the lowest level of
affective behavior?
A. Valuing
B.
Characterization
C. Responding
Organization

D.

172. With specific details in mind,


which one has (have) a stronger
diagnostic value?
A. Multiple choice test
B. Non-restricted essay test

C. Restricted essay test


D. Restricted and non-restricted
essay tests
173. What is the mean of this
score distribution: 4, 5, 6, 7, 8, 9,
10?
A. 7

B. 6

C. 8.5

D. 7.5

174. Availment of the Philippine


Education Placement Test (PEPT)
for adults and out-of-school youths
is in support of the governments
educational program towards
.
A. equitable access
quality

B.

C. quality and relevance


D. relevance
175. With-it-ness, according to
Kounin, is one of the
characteristics of an effective
classroom manager. Which phrase
goes with it?
A. Have hands that write fast.
B. Have eyes on the back of your
heads.
C. Have a mouth ready to speak.
D. Have minds packed with
knowledge.

176. Teacher B is a teacher of


English as a Second Language. She
uses vocabulary cards, fill-in-theblank sentences, dictation and
writing exercises in teaching a
lesson about grocery shopping.
Based on this information, which of
the following is a valid conclusion?
A. The teacher is reinforcing
learning by giving the same
information in, a variety of
methods.
B. The teacher is applying Bloom's
hierarchy of cognitive learning.

B. By informing them you are


allowed to act in loco parentis.

181. Which of the following


propositions is attributed to Plato?

C. By making them realize the


importance of good grades.

A. Truth is relative to a particular


time and place.

D. By making them feel you have


mastery of subject matter.

B. Human beings create their own


truths.

179. Which questioning practice


promotes more class interaction?
A. Asking the question before
calling on a student.
B. Focusing on divergent question

C. The teacher wants to do less


talk.

C. Focusing on convergent
questions.

D. The teacher is emphasizing


listening and speaking skills.

D. Asking rhetorical questions.

177. With synthesizing skills in


mind, which has the highest
diagnostic value?
A. Essay test
Performance test

B.

C. Completion test
choice tests

D. Multiple

178. How can you exhibit


legitimate power on the first day of
school?
A. By making your students feel
they are accepted for who they are.

180. Theft of school equipment like


tv, computer, etc. By teenagers in
the community itself is becoming a
common phenomenon. What does
this incident signify?
A. Prevalence of poverty in the
community.
B. Inability of school to hire security
guards.
C. Deprivation of Filipino schools.
D. Community's lack of sense of coownership.

C. Learning is the discovery of truth


as Latent ideas are brought to
consciousness.
D. Sense perception is the most
accurate guide to knowledge.
182. In the parlance of test
construction what does TOS mean?
A. Table of Specifics
Specifications

B. Table of

C. Table of Specific Test Items D.


Team of Specifications
183. Read the following then
answer the question:
TEACHER: IN WHAT WAYS OTHER
THAN THE PERIODIC TABLE MIGHT
WE PREDICT THE UNDISCOVERED
ELEMENTS?
BOBBY: WE COULD GO TOTHE
MOON AND SEE IF THERE ARE
SOME ELEMENTS THERE WE DON'T
HAVE.
BETTY: WE COULD DIG DOWN TO
THE CENTER OF THE EARTH AND

SEE IF WE FIND ANY OF THE


MISSING ELEMENTS.
RICKY: WE COULD STUDY DEBRIS
FROM THE METEORITES IF WE CAN
FIND ANY.
TEACHER: THOSE ARE ALL GOOD
ANSWERS BUT WHAT IF THOSE,
EXCURSIONS TO THE MOON, TO
THE CENTER OF THE EARTH, OR TO
FIND METEORITES WERE TOO
COSTLY AND TIME CONSUMING?
HOW MIGHT WE USE THE
ELEMENTS WE ALREADY HAVE
HERE ON EARTH TO FIND SOME
NEW ONES?
Question: Which questioning
strategy/ies does/do the exchange
of thoughts above illustrate?
A. Funneling
and reaping

B. Sowing

C. Nose-dive
Extending and lifting

D.

A. Mastery learning
Indirect Method

B.

C. Morrison method
Questioning method

D.

186. In a treatment for alcoholism,


Ramil was made to drink an
alcoholic beverage and then made
to ingest a drug that produces
nausea. Eventually, he was
nauseated at the sight and smell of
alcohol and stopped drinking
alcohoL Which theory explains this?

A. Operant conditioning
B. Social Learning Theory

C. on-line search
D. computer search
188. Which one can best evaluate
students' attitudinal development?
A. Essay test

B. Portfolio

C. Observation
answer test

D. Short

189. Which are direct measures of


competence?
A. Personality tests
Performance tests

B.

C. Paper-and-pencil tests
D. Standardized test
190. In instructional planning it is
necessary that the parts of the plan
from the first to the last have
.

C. Associative Learning

184. Referring to Teacher S, Nicolle


describes her teacher asfair, caring
and someone you can talk to.Which
power or leadership does Teacher S
have?
A. Referent powerB. Legitimate
power
C. Reward power
power

185. By what name is Indirect


instruction the Socratic method
also known?

D. Expert

D. Attribution Theory
187. The search for related
literature by accessing several
databases by the use of a
telephone line to connect a
computer library with other
computers that have database is
termed
A. compact disc search
B. manual search

A. clarity
B. symmetry
C.
coherence
D. conciseness
191. The cultivation of reflective
and meditative skills in teaching is
an influence of
.
A. Shintoism
Buddhism

B. Zen

C. Confucianism

D. Taoism

192. With which goals of


educational institutions as
provided for by the Constitution
is the development of work skills
aligned?

B. Study Now-Pay Later

A. To develop moral character

195. How would you select the


most fit in government positions?
Applying Confucius teachings,
which would be the answer?

B. To teach the duties of citizenship


C. To inculcate love of country
D. To develop vocational efficiency
193. Researchers conducted show
that teacher's expectations of
students become. Do not require
initial formal language teaching for
children self-fulfilling prophecies.
What is this phenomenon called?
A. Halo effect
Pygmalion effect
C. Ripple effect
effect

B.
D. Hawthorne

194. Under which program were


students who were not
accommodated in public
elementary and secondary schools
because of lack of classroom,
teachers, and instructional
materials, were enrolled in private
schools in their respective
communities at the government's
expense?
A. Government Assistance Program

C. Educational Service Contract


System
D. National Scholarship Program

A. By course accreditation of an
accrediting body
B. By merit system and course
accreditation
C. By merit system
D. By government examinations
196. How can a teacher enhance
his/her questioning technique for
an effective teacher-student
interaction?
A. Immediately
B. You may answer your own
question if no one can answer
C. Allow sufficient think time at
least 7 to 10 seconds
D. Extend wait time until the
students respond
197. A teacher who advocates the
pragmatic philosophy of education

believes that experience should


follow learning thus she has to?
A. Equip her students with basic
skills and abilities
B. Encourage her students to
memorize facts
C. Provide her students
opportunities to apply their skills
and abilities
D. require her students mastery of
the lesson
198. Respect for honest differences
of opinions is one objective
A. Human relationship
B. self-actualization
C. Civic responsibility and
conscience
D. Economic self sufficiency
199. In mastery learning, the
definition of an acceptable
standard of performance is called
A. SMART
B. Condition
C. Criterion Measure
D. Behavior
200. Teacher Luke clears his throat
to communicate disapproval of a

students behavior. Which specific


influence technique is this?
A. Proximity control
B. Interest boosting
C. Signal Inference
D. Direct appeal
201. The main purpose of
compulsory study of the
Constitution is to __________
A. develop students into
responsible, thinking citizens
B. acquaint students with the
historical development of the
Philippine Constitution
C. makes constitutional experts of
the students
D. prepare students for law-making
202. In self-directed learning, to
what extent should a teacher's
scaffolding be?
A. To a degree the student needs it.
B. None, to force the student to
learn by himself.
C. To the minimum, to speed up
development of student's sense of
independence.

D. To the maximum, in order to


extend to the student all the help
he needs
203. Which is the first step in
planning an achievement test?
A. Select the type of test items to
use.
B. Decide on the length of the test.
C. Define the instructional objective
D. Build a table of specification
204. Which may NOT be a benefit
derived from the use of graphic
organizers?
A. Make relationships among detail
clear
B. Enable students to identify
important ideas and details
C. Strengthen team work
D. Represent stated information in
concrete form
205. Rights which cannot be
renounced or transferred because
they are necessary for the
fulfillment of mans primordial
obligations are called
A. Perfect rights
rights

B. Alienable

C. Acquired rightsD. Inalienable


rights
206. Principal Miguel shares this
thought with his teachers: "Subject
matter should help students
understand and appreciate
themselves as unique individuals
who accept complete responsibility
for their thoughts, feelings, and
actions." From which philosophy is
this thought based?
A. Perennialism
B. Essentialism
C. Existentialism
D. Progressivism
207. How can a teacher establish
the reliability of a test?
1) Repeat the same test
2) Administer a parallel test
3) Split the test
4) Vary the number of items
A. 1, 2 and 3
4

B. 2,3 and

C. 1 and 4 only
only

D. 2 and 4

208. On which policy is R.A. 4670


known as the Magna Carta for
Public Teachers focused?

A. Right to establish or join an


organization
B. Code of ethics for professional
teachers
C. Recruitment shall take place
after training
D. Promotion and improvement of
social and economic status of
public school teachers
209. the 19th century saw the
development of state controlled
and state supported public school
system. Existing ideology was the
concept of national sovereignty.
Under a democracy, education
equips citizens with the principles
and duties of citizenship to
guarantee national stability. Such is
the ideology of __________ .

C. Students are given more


opportunity to act or experience
learning.
D. Evaluation is made as an
integral part of teaching
procedures.
211. Which of the following is NOT
a principle of development?
A. Development follows an orderly,
predictable sequence
B. Numerous studies show how
individual develop
C. Early development is more
critical than later development
D. Social expectations influence
development in early stage

A. Citizenship
Progressivism

B.

212. Which questioning technique


would be appropriate for inductive
lessons?

C. Nationalism
Education

D.

A. Use questions requiring only


memory responses

210. The following are the


characteristics of a good
teaching/learning practice EXCEPT?
A. Students are governed by fixed
and rigid standards
C. Life-like situations introduced as
learning experience in the
classroom

B. Expect participation only among


the more motivated students
C. Involve students actively in the
questioning process
D. as teacher, you ask no questions
213. What principle is reflected
when a teacher always provides for
the development of all essential

knowledge manipulative skills and


attitudes?
A. Principle of Needs
of Balance

B. Principle

C. Principle of Unity
of organization

D. Principle

214. A teaching method proceeds


from the details of a lesson towards
generalization is called:
A. Deductive
C. Debate
solving

B. Inductive
D. Problem

215. To accomplish many good


activities in a particular class
period, the teacher must

A. Add curricular activities


B.
Provide unique teaching aids
C. Institute a systematic plan D.
Use expensive material
216. one way to encourage
students who lack in reading to
love reading is to:
A. Reprimand himB. Ask the
students to buy books
C. Provide reading materials D.
Isolate Him

217. Which of the following style or


technique would make the teacher
a good classroom manager
teacher with eyes at the back of
the head?
A. Give your students the head to
toe look when angry
B. Uses eye contact technique with
your students
C. Fear and fight dominates the
classroom atmosphere
D. Being aware of all the actions
and activities in the classroom
218. The sociogram reveals that
four students is your class formed a
clique. This means that the
teacher should:
A. Allow them to be together all the
time

A. Playing is important in the


childs cognitive development
B. playing is a good form of
exercise
C. Playing gives children pleasure
D. Playing is a stage of childs
development
220. The ability to perceive how
objects are related in order to
mentally perceive what is seen,
thus creating concrete visual
images from memory refers to
A. Visual spatial intelligence
B. Musical Intelligence
C. Linguistic intelligence
D. Logical reasoning intelligence

A. Semantic webbing
B. Independent study
C. Role playing
D. Field trip
223. The ________ method is
observed if we wish the pupil learn
from real life situation dilemmas.
A. Situation

B. Lecture

C. Textbooks
Observation

D.

224. The ________ method is used


to make the learners study in detail
a specific thing, person or place not
known to them
A. Case study
Participation

B. Encourage them to join the


groups

221. The _________ method is used


when the learners are made to
observe things in a certain place
like the market.

C. Discourage them from joining


other groups

A. Participatory
study

B. Case

225. Which of the following method


will you used to verify a certain
findings and to make the learners
handle apparatus properly?

C. Simulation

D. Field trip

A. Textbook method
Laboratory method

B.

D. disband the group

C. Field trip method


Project method

D.

219. Parents negatively react to the


inclusion of play in childrens
curriculum. A teacher who knows
the importance of play would tell
the parents that __________?

222. The recommended method to


use if the teachers wishes each
learner to concentrate in learning a
topic to his skills are properly
assessed is.

C. Project

B.
D. Field trip

226. The ________ method is utilized


if the learners are trained to do
creative products.
A. Project
Field trip

B. Case study
D. Simulation

C.

227. If the material is dangerous


for the learners to handle, which of
the following method will you use?
A. Textbook
discussions

B. Group

C. Lecture-demonstration
Eclectic

D.

228. Which method is used to


develop scientific inquiry among
the learners?
A. Project

B. Case study

C. Problem solving
Simulation

D.

229. The ________ method is used


to find out the learners knowledge
about a certain topic assigned to
them.
A. Independent study
C. Lecture
answer

B. Textbook

D. Question and

230. The ________ approach is


utilized when the learners are
trained to ask intelligent question.
A. Process

B. Discovery

C. Inquiry
certification

D. Value

231. If you wish to relate a subject


matter to one of the four principles
of learning, the ________ approach
should be used.
A. Multidisciplinary
Interdisciplinary

B.

C. Conceptual
Integration

D.

232. The ________ approach is used


if the teacher wishes to solve a
problem being met in the school.
A. Mastery

B. Integration

C. Action learning D. Value


clarification
233. You wish to make the learners
learn or internalized fully a subject
matter to be taught to them. Which
of the following will you need?
A. Integrated
media
C. Master
disciplinary

B. MultiD. Multi-

234. Which of the following


approaches will be used if you wish
to relate a particular subject to all
disciplines of learning?
A. Multidisciplinary
Interdisciplinary

B.

C. Value clarification
Integration

D.

235. To enable the learners to learn


by their own pace of growth,
_________ approach is used.
A. Inquiry
Mastery

B. Discovery
D. Modular

C.

236. The ______ approach is


observed when the learners want
to meet the criterion level of
success act set by the teachers.
A. Mastery learning
Interdisciplinary

B.

C. Conceptual

D. Modular

237. Which approach is used to


emphasize the skills in informing
conclusions?
A. Multidisciplinary
Interdisciplinary

B.

C. Conceptual

D. Inquiry

238. Which of the following


approaches is used to include
issues confronting the societies?
A. Integration
Interdisciplinary

B.

C. Conceptual

D. Inquiry

239. The _______ approach is used


to make the learners enunciate

their feelings or attitudes about


certain issues.

B. The use of multimedia


approaches

A. Bilingual ability
cognitive skills

A. Value clarification
Mastery

B.

C. The concept of readiness in


learning

C. A photographic memory D.
Communicative competence

C. Integration
Interdisciplinary

D.

D. The use of reinforcement

246. What is the main concern of


spiraling a curriculum?

240. The _________ method is used


if the learners are to use their
senses effectively.
A. Textbook
demonstration

B. Lecture

C. Observation
Independent study

D.

241. The most important criterion


in test construction is
A. Preparation of table of
specifications
B. Congruency of the items with
objectives
C. Stem should contain the central
problem
D. Options should be almost the
same length
242. Which of the following is the
most important contribution of
Gestalt psychology to the theories
of learning?
A. Cognitive insight

243. What psychological principle


is used when teacher links the new
information to the previous one to
enable the students to gain a
holistic view of the topic?

B. High

A. curriculum renewal and revision


B. Horizontal articulation among
the students in a grade level

A. Stimulation
Accommodation

B.

C. Vertical articulation of a given


subject across a grade level

C. Assimilation
conceptualization

D.

D. Incorporating government
thrusts and societal concerns

244. Young children have a short


attention and interest span. What
kind of task should the teacher
give them?
A. Challenging and interesting
activities
B. Long but interesting activities
C. Easy and difficult activities
D. Short, varied, interesting
activities
245. What is possessed by the
learner when he can use language
with ease and fluency in any given
situation?

247. Which of this information is


not entered in Form 1 or the School
Registrar?
A. Alphabetical list of students,
boys separated from girls.
B. Daily attendance record of each
student.
C. Grade obtained by each student
in all his/her subjects.
D. Personal data of the students in
the registrar.
248. Which type of test is used to
discover further attitudes about
self and

A. Personality test
Intelligence test

B.

C. Achievement test
Diagnostic test

D.

249. Which behavior is exhibited by


a student who is strong in
interpersonal intelligence?
A. Works by his/her own.
B. Spends time meditating.
C. Keeps interest to himself/herself.
D. Seeks out a classmate for help
when problem occurs.
250. A parent visited you regarding
his sons low grades. He showed
you his quizzes, unit tests and
projects. You discovered that his
name is Mel and you wrongfully put
his name on the girls list. What will
you do?
A. Recognize your mistake and
promise to correct the grade
B. Insists: that you are right in
grading him.
C. Refer the matter to the principal
D. Ignore the complain

251. What should a cooperating


teacher do to help the student
teacher who has been assigned to
him/her?
A. Provide opportunities for the
student teacher to acquire the
skills and competencies to be an
effective teacher.
B. Show your lesson plan and let
him/her follow what is in your
lesson plan.
C. Write or prepare activities to be
done and let him/her execute these
in class
D. Dont allow him/her to make her
own decisions as to how the lesson
is to be introduced.
252. Which of these techniques is
BEST suited to developing skill in
asking and answering questions?
A. Interviews
analysis

B. Song

C. Pictorial review

D. Riddles

C. Give them remedial class


Suspend them from classes

D.

253. Children learn what they live


by. Treat them with respect and
they will respect others. Shout at
them and they will be shouting at
others, too. How would you explain
this behavior?
A. They are easily impressed B.
They are imitative
C. they cannot tell right from wrong
D. They are observant
254. What should be done with a
student in the upper grades who is
a non-reader?
A. Encourage him to join a reading
club
B. Give him comic books
C. Have him attend remedial
reading class
D. Transfer him to lower section

252. Most delinquents are found to


have low IQ. They can hardly read
and comprehend. How can a
teacher help them?
A. Call for their parents.
B. Refer them to a doctor.

255. In which situation is learning


most likely to happen?
A. When students work by
themselves
B. When students are quiet and
well behaved
C. When all the needed materials
are available

D. When students know the


importance of the task at hand
256. Social adjustment means the
ability to behave in accordance
with?
A. Universal truths
B. Self-concept

outs in his school so he could find


solutions to this problem. What
type of researched is used?
A. Applied

B. Action

C. Pure
Experimental

D.

weighs for the qualities of a


product or a performance?
A. Performance based
scales

B. Rating

C. Holistic rubric
rubric

D. Analytic

260. If we aim to produce globally


competitive graduates, the
Philippine education should give
major emphasis to _______________.

263. How can a teacher develop


the value of time such as
punctuality and maximal utilization
of time?

257. How can a teacher avoid


breakdown and interruptions I
daily class procedures?

A. English, Science and


Mathematics

A. Avoid disruptions due to


improper behavior

B. Technology and citizenship


education

B. Consistently follow schedule for


classroom routine

A. Punish the misbehaving students

C. Humanities and work education

B. Establish routine for daily tasks

D. Bilingual education and values


education

C. Rush if you are getting late for


the right time

C. Stereotyped behaviors
D. Social expectations

C. Assign a leader to assist


everyone
D. Allow students to make their
own regulations
258. Which is made after certain
norms has been established?
A. Departmental test
Local-city wide test

B.

C. Teacher-made test
Standardized test

D.

261. Which of the following can be


considered a form of civic
engagement?
A. Patronizing Filipino products
B. Critical of government officials
C. Bribing government officials
D. Electoral participation

259. A high school principal would


like to know the causes of drop-

262. A teacher wants to make a


rubric for scoring students' output.
Which format will use differential

D. Have a big clock installed in the


classroom for everyone's guidance
264. One learns by association and
also by insight. This shows that the
association and cognitive theories
of learning are:
A. Diametrically opposed
B. Complementary
C. Partly wrong
Partly correct

D.

265. Mothers who demand their 3


to 5 year old child to suspend their
time in serious academic study,
forget that early childhood is in the

A. Gang age
Questioning age

B.

C. Initiative age

D. Toy age

266. Identical twins are more than


alike than fraternal twins. Which of
the following statements/principles
is supported by this?
A. Heredity has a part in
determining physical appearance
B. Intelligence is determined partly
by pre-natal nutrition
C. Environment affects both
fraternal and identical twins
D. Intelligences hinges in physical
structure
267. Which of the following
activities is stressed by humanistic
education?

A. Enjoy the great works of man


such as classics
B. Learn the philosophy know
thyself
C. Make the distinctly civilized,
educated and refined
D. Develop man into a thinking
individual

268. For a teacher to be


competent, he/she is required to
specialize in certain area. This pillar
of learning is
A. Learning to do
to live together

B. Learning

C. Learning to be
to know

D. Learning

269. What pillar of learning is


concerned on material
development rather than of human
development?
A. Learning to do B. Learning to
live longer
C. Learning to be
to know

D. Learning

270. What is the advantage of


using computers in processing test
results?
A. Its processing takes a shorter
period of time.
B. Test results can easily be
assessed.

A. It is easy to conduct
B. It contains many responses
C. measure several competencies
in one test
D. It possess the qualities of other
types of tests
272. While serving during the
elections, some trouble makers
enter your precinct. What do you
think is the most appropriate thing
to do?
A. Challenge them to a duel
B. Close the precinct and go home
C. Ignore them and look for a safe
place
D. Seek the assistance of law
enforcers
273. Which of the following will you
recommend to a senior high school
scholar who is impregnated by a
fellow student?
A. Force her boyfriend to marry her

C. Its statistical computation is


accurate.

B. Tell the parents about the


condition

D. All of the above.

C. Stop schooling till after she gave


birth

271. Multiple choice test is


considered the BEST type of test
because _______________

D. Direct her to an abortion clinic

274. Which describes an


inappropriate practice in the
education of young children?
A. Individual differences are
expected and accepted
B. Integrated teaching-learning
C. Isolated skills development
D. positive guidance techniques
275. Honesty remains a value even
if nobody in a organization values
it. This pronouncement comes from
the mouth of a(an)
A. Pragmatist

B. Idealist

C. Reconstructionist
Progressivist

D.

276. I cannot forget my friends


birthday for it comes one day after
my birthday. Which principle of
association as applied to memory
explains this?
A. Contiguity
Similarity

B.

C. Frequency

D. Content

277. A fresh teacher graduate is


usually idealistic. Which one will
most likely inspire her to cling to
her idealism?
A. Introduction of educational
innovations

B. Support of living models

A. Life

C. Pressure of work
D. High salary
278. I cannot help but recall the
sisters convent which served as
my boarding house in high school
now that I am in a noisy boarding
house. Which principle of
association explains this?
A. Similarity
Contiguity

B.

C. Frequency

D. Contrast

C.

280. Whose thought is this:


Although there is an external
world from which human beings
acquire sensory information, ideas
originate from the workings of the
mind.
A. Idealist
Empiricist

B. Realist
D. pragmatist

282. This theory states that there


are 8 basic development stages
that the individual has to pass
through his life
A. Learning Theory
B. Psychoanalytic Theory
C. Psychosocial Theory
D. Cognitive Development

279. Which principle of association


ass applied to memory is this? The
recall of an object or idea triggers
recall of other objects like it
A. Contrast
B. Contiguity
Similarity D. Frequency

B. Birth C. Heredity
D. Character

C.

281. The process by which certain


potentials are inherited from the
parents for his development

283. Transition age from childhood


to adulthood where rapid physical
changes and sex maturity occur
resulting in changes in ways of
feelings, thinking and acting.
A. Puberty

B. Adolescence

C. Early Adulthood

D. Stage V

284. Modifying an existing scheme


after an individuals interaction
with the environment, resulting in
the creation of a new scheme.
A. Assimilation
Interaction

B.

C. Recognition
Accommodation

D.

285. Theory stating that a persons


behavior can be motivated by
urges towards self-satisfaction.

A. Psychoanalytic Theory
B. Cognitive Development Theory

B. Obtained in the field of music,


art and literature

C. Psychosocial Theory
D. Moral development theory
286. The ability of a child to
conceptualize the retention and
preservation of the same quantity
under various changes.

A. Recognition

B. Reversibility

C. Assimilat8ion D. Conservation
287. Refers to the idea that no
individual are exactly the same or
alike.
A. Cognitive theory
Exclusivity theory

B.

C. Individual Differences
D. Emotional Quotient
288. He is known as the Father of
Modern I.Q. Test
A. Lewis Terman
Erickson

A. Base on the premise that all


learning has emotional correlates

B. Erick

C. Laurence Kohlberg
Martin Lesley
289.Intellectual appreciative
experience is

D.

C. the acquisition and retention of


facts and information
D. assumes that human activities
are based on stimulus and
response
290. These statements imply that
children at the early learning stage
consider parents and teachers as
authorities and models.
A. Parents and teachers should
always coordinate childrens
activities
B. Parents should enforce strict
discipline at home and teachers in
school
C. Parents and teachers should be
the role models at all times
D. Parents and teachers should
always consult each other with
regards the childs intellectual
development
291. Any change in the behavior of
an individual
A. Learning

B. Response

C. Change

D. Development

292. Which of the following


principles IS NOT considered under
Classical Conditioning by Ivan
Pavlov?
A. Excitation
Adhesive Principle

B.

C. Stimulus Generalization
None of the above

D.

293. This stimulation of action best


explains the behavior of an
individual to take what he
perceives to be the shortest route
to his goals.

A. Recognition
Assimilation

B.

C. Response
Motivation

D.

294. The process by which an


individual acquires the social and
cultural heritage of the society
where he belongs.
A. Socialization
B. Internalization
C. Integration
D. Acquisition
295. What is the main function of
the philosophies of education?

A. To aid the leaner to build his/her


own personal philosophy.

D. Develop man into a thinking


individual

A. infancy
childhood

B. early

B. Define goals and set directions


from which educational efforts
should be exerted.

298. A teacher who advocates the


pragmatic philosophy of education
believes that experience should
follow learning, thus, she has to?

C. Puberty

D. Adulthood

C. Provide guidelines in the


foundation of educational policies
and programs

A. require her student mastery of


the lessons

D. Provide norms and standards for


evaluation purposes.

B. encourage her students to


memorize facts

296. According to Froebel,


kindergarten is also known as

C. equip her students with basic


skills and abilities

A. A place where children have fun


and enjoyment

D. provide her student with


opportunities to apply their skills
and abilities

B. Garden where children could


grow and develop
C. The learning center of life
D. A place where new beginnings
begin
297. Which of the following
statements is given emphasis by
humanistic education?
A. The great works of man such as
the classics should be enjoyed.
B. Man should learn the different
philosophies of education
C. Build a man who is distinctly
civilized, educated and refined

299. How are institutions of


learning encouraged to set higher
standards over and above the
minimum requirement for state
recognition?
A. Scholastic achievement
B. Faculty development
C. Academic freedom
Voluntary accreditation

301. Claustrophobia is an irrational


fear of
A. Darkness

B. Strangers

C. Closed Space

D. Height

302. An eye defect characterized


by clear vision in one dimension
but unfocused vision on the other
is called
A. Myopia

B. Astigmatism

C. Hyperopia
Presbyopia

D.

303. Which of the following


statements does not apply to
adolescents?
A. They desire the approval of their
peers
B. They seek dependence on their
parents

D.

300. The period of physical,


especially sexual, and mental
maturation which is characterized
by rapid somatic growth is known
as

C. They have a marked sex


development
D. None of the above

304. As young people mature,


society expects them to develop

competencies and assume social


roles in a conventional manner.

D. there is no marked differences in


their time of maturity

A. it remains fairly constant


is highly changeable

B. it

A. expectation of parents
B. influence of peers groups

308. Rationalization is used by


student who

C. it is affected by attitude
is never constant

D. it

C. influence of formal education


D. cultural demands
305. The founder of the theory of
psychology called psychoanalysis
was
A. Lock
B. Hume
D. Leibnitz

C. Freud

A. Always give explanation or


reason for their failures rather than
own their faults
B. Like to take the blame for their
faults
C. bribe their elders with promises

306. When the learner reaches a


point where no further
improvement can be expected, he
is in a so-called
A. Development crisis
plateau

B. Learning

C. Regression
Depression

D.

307. Regarding the sexual


maturation of boys and girls,
teachers should bear in mind that:
A. Girls mature at a late stage than
boys
B. Girls mature at an earlier stage
than boys
C. Boys and girls mature at the
same time

312. Transfer of training easily


takes place if the activities involved
A. Are different
B. Have identical element
C. Occur in the same place
D. Vary in difficulty

D. substitute words for deeds

313. When the learner is wellmotivated, he performs his task

309. Which of the following is true


of Abnormal Psychology?

A. with indifference
B. with disinterest

A. It studies the cause of


personality defects

C. with arrogance

B. It measures the
accomplishments of the individual

D. with enthusiasm

C. it concentrates on the scholastic


performance of the individual

314. A six-year-old child who has a


mental age of eight years has an IQ
of

D. it investigates the educational


background of the individual

A. 120
133

310. Which of the following is a


continuous variable?

315. The ratio obtained by dividing


mental age by chronological age
times 100 is called

A. Weight
Nationality

B. Sex
D. Race

C.

311. Which of the following is true


about ones IQ?

B. 130 C. 132

A. derived quotient
deviation

D.

B.

C. Intelligence quotient or IQ D.
Intelligence ratio
316. Which of the following was
written by Plato?
A. Sic et Non
School and Society

B. The

C. The Republic

D. Emile

317. Who among those below


asserted that Education is for
complete living?
A. Dewey

B. Spencer C. Kant
D. Froebel

318. The right of an educational


institution and its faculty to
prescribe the methods/strategies of
teaching refers to:
A. Building style
B. Choice of curriculum
C. Academic freedom
D. Co and extra-curricular program
319. The 1987 Constitution
provides that religious instruction
can be given
A. with the students consent
B. with the parent/guardian
approval

C. with mayors permit

320. Public schools in the


Philippines are the contribution of
which colonizer?

323. A teacher who gives a uniform


assignment to be worked out by all
learners in Arithmetic is not
observing a characteristic of a
good assignment. Which
characteristic is overlooked?

A. American

B. British

A. It should be definite

C. Japanese

D. Spanish

B. It should be stimulating

D. with the schools support

321. Hardship allowance is given to


a teacher when

C. It should emphasize the


essential

A. s/hes assigned in a depressed


area

D. It should provide for individual


differences

B. s/hes given additional teaching


load

324. If a student asks a question


which the teacher does not have a
ready answer, the latter should:

C. s/hes assigned in a barangay


high school
D. s/hes assigned in a hazardous
area
322. Which of the following
principles of human growth refers
to the situation that girls mature
earlier than boys?
A. Growth follows a sequential
pattern
B. Development rates vary
C. Each stage of development has
characteristics traits
D. Maturation should precede
certain type of learning

A. Dismiss the question as


irrelevant
B. Offer a bluff
C. Admit the fact that he doesnt
know the answer
D. Ask volunteers to answer the
question and do research on it
later.
325. The heredity traits acquired
by a person in his lifetime
A. Are transmissible to his offspring
B. Reappears in his future
grandparent

C. Have no influence on the


offspring

C. That the girl has a chronological


age of 15

A. Visual intelligence
Linguistic Intelligence

B.

D. Become recessive traits

D. That she has a mental age of 13

C. Feelings sensitivity

D. Jargon

326. When student are given a


chance to settle differences of
opinion by discussion, they
develop:

329. Which statement is not


necessary to achieve the learners
interest in a learning activity?

332. The sensitivity to tone and


pitch, allowing one to produce
musical scoring is intelligence in?

A. The activity must lead to a


practical end

A. Musical
Ability

B. The activity must be within the


ability of the learner

C. Quantitative exercises
D. Qualitative analysis

C. The activity must fill a need


recognized by the learner

333. Ones ability to do abstract


reasoning and manipulate symbols
refers to what type of intelligence?

A. Fair play

B. Tolerance

C. Irritants

D. sociability

327. The schools responsibility


towards teenagers gang age is:
A. Provide the gang all the freedom
it needs
B. Gives classroom activities to
give direction to out-of-school
youth activities
C. Supervise gang activities
D. set up norms of conduct or the
member of the gang

328. In an intelligence test, a 13year old girl got a score equivalent


to that of a 15-year old. This
means:
A. That the girl must be
accelerated
B. That the girl 12-years older
mentally

D. The learner must have the


experience that will furnish the
background for the activity
330. He is responsible for the
theory which recognizes the
importance of developing multiple
intelligence?
A. Jean Piaget
Gardner

B. Howard

C. Frederick Froebel
Freud

D. Sigmund

331. The need to recognize and


develop special sensitivity to
language, thus helping the learners
to use the right word, phrase
and/or graph to grasp new
meaning refers to

A. Musical
identification

B. Verbal

B. Personality

C. Mental ability
D. Mathematical-logical
334. The ability to perceive how
objects are related in order to
mentally perceive what is seen,
thus creating concrete visual
images from memory refers to?
A. Visual-spatial Intelligence
B. Musical
C. Language
D. Logical reasoning
335. The capacity to analyze ones
feelings and thus be able to
understand and be able to know

the motives of other peoples


actions.

other, adjustments to varying


situations, behavior an motivations

A. Spatial

B. Personal

A. Emotional Quotient (E.Q.)

C. Logical
Diametric

D.

B. Intelligence Quotient (I.Q.)


C. Maladjustment personality
D. Anticipated behavior

336. The type of intelligence which


enables a person to understand
other persons feelings, behavior
and motivation.
A. Emotional
Spatial

B.

C. Multiple intelligences

337. The type of intelligence which


characterizes actress, actors,
mimes, dancers and people of the
Arts?

C. Research
Emotions

A. Emotional quotient (E.Q.)


B. Intelligence Quotient (I.Q.)

C. Social intelligence
D.
Quantitative and Qualitative

A. Bodily-kinesthetic
Scientific

339. It is a measurement of
personality which is the result by
dividing the mental age by the
chronological age.

B.
D.

338. An emerging thrust in


determining ones personality,
whether pleasant or unwholesome,
this type of personality
measurement is the
wholesomeness of ones virtues,
i.e., values, relationships with

D. Forecasted behavior quotient


340. The teacher must be aware
that both heredity and environment
represent complex factors, exerting
many specific influences on an
individuals growth. Which of the
following statements best
represents the influence of heredity
and environment?
A. Heredity counts; environment is
less important.
B. If the environment is changed,
heredity becomes less important.

C. The relative influences of


heredity and environment can vary
widely in an individuals growth
D. In the long run, both tend to
cancel each others influences
341. Heredity has a part in
determining intelligence. Which of
the following statements support
this principle?
A. Environment affects both
fraternal and identical twins
B. Intelligence hinges in physical
structures
C. Intelligence is determined partly
by pre-natal nutrition
D. Identical twins are more alike
than fraternal twins
342. Educators who contributed to
the open education movement
include:
A. Neill and piaget
Kohl and kozol

B.

C. Bruner and Silberman


D. All of the above
343. A childs social skills can be
measured by:
A. direct observation and parentteacher conferences
B. psychological test

C. adaptive behavior scales


D. A and C above
344. A teacher uses behavioral
modification techniques in his
classes. Which of the following
student behaviors would he find
most difficult to change?

B. They have short attention spans


and experience difficulty in
generalizing
C. Their reading, writing, and
arithmetic skills cannot be
improved
D. A and B above

A. Aggressive tendencies towards


classmates

347. Which of the following are


characteristics of a dyslexic child?

B. Poor habits in organizing work


materials

A. Mirror writing
listlessness

C. Interrupting a speaker

C. Below-average intelligenceD.
Hyperactivity

D. Abandoning a project before it is


finished
345. Learning-disabled children
most characteristically have:

B.

C. An average level of intelligence


D. minimal brain damage
346. Which of the following is true
about educable mentally retarded
children?
A. Their IQ range between 50 and
70

B. Eliminate testing
C. Eliminate extrinsic motivations
D. maintain a certain anxiety level
for increased competition
350. The knowledge explosion has
led to crowding more and more
information into curriculum
courses. A likely result is that:
A. the textbook will no longer be
the main instructional medium in
many classes

348. Students with secondary


reading problems have capacity to
read, but are non-readers because
of:

B. the child may spend more time


in school

A. auditory problems

D. all of the above

B. congenital defects

351. During the learning process


the teacher has most control over:

A. low IQ
B. poor socio-economic
backgrounds

A. Meet both physiological and


intellectual needs of students

C. visual-acuity impairment
D. Environmental or Emotional
factor
349. If a teacher accepts Maslows
theory on the hierarchy of needs,
he or she will probably structure
objectives to:

C. the teacher may have to rely


more on these multimedia
materials

A. The learners
B. The learning environment
C. The Learning process
D. The behavior of the learners
352. Which of the following
conditions does NOT contribute to

a climate psychologically suited to


learning?
A. The teacher acts like a real
person.
B. The teacher makes all of the
decisions about students learning
activities.

D. A sensitivity-training meeting for


the purpose of helping students
ace their school-related problems
and learn how their actions can
affect others.

LET REVIEWER
GENERAL EDUCATION- SOCIAL
STUDIES

C. The teacher accepts students as


they are

1. Which statement is TRUE of the


pre- Spanish Filipino government?

D. The teacher shows trust in


students decisions

a. The datu exercised all the


powers of government.

353. William Glasser advocates the


frequent use of classroom
meetings, with teacher and
students sitting in a small circle.
Which one of the following types of
discussion would NOT be
appropriate in such a setting?

b. Laws were formulated by a law


making body elected by the datu.

A. An educational-diagnostic
conference on the learning
weaknesses of individual students
B. .An open.-ended meeting for the
purpose of exploring and
discussing students ideas about
the curriculum.
C. A social-problem-solving
meeting to resolve teacher or
student problems relating to the
school, the class, or any individual
member.

c. Laws were formulated by a law


making body elected by the
community.
d. There was a court created by the
datu to hear complaints.
2. What characteristic/s of the
government is established by the
1987 Constitution?
I. Presidential system of
government with three branches.
II. Parliamentary system of
government.
III. The three branches of
government are separate and
independent of one another.

IV. The three branches of


government have a check and
balance over one another.

5. In the decade of the 70s, one


clamor of the activists who staged
street demonstration on

a. I only

the streets was Down with the


oligarchs! What did they accuse
government of as

b. II only
c. II, III, and IV
d. I, III, and IV
3. If our present government is a
democracy, where does power
reside?

d. Capitalism

b. A form of dictatorship

9. In which form of government


does power or authority reside in a
few persons who govern

c. Anti-poor

6. With the Batasang Pambansa


performing legislative and
executive powers in the Marcos

about the Philippine


Commonwealth? The Filipino
people _____________________.
a. Were fully independent.
b. Were partially independent.
c. Were granted total freedom to
course their destiny only in matters
of education.
d. Were citizens of the United
States.

b. Totalitarianism

a. A rule of the few rich

b. In Congress

4. In President Quezons time, the


country had the Philippine
Commonwealth. What is TRUE

a. Colonialism

c. Democracy

d. Elitist

d. In the Supreme Court

8. To which type of political system


do we belong?

suggested by the underlined word?

a. In the Filipino people

c. In the President

d. Monarchical

regime, which form of government


was implemented?
a. Parliamentary
b. Dictatorial
c. Monarchial
d. Presidential
7. What form of government is
characterized by the separation of
powers?
a. Parliamentary
b. Presidential
c. Aristocracy

for their own interest?


a. Oligarchy
b. Monarchy
c. Democracy
d. Parliamentary
10.Which of the following
represents the smallest subunit of
government in the Philippines?
a. Purok
b. Barangay
c. Sitio
d. Zone
11. The Philippine government is
divided into three branches:
executive, legislative and

judiciary. Which among the


following doctrines best describes
the model of governance in
the Philippines?
a. Tricameralism
b. Separation of powers
c. Emancipation of state
d. Division of labor
12.The Philippine legislature is
divided into two major bodies, the
Senate and the House of
Representatives. Which among the
following describes this division of
the legislative body
of the country?
a. Bipartisanism
b. Co- legislative power
c. Unicameralism
d. Bicameralism
13.Which type of governance is
characterized by a union of
partially self- governing states or

c. Democracy

b. Right to secure persons

d. Totalitarianism

c. Right to process paper

14.In England, Queen Elizabeth


acts as a head of state. This
country also has a parliament

d. Right to protection

and a Prime Minister. Which among


the following describes the form of
government?
a. Totalitarianism

integration of persons with


disabilities into the mainstream of
society?

b. Constitutional monarchy
c. Oligarchy

a. Republic Act No. 7277

d. Aristocracy

b. Republic Act No. 7784

15.What right is violated when one


opens a letter without permission
from the addressee?

c. Republic Act No. 7722

a. Right to privacy of
communication and
correspondence.
b. Right to read a letter.
c. Right to open the envelope
without permission.
d. Right to private affairs.

a. Federalism

16.When an individual is
imprisoned without proper
investigation, what right is
violated?

b. Socialism

a. Right to due process of law

regions united by a central


government?

17.What law was passed by the


Philippine Congress in 1995 which
affirms the total

d. Republic Act No. 7776


18. An individual has the right to
file writ of amparo before the
investigation of an
administrative case filed against
him/ her. What fundamental right is
invoked by the
individual?
a. Right to life, liberty and security
b. Right to due process
c. Right to be defended by a public
attorney
d. Right to self- defense

19.What is the writ of habeas


corpus?

c. Private enterprise and incentives


to needed investments

a. Right to information privacy

d. Rural development and agrarian


reform

b. Right to accumulate data


c. Right to transmit data
d. Right to search for private
information
20. Which principle states that no
man in his country is above the law
and that laws must be
obeyed by all and applied to
everyone rich or poor, lowly or
powerful, without fear or

22. Filipino citizens have the power


to participate in the establishment
or administration of
government such as right to vote
and be voted upon as an exercise
of
____________________.
a. Political rights
b. Civil rights

favor?

c. Social rights

a. Rule of law

d. Economic rights

b. Rule of the majority

23. Which right is violated by


wiretapping?

c. Separation of church and state


d. Social justice
21.In connection with government
transactions involving public
interest which policy is
adopted in the Constitution to
assure public of accountability and
transparency?

c. Claro M. Recto
d. Eulogio Rodriguez
25.Who first introduced the Islamic
religion to the Philippines?
a. Rajah Baguinda
b. Idi Amin
c. Mukdum
d. Abu Bakr
26. The Japanese successful
invasion was climaxed by the
surrender of the joint Filipino
American forces on May 6, 1942.
Where did this happen?
a. Bataan
b. Corregidor
c. Capas

a. Freedom of conversation

d. Manila

b. The right to private property

27.Who is the President who is


known for his Filipino First Policy
and Austerity Program?

c. The right to privacy of


communication
d. Right to information on matters
of public concern

a. Full public disclosure

24.In Philippine history, who was


known as The Great Dissenter?

b. Balanced and healthful ecology

a. Camilo Osias
b. Manuel Roxas

a. Diosdado Macapagal
b. Carlos P. Garcia
c. Manuel Roxas
d. Ramon Magsaysay
28. With whom is the Strong
Republic associated?

a. Fidel V. Ramos
b. Joseph Estrada

c. The total value in pesos of


services produced during the year.

b. Gloria M. Arroyo
c. Corazon C. Aquino

d. Corazon Aquino

d. The total value in pesos of goods


and services produced for 2
quarters.

29.When one company controls the


supply of sugar, which term
applies?

32. Which term refers to social


aspects of sex or to socially defined
roles and expectations

a. Monopoly

that are associated with sex?

their marriage customs

b. Inflation

a. Social differentiation

c. Economic stability

b. Social class

b. A student who tutors an


immigrant in English

d. Equilibrium

c. Gender

30. Tomatoes as off season and so


price for tomatoes are up. Which
law/principle in

d. Cultural context

c. Gloria Macapagal- Arroyo

Economics explains it?

33. Which does one portray when


he thinks that what is foreign is
best and that what is local

a. Substitution effect

is inferior?

b. Law of cause and effect

a. Xenocentrism

c. Principle of marginal utility

b. Relativism

d. Law of supply and demand

c. Ethnocentrism

31. Which explains GNP?

d. Favoritism

a. The total value in pesos of goods


and services produced during the
year.

34. The statement that success


often comes to those with humble
beginnings would apply

b. The total value in pesos of goods


produced during the year.

best to which of the following


figures?
a. Ramon Magsaysay

d. Joseph E. Estrada
35.Which of the following represent
ethnocentric behavior?
a. A tourist who lectures his foreign
hosts on the "uncivilized" nature of

c. A Hispanic community group


demands that public aid forms be
published in English
and Spanish
d. A peace Corps volunteer who
helps dig wells in Central Africa
36.With the promotion of social
justice in mind, which does NOT
belong to the group?
a. Equitable access to education
b. Profit sharing
c. Diffusion of wealth
d. Absolute right over property
37.Which part of the Visayas
receives the least precipitation?
a. Northern

b. Eastern

d. price ceiling

c. Western

41.The following are legitimate


children, EXCEPT ___________.

d. Central
38.When the Filipino reformists
asked for the assimilation of the
Philippines by Spain, what

a. Those born by artificial


insemination.

c. a geographical aggregate who


live a common life and interact
with one another
d. all of the above
e. none of the above

b. Those legitimated.

44.Sociologists define culture as:

did they ask for? For the Philippines


to ________.

c. Those born during a valid


marriage of parents.

a. a state of refinement

a. become independent from Spain

d. Those born out of a valid


marriage of the parents.

b. become a province of Spain


c. be independent from Spain with
certain conditions
d. be represented in the Spanish
Cortes
39.How is the so-called colonial
mentality manifested?
a. Cultural relativism
b. Cultural diversity
c. Xenocentrism
d. Ethnocentrism
40. The maximum price that can be
legally changed for a good or
service is called _____.
a. minimum wage
b. price floor
c. legal wage

42.Which location should have


most nearly twelve hours of
daylight and twelve hours of
darkness during December? A town
that is located __________.
a. Halfway between the Equator
and the South Pole.
b. Close to the Equator.
c. Close to the North Pole.
d. Close to the South Pole.
43.Society denotes
______________________.

b. the plastic and graphics arts


c. activities such as TV sitcoms,
soap operas or rock music
d. the totality of meanings, values,
customs, norms, ideas and symbols
relative to a society
e. none of the above
45.The obligation to repay a person
for whom one has received a favor
with undefined
quantification is ___.
a. Pakikisama d. utang na loob
b. kusang loob e. none of these
c. magandang loob

a. the totality of social organization


and its complex of social
relationships

46.The occurrence of graft and


corruption, nepotism and favoritism
in the Philippines if

b. a group of people who share a


common culture

often attributed to the value of:


a. Personalism d. hospitality

b. Impersonalism e. none of these


c. utang na loob
47.The type of residence which
permits the newly married couple
to reside independently of
the parents of the bride and the
groom is
a. patrilocal d. neolocal residence
b. matrilocal e. none of these
c. bilocal
48.The movement of a person or
group of persons to another place
more or less for
permanent residence is
a. migration
b. population growth
c. population change
d. ecological movement
e. none of these
49.____ is the material wealth given
to the bride and her family before
marriage.
a. bride wealthd. fetish
b. fosterage all of these
c. dowry

50.The division of society into


layers in termed as

b. Right to be given the due


process of law.

a. stratification d. life chance


matrix

c. Right to be defended by the


organization of teachers to which
he/ she is a member.

b. status inconsistency e. none of


these
c. status defects
51.The most preferred form of
marriage in human societies is
a. monogamy d. polyandry
b. polygamy e. all of these
c. bigamy
52. What is writ of habeas data?
a. Right to information privacy
b. Right to accumulate data
c. Right to transmit data
d. Right to search for private
information
53.When a teacher is charged with
an administrative case committed
in the lawful discharge
of professional duties, what right
may the teacher invoke for her
defense?
a. Right to receive compensation
for the duration of the case.

d. Right to be provided with free


legal service by the appropriate
office.
54.Can you be arrested without a
warrant of arrest?
a. No, if you are a minor.
b. No, if you are more than 60
years old.
c. Yes, if you were reported to have
committed a crime.
d. Yes, if you are in the act of
committing a crime.
55.Can a person be imprisoned for
debt?
a. No, if he cant pay the interest of
his debt.
b. Yes, if he has no property with
which to pay his debt.
c. No.
d. Yes, if he stubbornly refuses to
pay.

56.What does presumption of


innocence mean in so far as
human rights are concerned?
a. A suspect is considered guilty
until proven otherwise.
b. A suspect has the right to remain
silent.

59.Who has the power to declare


the existence of a state of war?

b. It has been an independent


nation ever since.

a. Chief justice

c. It has evolved from a colony to a


fully independent nation.

b. President
c. Senate President
d. Congress

c. A suspect has the right to a legal


counsel.

60.How is the crime of rape


classified?

d. A suspect remains innocent until


proven guilty.

a. Heinous

57.No person may be elected as


President of the Philippines unless
he/ she is a ____________.
a. Natural born citizen
b. Resident of the Philippines for at
least 2 years
c. At least 21 years old
d. Professional
58.A bill becomes a law even if not
signed by the President after ___
days.
a. 60

b. Homicide
c. Slander
d. Malicious Mischief
61.All Filipino citizens have the
right to vote and to be voted upon
as a government official.

d. 30

63.Basically, the Philippine agrarian


reform program is a question of
_______________.
a. Land distribution
b. Nepotism and corruption
c. Graft and corruption
d. Environmental degeneration
64.Which is TRUE of the
Philippines?

What is this constitutional right


called?

a. It has been colonized by two


European colonial powers.

a. Passive right to vote

b. It has never been united as a


nation.

b. Political franchise
c. Suffrage
d. Electoral right

b. 40
c. 20

d. It has not achieved full


independence from the very
beginning.

c. There have been attempts to


change its government to a
parliamentary
form.

62.Which is TRUE of the historical


development of the Philippines?

d. It is the only Christian country in


Asia.

a. It has never been an


independent nation.

65.Which is TRUE of former


president Corazon Aquino?

I. The first female president of the


Philippines.
II. The first female president in
Asia.
III.Described as the icon of
democracy.

68.During the Spanish era, who


was the revolutionary leader who
waged a long war against
the government because it did not
allow proper burial for his brother?
a. Macario Sakay

a. I, II, and III

b. Andres Bonifacio

b. I and II

c. Diego Silang

c. I and III

d. Francisco Dagohoy

d. I only

69.What was the first book


published in the Philippines?

66.Which of the following was first


to happen?

a. Del Superior Govierno

a. Aguinaldo was captured.

b. Doctrina Christiana

b. Aguinaldo declared Philippine


independence.

c. Pasiong Mahal

c. Guerilla warfare against the US


was initiated.
d. The Philippines was ceded to the
US by the Treaty of Paris.
67.In 1565, Legaspi concluded a
blood compact with the chief of
Bohol. Who is referred to?
a. Raja Sulayman
b. Sikatuna
c. Lakandula
d. Rajah Tupas

d. Barlaame Josaphat
70.While in the capitalist system,
to the strongest goes the spoil,
what happens in the
cooperative system?
a. All benefits are shared to the
members proportionate to the
shared capital.
b. Goods are distributed among the
members but benefiting mostly the
policymaking
body.

c. Goods are distributed among the


members.
d. Goods are distributed among the
members but elected and
appointed officials get
more to comprehensive their
service.
71.I rent out for Php 10, 000 per
month my only building and lot to
the Jesus is Lord Group
which uses it exclusively for
religious purposes. I am exempt
from payment of
________________.
a. Property tax
b. Professional tax
c. Income tax
d. Community tax
72.Which is an unlawful act of
escaping from payment of taxes?
a. Evasion
b. Avoidance
c. Exemption
d. Shifting
73.What does capital in
economics refer to?
a. Investment and loss computed

b. Outcome of business transaction

c. Favoritism

b. Communistic

c. Money/ machines invested to


transact business

d. Racial discrimination

c. Capitalistic

76.The primary purpose of taxation


is to _____________________.

d. Bureaucratic

d. Profit and labor spent spent for


business
74.Which term refers to duties
payable on goods, whether
imported or exported?
a. Assessment
b. Tariff
c. Subsidiary
d. Revenue
75.The expression promdi connotes
______________________.
a. Ethnocentrism
b. Xenocentrism

a. Raise revenue for the support of


the government.
b. Reduce inequalities in wealth
and incomes.
c. Fortify the government against
invaders.
d. Make the country a leading
industrialized country in the world.
77.How do you describe a typical
social structure which describes a
pattern through which
relationships at work are ordered?
a. Technological

78.When parents decide to send


their children to a private school
and pay their tuition fee
instead of sending them to public
school, they dont get their share in
their benefit
derived from the taxes they pay.
This is a case of
____________________.
a. Social injustice
b. Double jeopardy
c. Double taxation
d. Social inequity
10

You might also like